Download as pdf or txt
Download as pdf or txt
You are on page 1of 93

for More Books Click HERE -> NEETpassionate.

com
Electrostatic Potential and Capacitance 179

66 (a) In given figure, C 2 and C 3 are in parallel. When we ground the charged body, potential difference
between body and ground is zero but not the charge and same
C2 = 1.0 µF for negatively charged body.
C1 = 4.0 µF
68 (a) The work done against the force of repulsion in moving
the two charges closer, increases the potential energy of the
C3 = 3.0 µF system.
A
C4 = 6.0 µF
69 (b) Earth is a conducting sphere of large capacitance.
B
V = q /C
As, C is very large, soV → 0 for all finite charges.
C6 = 8.0 µF
C5 = 2.0 µF Hence, earth is a good conductor.
70 (a) A current flows in the circuit during the time, the
∴ C ′ = C 2 + C 3 = 4 µF capacitor is charged. After the capacitor gets fully charged,
1 1 1 1 1 the current stops flowing. It means when a capacitor is
As C′ and C1 are in series, = + = + connected to a battery, a current flows in the circuit for
C S1 C ′ C1 4 4
sometime, then reduces to zero.
⇒ C S1 = 2 µF
1
Similarly, C 4 and C 5 are in parallel, C′′ = 6 + 2 = 8 µF 71 (c) For a point charge,V ∝
r
1 1 1 1 1 1 p
C′′ and C 6 are in series, = + = + For a dipole,V = ⋅ [on axial position]
C S 2 C ′′ C 6 8 8 4πε 0 r 2
⇒ C S 2 = 4 µF 1
Now, C S1 and C S 2 are in parallel, C = 4 + 2 = 6 µF i.e. V∝
r2
67 (d) A positively charged body can have positive, negative or The electric dipole potential varies as 1/r at large distance is
zero potential. not true.
for More Books Click HERE -> NEETpassionate.com

CHAPTER
03

Current Electricity
We have read that the directional flow of charge in a conductor under a potential
difference maintained between the ends of the conductor, constitutes an electric
current in the conductor. In this chapter, we will study about the basic properties
of electric current, property of batteries and how they cause current and energy
transfer in a circuit. In this chapter, we will use the concepts of current, potential
difference, resistance and electromotive force.

ELECTRIC CURRENT
Electric current is defined as the rate of flow of charge through any
cross-sectional area of the conductor. It is denoted by I.
If electric current is steady, then it can be expressed as
Total charge flowing (q ) q ne
I= ⇒ I = =
Total time taken (t ) t t
where, n = number of carriers (electrons) of electricity and e = electronic charge.
Inside
But current is not always steady, so it can be defined in two ways
(i) Average current If a charge ∆q flows through a conductor in the time 1 Electric current
∆q Current density
interval t to (t + ∆t ), then average current is defined as I av = . Electric current in conductors
∆t
2 Ohm's law
(ii) Instantaneous current Current at any instant of time is called Resistance and resistivity
instantaneous current. If a charge dq flows through a conductor in small Colour code for carbon resistor
∆q dq Combination of resistances
time dt, i.e. limit of ∆t tending to zero, then I inst = lim = .
∆t → 0 ∆ t dt Cells, emf and internal
resistance
Grouping of cells
Important points about electric current
3 Kirchhoff’s laws
(i) Current is a scalar quantity. Electrical energy and power
(ii) Its SI unit is ampere (A) and CGS unit is emu and is also called biot (Bi) or Heating effects of current
 1 Power consumption in a
ab ampere, i.e. |A | =   Bi (ab amp). combination of bulbs
 10  4 Measuring instruments for
current and voltage
(iii) One ampere is the current through a wire, if a charge of one coulomb flows
Wheatstone’s bridge
through any cross-section of the wire in one second, i.e.
Meter bridge
1 coulomb (C )
1 ampere (A) = = 1coulomb per second = 1 Cs −1 Potentiometer
1 second (s)
for More Books Click HERE -> NEETpassionate.com
Current Electricity 181

(iv) As a matter of convention, the direction of flow of Example 3.2 10 20 electrons, each having a charge of
positive charge gives the direction of current. This is 1.6 × 10 −19 C , pass from a point A towards another point B
called conventional current. The direction of flow in 0.1 s. What is the current in ampere? What is its
of electrons gives the direction of electronic current. direction?
Therefore, the direction of electronic current is Sol. Given, n = 1020, e = 1. 6 × 10− 19 C and t = 0.1 s
opposite to that of conventional current. As we know,
q ne (1020 × 1.6 × 10−19 ) C
I I
Current, I = = = = 160 A
t t 0.1 s
Direction of current is from point B to A (in the direction
opposite to flow of electrons).
Fig. 3.1 Flow of electronic current is equivalent to the
flow of conventional current in opposite direction Example 3.3 A wire carries a current of 2 A. What is the
charge that has flowed through its cross-section in 1.0 s?
(v) As charge is conserved and current is the rate of
How many electrons does this correspond to?
flow of charge, i.e. charge entering per second at one
end of conductor is equal to the charge leaving per Sol. Given, i = 2 A and t = 1.0 s
second the other end. As, current, i =
q
Note t
(i) In case, a charge q is revolving in a circle of radius r with uniform ∴ q = it = 2 × 1 = 2 C
velocity v, current is given by
q q qv Also, q = ne
i = = qf = = q 2
t 2 π / ω 2 πr ∴ Number of electrons, n = = = 1.25 × 1019
(ii) Total charge in time interval t 1 to t 2 can be given as e 1.6 × 10–19
t2
Q= ∫t
1
I ⋅ dt Example 3.4 If an electron revolves in a circle of radius
Area under the graph I versus t in the interval t 1 to t 2 as shown in π/2 cm with uniform speed 6 × 10 5 m/s. Find the electric
the figure current. (Take, π 2 = 10)
I Sol. Time period is given by
2 πr
t=
v
 π π
Here, r =   cm = × 10−2 m and v = 6 × 105 m/s
t2 t1  2 2
t
ev 1.6 × 10−19 × 6 × 105
Fig. 3.2 Area under I versus t graph So, electric current, i = =
2πr π
So, average current in this interval, 2π × × 10−2
2
t2
Q ∫t I ⋅ dt = Area under the I versus t graph = 9.6 × 10−13 A
I av = = 1
t 2 − t1 t 2 − t1 Time interval
Example 3.5 If the amount of charge flowed in time t through a
Example 3.1 How many electrons pass through a bulb in 1 min, if cross-section of wire is q = βt − γt 2, where β and γ are
− 19
the current is 400 mA? (Take, e = 1.6 × 10 C) constants.
−3 (i) Find the current in terms of t.
Sol. Given, current, I = 400 mA = 400 × 10 A
(ii) Sketch i versus t graph.
Time, t = 1 min = 60 s
Charge, e = 1.6 × 10− 19 C Sol. Given, q = β t − γt 2
Thus, charge passing through a bulb in 1 min is (i) As we know, current in a wire is given by
dq d
q = I × t = 400 × 10− 3 × 60 = 24 C i= = (β t − γt 2 ) = β − 2γt
dt dt
Also, q = ne (ii) To plot a graph i versus t, we will compare the given
∴ Number of electrons, current with the equation of a straight line, i. e .
q 24 y = mx + c
n= =
e 1.6 × 10−19 So, at t = 0, i = β
β
= 1. 5 × 1020 electrons and when i = 0, then t = .

for More Books Click HERE -> NEETpassionate.com
182 OBJECTIVE Physics Vol. 2

Area=A Area=A
which means the graph for given current equation is a
straight line with negative slope, i. e . −2γ and positive I I
intercept, i. e .β.
A An
θ
β
A
(a) (b)
Fig. 3.3 Direction of current density
i
If the cross-sectional area A is not perpendicular to the
O
t β/2γ current and normal to the area makes an angle θ with the
So, the plot will be as shown alongside. direction of the current as shown in Fig. 3.3 (b), then
An = A cosθ
Example 3.6 The current in a wire varies with time
I I
according to the relation, ∴ Current density, J = = or I = J ⋅A
i = 3 (A) + 2 (A / s ) t An A cosθ
(i) How many coulombs of charge pass through a cross-section If area is finite,
of the wire in the time interval between t = 0 to t = 4 s ?
(ii) What constant current would transport the same charge in I = ∫ J ⋅ dA
the same time interval?
Sol. Given, i = 3(A) + 2(A/s) t
where, dA is a small element of the given surface.
(i) Current through a cross-sectional area is given by The SI unit of current density is ampere per square metre,
dq i.e. Am− 2 and its dimensional formula is [AL −2 ].
i=
dt Example 3.7 An electron beam has an aperture of 2 mm 2 . A
∴ dq = idt total of 7 × 1016 electrons flow through any perpendicular
On integrating both sides between the given time interval, cross-section per second. Calculate the current density in the
i.e. 0 to 4 s, we get electron beam.
q 4
Sol. As we know, current density in the electron beam,
∫ dq = ∫ idt I ne ne
0 0 J= = /A =
4 A t tA
4  2t 2 
∴ Charge, q = ∫ (3 + 2t ) dt = 3t +  Substituting the values, we get
0
 2 0
7 × 1016 × 1.6 × 10−19
4 J =
= [3t + t 2] 0 = [12 + 16] = 28 C 1 × 2 × 10−6
= 5. 6 × 103Am−2
(ii) Therefore, the current flowing through wire would be
q 28
i= = = 7A Electric current in conductors
t 4
The electric current in a conductor can be explained by
Current density the electron theory. In an atom of a substance, the
electrons in the orbits close to the nucleus are bound to it
Current density at any point inside a conductor is defined
under the strong attraction of the nuclear positive charge,
as ‘‘the amount of charge flowing per second through a
but the electrons far from the nucleus experience a very
unit area held normal to the direction of the flow of charge
feeble force.
at that point’’.
Hence, the outer electrons can be removed easily from the
Current density is a vector quantity and its direction is
atom (by rubbing or by heating the substance). In fact, a
along the motion of the positive charge as shown in
few outer electrons, leave their atoms and move freely
Fig. 3.3 (a).
within the substance (in the vacant spaces between the
q /t I atoms). These electrons, called free electrons or
Current density, J = = conduction electrons, carry the charge in the substance
A A
from one place to the other.
for More Books Click HERE -> NEETpassionate.com
Current Electricity 183

Therefore, the electrical conductivity of a solid substance v d eτ


µ= =
depends upon the number of free electrons in it. In metals, E m
this number is quite large (≈ 10 29 /m3 ). Hence, metals are Its unit is m 2 /V-s.
good conductors of electricity. Silver is the best conductor Mobility of free electrons is independent of electric field.
of electricity than are copper, gold and aluminium,
respectively. Example 3.8 Find the current flow through a copper wire of
There are some other materials in which the electrons will length 0.2 m, area of cross-section 1 mm 2 , when connected
be bound and they will not be accelerated, even if the to a battery of 4 V. (Take, electron mobility is
electric field is applied, i.e. no current flow on applying 4.5 × 10 −6 m 2V −1s −1, charge on an electron is 1.6 × 10 −19 C
electric field. Such materials are called insulators, e.g. and density of electrons in copper is 8.5 × 10 28 m −3 )
wood, plastic, rubber, etc. Sol. Given, length of copper wire, l = 0.2 m
Cross-sectional area, A = 1 mm2 = 10−6m2
Drift velocity Potential difference,V = 4 V
It is defined as ‘‘the average velocity with which the free Electron mobility, µ = 4.5 × 10−6 m2V −1s−1
electrons in a conductor get drifted towards the positive Charge on an electron, e = 1.6 × 10−19 C
end of the conductor under the influence of an electric
field applied across the conductor’’. Density of electrons in copper, n = 8.5 × 1028m−3
We know that, electric field set up across the conductor,
eE 4
It is given by v d = τ E= =
V
= 20 Vm−1
m l 0.2
where, e = charge on electron, E = electric field, v
Mobility of electrons, µ = d
m = mass of the electron and τ = relaxation time. E
The electric current relates with drift velocity as ⇒ v d = 4.5 × 10−6 × 20 = 9 × 10−5 m/s
i = neAv d So, the current flow through the copper wire is
I = nev d A = 8.5 × 1028 × 1.6 × 10−19 × 9 × 10−5 × 10−6
i J
⇒ vd = = = 1.224 A
neA ne
Example 3.9 An aluminium wire of diameter 0.24 cm is
Hence, current density is also given by connected in series to a copper wire of diameter 0.16 cm.
i The wire carry an electric current of 10 A. Find (i) current
J = = nev d density in the aluminium wire (ii) and drift velocity of
A electrons in the copper wire.
The direction of drift velocity for electrons in a metal is (Take, number of electrons per cubic metre volume of
opposite to that of applied electric field. The drift velocity copper, n = 8.4 × 10 28 )
of electron is very small of the order of 10 −4 ms −1 as 0.24
Sol. (i) Radius of Al wire, r = = 0.12 cm = 0.12 × 10−2 m
compared to thermal speed (≈ 10 5 m/s) of electron at room 2
Area of cross-section of Al wire, A = πr 2
temperature.
= 3.14 × ( 0.12 × 10− 2 )2
Relaxation time ( τ )
= 4. 5 × 10− 6 m2
As free electrons move in a conductor, they continuously
I 10
collide with positive ions. The time interval between two ∴ Current density, J = = = 2.2 × 106 Am−2
successive collisions of electrons with the positive ions in A 4.5 × 10−6
the metallic lattice is defined as relaxation time. 016
.
(ii) Radius of copper wire, r = cm = 0.08 × 10−2 m
mean free path λ 2
τ= = Area of cross-section of copper wire,
rms velocity of electrons v rms
. × (0.08 × 10− 2 )2 = 2 × 10− 6 m2
A = πr 2 = 314
With rise in temperature v rms increases, consequently τ
decreases. Also, n = 8.4 × 1028 m−3, e = 1.6 × 10−19 C, I = 10 A
∴ Drift velocity of electrons in the copper wire,
Mobility (µ)
I 10
Drift velocity per unit electric field is called mobility of vd = =
enA 1.6 × 10 × 8.4 × 1028 × 2 × 10−6
−19
electron.
= 3.7 × 10−4 ms −1
for More Books Click HERE -> NEETpassionate.com
184 OBJECTIVE Physics Vol. 2

Example 3.10 What is the drift velocity of electrons in a So, number of silver atoms in this can be calculated by
silver wire of length 2 m, having cross-sectional area multiplying the number of moles by Avogadro’s number.
6.14 × 10 −6 m 2 and carrying a current of 5A? i.e., number of silver atoms = 6.9 × 104 × 6.023 × 1023
(Take, atomic weight of silver = 108, density of silver
= 7.5 × 10 3 kg/m 3 , charge on electron = 1.6 × 10 −19 C and −~ 4.16 × 1028 atoms
Avogadro’s number = 6.023 × 10 26 per kg atom) Now, since the valency of silver is one, we can assume each
atom of silver contributes one electron. So, finally
Sol. First, we will calculate, n = number of electrons per unit
volume. ⇒ n = 4.16 × 1028 per m3
Imagine the volume of silver to be 1 m3. Given, I = 5 A and A = 6.14 × 10−6 m2
Now, its mass = density × volume = 7.5 × 103 × 1 We use the formula, drift velocity,
= 7. 5 × 103 kg I 5
vd = =
Now, number of moles of silver, neA 4.16 × 1028 × 1.602 × 10−19 × 6.14 × 10−6
7. 5 × 103 = 1.22 × 10−4 m/s
= −3
= 0. 069 × 106 = 6. 9 × 104 mol
108 × 10

CHECK POINT 3.1


1. The current through a wire depends on time as (a)1.25 × 1019 (b) 6.25 × 1020
I = 3 t + 2t + 5. The charge flowing through the
2
(c) 5.25 × 1019 (d) 2.25 × 1020
cross-section of the wire in time interval between t = 0
5. Drift velocity v d varies with the intensity of electric field as
to t = 2 s is
(a) 22 C (b) 20 C (c) 18 C (d) 5 C per the relation,
1
(a) vd ∝ E (b) vd ∝
2. The charge on an electron is1.6 × 10−19 C. How many E
electrons strike the screen of a cathode ray tube each (c) vd = constant (d) vd ∝ E 2
second when the beam current is 16 mA?
(a) 1017 (b) 1019 6. When current flows through a conductor, then the order of
(c) 10−19 (d) 10−17 drift velocity of electrons will be
(a) 1010 cms −1 (b) 10−2 cms −1
3. A conductor carries a current of 0.2 A. In 30 s, how many (c) 104 cms −1 (d) 10−1 cms −1
electrons will flow through the cross-section of the
conductor? (Take, q =1.6 × 10 −19 C) 7. The number density of free electron in a copper conductor is
19 19 8.5 × 10 28 m−3. How long does an electron take to drift from
(a) 0.375 × 10 (b) 375 × 10
one end of a wire, 3.0 m long to its other end? The area of
(c) 3.75 × 1019 (d) 37.5 × 1019
cross-section of the wire is 2.0 × 10 −6 m2 and it is carrying a
4. In a closed circuit, the current I (in ampere) at an instant of current of 3.0 A.
time t (in second) is given by I = 4 − 0.08 t. The number of (a) 2.73 × 104 s (b) 4.73 × 104 s
electrons flowing in 50 s through the cross-section of the (c) 5 × 104 s (d) 6 × 108 s
conductor is

OHM’S LAW
It states that, “the current I flowing through a conductor is T1
always directly proportional to the potential differenceV T2
V V 1
across the ends of the conductor”, provided that the 2
physical conditions (temperature, mechanical strain, etc)
are kept constant. θ θ1 θ2
I I
Mathematically, I ∝V (a) Slope of the line (b) Here, tan θ1 > tan θ2
or V ∝I = tan θ = V = R So, R1 > R2
I i.e. T1 > T2
or V = IR Fig. 3.4
where, R is resistance of the conductor. Note The substances which obey Ohm’s law, e.g. cooper, silver,
Graph betweenV and I for a metallic conductor is a aluminium are known as ohmic substances. The devices or
straight line as shown. At different temperature, V-I substances which don’t obey Ohm’s law, e.g. gases, crystal
rectifiers, thermionic valve, transistors, etc., are known as
curves are different. non-ohmic or non-linear conductors.
for More Books Click HERE -> NEETpassionate.com
Current Electricity 185

Resistance and resistivity Conductance and conductivity


The property of a substance by virtue of which it opposes Conductance
the flow of current through it, is known as the resistance.
Reciprocal of resistance is known as conductance G, i.e.
It is defined as ‘the ratio of the potential difference applied
across the ends of the conductor to the current flowing 1
G=
through’’. R
V
Mathematically, R = Its unit is 1/Ω or Ω −1 or Siemen.
I
The resistance of the conductor depends upon the
following factors tan θ = G = 1
I R
(i) It is directly proportional to the length of the conductor,
i.e. R ∝l ...(i)
θ
(ii) It is inversely proportional to the area of the
cross-section of the conductor, V
1 Fig. 3.5
i.e. R ∝ ...(ii)
A Conductivity
From Eqs. (i) and (ii), we get
Reciprocal of resistivity is called conductivity (σ), i.e.
l l
R ∝ ⇒ R =ρ ...(iii)
A A 1
σ=
where, ρ is the constant of proportionality known as ρ
resistivity or specific resistance of the conductor.
Its unit is mho/m.
Resistivity is the intrinsic property of the substance. It is
independent of shape and size of the body and depends The dimensional formula of conductivity is [M−1L−3 T 3A 2 ].
upon the nature of the material.
Vector form of Ohm’s law
Example 3.11 Resistance of a conductor of length l and area
of cross-section A is R. If its length is doubled and area of
Electric field in a conductor of length l and having
cross-section is halved, then find its new resistance. potential differenceV at its ends is given by
Sol. Initial length = l, area = A V = El
Iρ l
So, initial resistance, R = ρ
l Also, V = IR ⇒ El =
A A
A I
Final length, l ′ = 2l, area, A′ = ⇒ E = ρ
2 A
l′ 2l l
New resistance, R ′ = ρ =ρ = 4ρ = 4R ∴ E = ρJ
A′  A A
  or J =σE ...(iv)
 2
If its length is doubled and area of cross-section is halved, The above equation is the vector form of Ohm’s law.
then new resistance becomes four times the initial value. Also, current density,
Unit and dimensional formula of resistance  eEτ 
J = nev d = ne  
The SI unit of resistance is ohm and is denoted by symbol Ω.  m 
1 volt (V)  ne 2τ 
1 ohm (Ω ) = = 1 volt/ampere (or V/A) J =   E
1 ampere (A)  m 
The resistance of a conductor is said to be one ohm, if one
Comparing with Eq. (iv), we get
ampere of current flows, when a potential difference of
one volt is applied across the ends of the conductor. ne 2τ
Conductivity, σ =
Its dimensional formula is [ML2 T −3A −2 ]. m
1 m
Note Any conducting material that offers some resistance is known as and resistivity, ρ = =
resistor. σ ne 2τ
for More Books Click HERE -> NEETpassionate.com
186 OBJECTIVE Physics Vol. 2

Example 3.12 All the edges of a block with parallel faces are Example 3.15 A current of 1A is flowing through a copper
unequal and its longest edge is four times its shortest edge. wire of length 0.1m and cross-section 1 × 10 −6m 2 .
Find the ratio of the maximum to the minimum resistance (i) If the specific resistance of copper is 1. 7 × 10− 8 Ω-m, then
between different faces.
calculate the potential difference across the ends of the wire.
Sol. Let l = longest edge, b = shortest edge. (ii) Determine current density in the wire.
According to question, l /b = 4 (Take, density of copper = 8.9 × 103 kg m − 3, atomic weight
= 63.5 and N = 6.02 × 1026 per kg-atom)
h . m, A = 1 × 10−6 m2,
Sol. Given, I = 1 A, l = 01
ρ = 1.7 × 10−8 Ω-m, d = 8.9 × 103 kg m−3.
b ρl 1.7 × 10−8 × 01
.
(i) Resistance of wire is, R = =
l A 1 × 10−6
Resistance between different faces of block = 1.7 × 10−3 Ω
(i) Area, A1 = (l × b )
l ρh ∴ Potential difference,V = IR = 1 × 1.7 × 10−3
∴ Resistance, R1 = ρ =
A lb = 1.7 × 10−3 V
(ii) Area, A2 = (b × h ) I 1
(ii) Current density, J = = = 1 × 106 Am−2
ρl A 1 × 10−6
∴ Resistance, R 2 = = R max
bh
(iii) Area, A3 = (l × h ) Example 3.16 Find the time of relaxation between two
ρb collisions and free path of electrons in copper at room
∴ Resistance, R 3 = = R min temperature. (Take, resistivity of copper = 1.7 × 10 − 8 Ω-m,
lh
density of electrons in copper = 8.5 × 10 28 m − 3 , charge on an
R l 2 (4b )2 electron = 1.6 × 10 − 19 C, mass of electron = 91
. × 10 − 31 kg
Hence, the ratio of max = 2 = 2 = 16
R min b b and drift velocity of free electrons = 1.6 × 10 − 4 ms −1)
Sol. Given, ρ = 1. 7 × 10− 8 Ω-m, n = 8. 5 × 1028 m− 3,
Example 3.13 Calculate the electric field in a copper wire of
cross-sectional area 2.0 mm 2 carrying a current of 1A. The e = 1. 6 × 10− 19 C, m e = 9.1 × 10− 31 kg,
conductivity of copper is 6.25 × 10 7 Sm − 1. v d = 1. 6 × 10− 4 ms −1
2 − 6 2
Sol. Given, area, A = 2 mm = 2 × 10 m m
We know that, ρ = 2e
Current, I = 1 A ne τ
Conductivity, σ = 6.25 × 107 Sm− 1 ∴ Relaxation time,
I
As, current density, J = = σE me 9.1 × 10− 31
τ= = − 19 2
A 2
e nρ (1. 6 × 10 ) × 8. 5 × 1028 × 1. 7 × 10− 8
I 1
∴ Electric field, E = = = 2.5 × 10− 14 s
Aσ 2 × 10−6 × 6.25 × 107
Mean free path of electron (distance covered between two
= 8 × 10−3 Vm−1 collisions) = v d τ
Example 3.14 A copper wire has a resistance of 10 Ω and an = 1. 6 × 10− 4 × 2.5 × 10− 14
2
area of cross-section 1 mm . A potential difference of 10V
= 4 × 10− 18 m
exists across the wire. Calculate the drift velocity of
electrons, if the number of electrons per cubic metre in Example 3.17 A wire has a resistance R.
copper is 8 × 10 28 electrons.
(i) If radius of cross-section of a cylindrical wire is doubled,
Sol. Given, R = 10 Ω, A = 1mm2 = 10− 6 m2 then find the ratio of initial to final resistance.
V = 10 V, n = 8 × 1028 electrons/m3 (ii) If length of wire is increased by 10%, then find the
Now, electric current, I = enAv d percentage increase in its resistance.
Using Ohm’s law, (iii) If length of wire is increased by 20%, then find the
V V percentage increase in its resistance.
∴ = enAv d or v d =
R enAR
10
= r
1.6 × 10− 19 × 8 × 1028 × 10− 6 × 10
l
= 0.078 × 10− 3 ms −1 = 0.078 mm/s
for More Books Click HERE -> NEETpassionate.com
Current Electricity 187

Sol. (i) According to the question, we draw a figure given below where, ρ = resistivity at temperature T, ρ 0 = resistivity at
temperature T 0 and α = temperature coefficient of
resistivity.
2r ρ − ρ0
i.e. α=
l′ ρ 0 (T − T 0 )
We have, A1 l1 = A2 l 2 In terms of resistance,
πr 2l = π (2r )2l 2 ⇒ l 2 = l / 4 R ∝ρ
ρl ρl ρl / 4 1 ρl ∴ R = R 0 [1 + α (T − T 0 )]
Resistance, R1 = 2 , R 2 = 2 = =
πr A2 4πr 2 16 πr 2 R − R0
R ⇒ α=
Ratio of initial to final resistance, 1 = 16 R 0 (T − T 0 )
R2
where, R 0 = resistance of conductor at 0°C
(ii) Given, l1 = l , A1 = A, l 2 = l + 10% of l
and R t = resistance of conductor at t°C.
= l + 0.1l = 1.1l, A2 = A′
We have, A1 l1 = A2 l 2 ⇒ Al = A′ (1.1l ) Important points related to resistivity
Area, A′ =
A For metals, α is positive as their resistivity increases with
1.1 rise in temperature. The graph of ρ plotted against T would
ρl ρl ρ × l × 1.1 ρl be a straight line as shown in Fig. 3.6 (a).
Resistance, R = , R ′ = 2 = = (1.1)2
A A2 A / 1.1 A For semiconductors, α is negative as their resistivity
R′ decreases with rise in temperature. Variation in resistivity
= (1.1)2 = 1.21
R with the temperature for semiconductor is shown in
Percentage increase in resistance, Fig.3.6 (b).
R ′  The resistivity of alloys also increases with rise in
 − 1 × 100 = (1.21 − 1) × 100 = 21% temperature. The graph between resistivity and
R 
(iii) Given, l1 = l, A1 = A temperature is shown in Fig. 3.6 (c).
 20  6l ρ
l 2 = l 1 +  = , A2 = A′
 100 5
Resistivity

ρ0 P
We have, A1 l1 = A2 l 2
6l 5A
Al = A′ ⇒ A′ =
5 6
T0 T
ρl 2
Temperature
R 2 A2 l 2 A1 6l / 5 A 36 (a)
∴ = = ⋅ = ⋅ =
R1 ρl1 l1 A2 l 5A / 6 25
Resistivity

A1
Resistivity

Percentage increase in resistance,


R2   36 
 − 1 × 100 =  − 1 × 100 = 44%
 R1   25 
Temperature Temperature
(b) (c)
Temperature dependence of resistivity Fig. 3.6 Resistivity as the function of temperature
Resistivity of a material depends on the temperature. for (a) metals, (b) semiconductors and (c) alloys
1
As, ρ∝ Example 3.18 The resistance of a thin silver wire is 1.0 Ω at
τ 20°C. The wire is placed in a liquid bath and its resistance
When temperature increases in a conductor, average speed rises to 1.2 Ω. What is the temperature of the bath?
of free electrons increases, hence relaxation time (Take, α for silver = 3.8 × 10 −3 per °C)
decreases. Thus, resistivity increases. Sol. We know that, R (T ) = R 0[1 + α (T − T0 )]
Resistivity of a metal conductor is given by Here, R (T ) = 1.2 Ω, R 0 = 1.0 Ω
α = 3.8 × 10−3 per °C
ρ = ρ 0 [1 + α (T − T 0 )]
and T0 = 20° C
for More Books Click HERE -> NEETpassionate.com
188 OBJECTIVE Physics Vol. 2

Substituting the values, we get


Band D indicates the tolerance in percent as per the
1.2 = 1.0 [1 + 3.8 × 10−3 (T − 20° )]
indicated value. In other words, it represents the
or 3.8 × 10−3 (T − 20° ) = 0.2 percentage accuracy of the indicated value.
Solving this, we get T = 72.6° C The tolerance in the case of gold is ± 5 % and in silver is
±10 %. If only three bands are marked on carbon resistor,
Example 3.19 Resistance of platinum wire in a platinum
resistance thermometer at melting ice, boiling water and at a then it indicates a tolerance of 20%. (i.e. no colour)
hot bath are 6 Ω, 6.5Ω and 6.2 Ω, respectively. Find the R = AB × C ± D %, where D is tolerance.
temperature of hot bath. Sequence of colour code for carbon resistor
Sol. Given, R 0 = 6 Ω, R100 = 6.5 Ω and Rt = 6.2 Ω Letters as an Multiplier
Colour Figure (A, B)
The temperature of platinum resistance thermometer (t ) is aid to memory (C)
given by B Black 0 100
R − R0 6.2 − 6
t= t × 100° C = × 100° C B Brown 1 101
R100 − R 0 6.5 − 6
R Red 2 102
0.2
= × 100° C = 40° C O Orange 3 103
0.5
Y Yellow 4 104
Example 3.20 The temperature coefficient of resistance of a
wire is 0.00145 °C −1. At 100 °C, its resistance is 2 Ω. At G Green 5 105
what temperature, the resistance of the wire will be 3 Ω? B Blue 6 106
Sol. Using the relation, R = R 0[1 + αT ] V Violet 7 107
R1 = R 0[1 + αt1] G Grey 8 108
∴ 2 = R 0 [1 + 0.00145 × 100] ...(i) W White 9 109
and R 2 = R 0[1 + αt 2]
Note To remember the sequence of colour code following sentence
∴ 3 = R 0 [1 + 0.00145 × t 2] ...(ii) should be kept in memory. B B ROY Great Britain Very Good
On dividing Eq. (ii) by Eq. (i), we get Wife.
3 1 + 0.00145 × t 2
= Example 3.21 How will you represent a resistance of
2 1 + 0.00145 × 100 3700 Ω ± 10% by colour code?
or t 2 = 494.8° C = 767.8K Sol. The value of resistance = 3700 Ω ± 10%
⇒ R = 37 × 102 ± 10%
Colour code for carbon resistor The colour assigned to numbers 3, 7 and 2 are orange, violet
In electrical and electronic circuits, there are two major and red. For ± 10% accuracy, the colour is silver.
types of resistors, which are being used, i.e. wire wound Hence, the bands of colour on carbon resistor in sequence are
resistors and carbon resistor, offering resistances over a orange, violet, red and silver.
wide range. Wire wound resistors have resistances upto
few hundred ohm, whereas for higher resistances, resistors
are mostly made up of carbon.
Combination of resistances
To know the value of resistance of carbon resistors, colour There are two types of combination of resistances, i.e.
code is used. These codes are printed in the form of set of series combination and parallel combination.
rings or strips. By reading the values of colour bands, we
can estimate the value of resistance.
1. Series combination
The carbon resistor has normally four coloured rings or In this combination, resistors are connected end-to-end,
bands say A, B, C and D as shown in the Fig. 3.7. i.e. second end of first resistor is connected to first end of
the second resistor and so on.
A B C D
In series combination, same current flows through each
resistance but potential difference distributes in the ratio
Fig. 3.7 Carbon resistor of their resistance, i.e.V ∝ R . So, the total potential
difference is equal to the sum of potential difference applied
Colour bands A and B indicates the first two significant across the combination of resistors.
figures of resistance in ohm. Equivalent resistance in series combination is given by
Band C indicates the decimal multiplier, i.e. the number
of zeros that follows the two significant figures A and B. R eq = R 1 + R 2 + R 3
for More Books Click HERE -> NEETpassionate.com
Current Electricity 189

i.e. equivalent resistance is greater than the maximum As shown in figure, the two resistances of value 3R each are
value of resistance in the combination. in parallel with each other. So, the resistance between the
ends A and B of a diameter is
R1 R2 R3
R1 × R 2 3R × 3R 9R 2 3
R′ = = = = R
V1 V2 V3 R1 + R 2 3R + 3R 6R 2
I

Example 3.23 Find the equivalent resistance between A and


V B in the following cases.
Fig. 3.8 Resistances in series 8Ω 4Ω
C
A
If n identical resistors of resistance R each, are connected
in series, then equivalent resistance, R eq = nR and 20 Ω 30 Ω
(i) (ii) 3Ω 1Ω
potential difference across each resistor isV ′ = V /n.
B
2. Parallel combination A
50 Ω
B

In this combination, first end of all resistors are connected Sol. (i)
to one point and second end of all resistors are connected C 50W
to other point.

30
W
Same potential difference appears across each resistor but

W
20
A B
current distributes in the reverse ratio of their resistance.
V
i.e. I ∝ . So, the total current is equal to the sum of A
50W
B 50W
R
From the above dotted region,
currents through each resistance. From the above dotted region,
50 W and 50 W are in parallel,
20 W and 30 W are in series,
R1 50 ´ 50 2500
Þ R = 20 + 30 = 50 W Þ Req = = = 25 W
50 + 50 100
I1 I2 R2
8W
I3 R3 8W 4W A
I (ii)
A
3W 5W
3W 1W
V
B
Fig. 3.9 Resistances in parallel From the above dotted region,
B From the above dotted region, 3 W and 5W are in parallel,
Equivalent resistance in parallel combination is given by 4 W and 1W are in series, 1 1 1 1 1
Þ = + = + Þ R=
15
W
Þ R=4+1=5W R R1 R2 3 5 8
1 1 1 1
= + +
R eq R1 R 2 R 3 8W
A

i.e. equivalent resistance is less than the minimum value


of resistance in the combination. (15/8 )W
If n identical resistors of resistance R each, are connected
B
R
in parallel, then equivalent resistance, R eq = and From the above dotted region, we have
n 8 W and (15/8) W are in series,
I Þ Req = 15/8 + 8 = 79/8 W
current through each resistor is I ′ = .
n
Example 3.24 A letter ‘A’ consists of a uniform wire of
Example 3.22 A wire of resistance 6R is bent in the form of a resistance 0.2 ohm per cm. The sides of letter are each
circle. What is the effective resistance between the ends of 20 cm long and the cross-piece in the middle is 10 cm long
the diameter? while apex angle is 60°. Find the resistance of the letter
Sol. 3R between the two ends of the legs.
3R
Sol. Clearly it is given that,
Þ
A B A B AB = BC = CD = DE = BD = 10 cm
3R
3R and R1 = R 2 = R 3 = R 4 = R 5 = 2 Ω
for More Books Click HERE -> NEETpassionate.com
190 OBJECTIVE Physics Vol. 2

As R 2 and R 3 are in series, their combined resistance Now, R ′′ and 3 Ω are in series,
=2+2= 4Ω 40 79
R ′′′ = R ′ ′ + 3 =+3= Ω
This combination is in parallel with R 5 (= 2 Ω ). 13 13
Hence, resistance between points of B and D is given by Now, R ′′′ and 1 Ω are in parallel, so resistance between A and
1 1 1 B,
= +
R 4 2 1 × 79 /13 79
(R ′′′′ ) = = Ω
1 6 1 + 79 /13 92
⇒ =
R 8
4 Example 3.26 Find the effective resistance between A and B
⇒ R= Ω for the network shown in the figure below
3
C D E
2Ω 2Ω 2Ω
R2 R3 C 4Ω 4Ω
F
2Ω 2Ω
B D 4Ω
R5 A B
R1 R4 2Ω

A E Sol. At points A and D, a series combination of 2 Ω, 2 Ω


resistance (along AC and CD) is in parallel with 4 Ω resistance
Now, resistance R1, R, R 4 form a series combination. So,
resistance between the ends A and E is (along AD), therefore resistance between A and D
4 1
R ′ = 4 + + 4 = 9.34 Ω = =2Ω
1 1
3 +
2+2 4
Example 3.25 In the given network of resistors, find the
Similarly, proceeding this way the resistance between A and F
equivalent resistance between the points A and B and
1
between the points A and D. = =2Ω
1 1
6Ω +
D C 2+2 4
Finally, resistance between A and B
2Ω 5Ω 3Ω 1 4
= = Ω
1 1 3
+
A
1Ω
B 2+2 2
4
Sol. Resistance between A and D, Thus, the effective resistance between A and B is Ω.
3
As we can see from the circuit, 1 Ω and 3 Ω are in series,
⇒ R1 = 1 + 3 = 4 Ω Example 3.27 Find the equivalent resistance between A
and B.
4 × 5 20
Now, R1 and 5 Ω are in parallel, R 2 = = Ω 4Ω 2Ω 1Ω
4+5 9 A
74
Now, R 2 and 6 Ω are in series, R 3 = 20 / 9 + 6 = Ω
9 9Ω 5Ω 1Ω
74
×2 B
37
Now, R 3 and 2 Ω are in parallel, R 4 = 9 = Ω 4Ω 3Ω 1Ω
 74  23
 + 2 Sol. 4Ω 2Ω 1Ω
 9  A
Resistance between A and B,
The resistors AD (= 2 Ω ) and DC (= 6 Ω ) are in series to give a 9Ω 5Ω 1Ω
total resistance R ′ = 8 Ω. The resistance R ′ (= 8 Ω ) and the
resistor AC (= 5 Ω ) are in parallel. Their equivalent B
5 × 8 40 4Ω 3Ω 1Ω
resistance is, R ′′ = = Ω
5 + 8 13
for More Books Click HERE -> NEETpassionate.com
Current Electricity 191

As per the dotted region, 1 Ω, 1 Ω and 1 Ω are in series,


R =1+1+1= 3 Ω B
4Ω 2Ω 6R
A R 9R
(iii)
A O
9Ω 5Ω 3Ω 12R

B
4Ω 3Ω
Sol. (i) The points connected by a conducting wire are at same
In dotted region, 3W and 5 W are in parallel, potential. Then, redraw the diagram, by placing the
Þ R1 = 3 ´ 5 = 15 W points of same potential at one place and then solve for
3+5 8 equivalent resistance.
4W 2W
A

6R X 9R 12R
9W (15/8 )W A B
Y
B
4W 3W

In dotted region, 2 W, 3W and (15/8) W are in series,


Þ R2 = 2+3+ 15 = 55 W 6R Parallel
8 8
4W
A
9R
A, Y B, X
9W 55 W
8
12 R
B
4W
In dotted region, (55/8) W and 9 W are in parallel,
Hence, from the new figure A and Y are at same potential;
55 ´ 9
8 B and X are at same potential.
Þ R3 = = 495 W
55 + 9 127 1 1 1 1 13
⇒ = + + =
8 R eq 6R 9R 12R 36 R
4W
A 36R
R eq = = 2.77R
495 13
W
127 B
B B
4W 2R
In dotted region, 4 W, 4W and (495/127) W are in series, 2R A 2R
(ii) B B
Þ Req = 4 + 4 + (495/127) Þ Req = 1511 W
127
2R
Example 3.28 Find the equivalent resistance between A and B.
B
Similarly, placing the points of same potential at one
(i) A B place, then the equivalent resistance is
6R 9R 12R
2R

2R
A B
B
2R 2R
2R A 2R
(ii)
2R
2R In parallel, Req = 2R = 0.5R
4
for More Books Click HERE -> NEETpassionate.com
192 OBJECTIVE Physics Vol. 2

B 6R V 12 × 3
So, current supplied by battery, I = = = 9A
B R 4
6R
R 9R (ii)
R 2W
(iii) A B A B
O 9R 6W
12R 12 R X Y
4W
B I

1 1 1 1 13 36
In parallel, = + + = ⇒ R′ = R
R ′ 6R 9R 12R 36R 13 1W
20 V

In dotted region, resistances are in parallel,


R 36R/13 4×2 8 4
A B ⇒ R1 = = = Ω
4+2 6 3
36R 49 6W 4/3 W
In series, R eq = R + = R = 3.77R Y
13 13 X
Example 3.29 (i) Determine the current supplied by the
battery in the circuit as shown
4Ω

20 V 1W
12 V 8Ω 4Ω 8Ω
All resistance in the circuit are in series,
4 25
R eq = 6 + 1 + = Ω
3 3
(ii) Find currents in resistances 2 Ω and 4 Ω.
V 20 × 3 12
2Ω ∴ Current supplied by battery, I = = = A
6Ω R 25 5
In order to calculate the current in resistance 2 Ω and 4 Ω,
4Ω we can redraw the circuit as
I1 2W

I = (12/5) A
20 V 1Ω
6W X Y
Sol. (i) Placing the points of same potential as shown
4W I2 4W
I A A A
(16/5) V
12 V 8W 4W 8W
Potential difference across
12  4 16
XY , VXY = IR1 =     = V
 5   3 5
B B B B
2 Ω and 4 Ω are parallel across XY . Hence, they have
All resistances in the circuit are in parallel, same potential.
1 1 1 1 1 V 16 8 V 16 4
⇒ = + + + ⇒ I1 = XY = = A and I 2 = XY = = A
R eq 4 8 4 8 2 5×2 5 4 5×4 5
2 +1+ 2 +1 6 3 Example 3.30 Calculate the current shown by the ammeter A
=
= =
8 8 4 in the circuit shown in figure.
R eq = (4 /3) Ω 4Ω
Now, the circuit becomes 8Ω 8Ω
8Ω 8Ω
I
4Ω +
12 V A
4/3 W

12V
for More Books Click HERE -> NEETpassionate.com
Current Electricity 193

Sol. The given circuit can be redrawn as


8R
8Ω 4Ω 8Ω
4r
8Ω 8Ω P Q
4Ω + 8R
A

All resistances in the circuit are in parallel,


12V 1 1 1 1 R +r
= + + =
From the above figure, the two 8 Ω resistances are connected R eq 8R 4r 8R 4Rr
8×8 4Rr
in parallel, so equivalent resistance, R eq = = 4 Ω. ⇒ R eq = Ω
8+8 R +r
These two combinations are connected in series, so equivalent (ii) Similarly as in (i), we see that there is symmetry about
resistance = 4 + 4 = 8 Ω. PQ and X,Y and Z are at same potential. So, remove
Now, we have resistances of 4 Ω, 8 Ω and 4 Ω connected in resistances between X andY ;Y and Z.
parallel, so Series, 4R
P
1 1 1 1 5 8 2R
⇒ = + + = or R = Ω P
2R
R 4 8 4 8 5 Z
2R 2R
Also, V = 12V (given)

2R
V 12 15
∴ Current, I = = = = 7.5 A 2R 2R 2R 2R

Se
R 8/5 2 Y

rie

2R
s,
2R 2R

4
Example 3.31 Find the equivalent resistance between P and Q.

R
X
2R
(i) 4R 4R Q
2R
R Series, 4 R Q

P Q
2r 2r P
R

4R
4R 4R

4R
(ii) P 2R
4R

2R 2R

2R 2R Q

2R 2R In parallel, Req = 4R W
3
2R Q Example 3.32 Find the equivalent resistance between A and B.
Sol. (i) It can be seen that, this diagram is symmetrical about PQ, R R R
A ∞
so points on the perpendicular bisector of PQ, i.e. X,Y and Z
are at same potential. So, in this type of diagrams, to calculate 2R 2R 2R
the equivalent resistance, we can remove the resistances at
the same potential, i.e. the resistances between X andY ,Y B ∞
and Z, are redundant and can be removed. Sol. R C R R
A ∞
Series, 8R
4R X 4R 4R 4R
2R 2R 2R
R Series, 4r
Y 2r 2r B ∞
P Q P Q D
2r 2r
Here, we have infinite pairs of R and 2R. Suppose, the
R
equivalent resistance is R 0 between C and D, i.e. excluding
4R Z 4R 4R 4R one pair near AB (since, pairs are infinite, equivalent
Series, 8R resistance will remain same, if we include pair near AB).
for More Books Click HERE -> NEETpassionate.com
194 OBJECTIVE Physics Vol. 2

R R
A
A
Parallel, Terminal potential difference (V )
2R . R0 2R . R0
2R R0 = Terminal potential difference of a cell is defined as the
2R + R0 2R + R0
potential difference between the two terminals of the cell
B B in a closed circuit (i.e. when current is drawn from the
2RR 0 cell). It is represented by V and its unit is volt.
⇒ R eq = R0 = R +
2R + R 0 Terminal potential difference of a cell is always less than
(R 0 − R )(2R + R 0 ) = 2RR 0 the emf of the cell (i.e.V < ε). In closed circuit, the current
flows through the circuit including the cell, due to internal
2RR 0 − 2R 2 + R 02 − RR 0 = 2RR 0 resistance of the cell, there is some fall of potential.
R 02 − RR 0 − 2R 2 = 0 This is the amount of potential by which the terminal
potential difference is less than the emf of the cell.
R ± R 2 + 8R 2 R ± 3R
R0 = = = 2R or − R Relation between terminal potential difference, emf
2 2
Equivalent resistance between A and B = 2R (Q equivalent and internal resistance of a cell
resistance cannot be negative). (i) If no current is drawn from the cell, i.e. the cell is in
open circuit, then emf of the cell will be equal to the
Cells, emf and internal resistance terminal potential difference of the cell.
An electric cell is a source of energy that maintains a I = 0 or V = ε
continuous flow of charge in a circuit. Electric cell (ii) Consider a cell of emf ε and internal resistance r is
changes chemical energy into electrical energy. connected across an external resistance R.
R
Electromotive force (emf) of a cell (ε)
Electric cell has to do some work in maintaining the I

current through a circuit. The work done by the cell in r


moving unit positive charge through the whole circuit +
ε

(including the cell) is called the electromotive force (emf) Fig. 3.10
of the cell. ε
Current drawn from the cell, I = …(i)
If during the flow of q coulomb of charge in an electric R +r
circuit, the work done by the cell is W, then V
Now from Ohm’s law, V = IR ⇒ I = …(ii)
W R
emf of the cell, ε = From Eqs. (i) and (ii), we have
q
V ε ε 
Its unit is joule/coulomb or volt. = ⇒ r =  − 1 R
R R +r V 
If W = 1 joule and q = 1 coulomb, then ε = 1 volt, i.e. if in
the flow of 1 coulomb of charge, the work done by the cell Charging of cell
is 1 joule, then the emf of the cell is 1 volt. During charging of a cell, the positive terminal (electrode) of the cell
is connected to positive terminal of battery charger and negative
Internal resistance (r) terminal (electrode) of the cell is connected to negative terminal of
battery charger.
Internal resistance of a cell is defined as the resistance
offered by the electrolyte of the cell to the flow of current + –
Charger
through it. It is denoted by r and its unit is ohm. I I

Internal resistance of a cell depends on the following factors ε


(i) It is directly proportional to the separation between Fig. 3.11
the two plates of the cell. In this process, current flows from positive electrode to negative
(ii) It is inversely proportional to the plates area dipped electrode of the cell. From figure above, V = ε + Ir
into the electrolyte. Thus, the terminal potential difference of a cell becomes greater than the
emf of the cell.
(iii) It depends on the nature, concentration and
The potential difference across internal resistance of the cell is called
temperature of the electrolyte and increases with lost voltage, as it is not indicated by a voltmeter. Its value is equal to
increasing the concentration of electrolyte. Ir.
for More Books Click HERE -> NEETpassionate.com
Current Electricity 195

Potential difference (V ) across the Sol. Total resistance = 10 + 2 + r = (12 + r ) Ω


terminals of a cell 10 Ω
(i) When the cell is in open circuit, R
Battery A 2Ω
∴ V = VP − VQ = E , i.e.V = E 2V r

I=0 + E – r
P Q Now, current = 50 mA = 50 × 10−3A and emf = 2 V.
emf
Fig. 3.12 So, resistance =
current
(ii) Discharging of cell when cell is supplying current, 2
12 + r = = 40
∴ V = VP − VQ = E − Ir , i.e.V < E 50 × 10−3
R ⇒ r = 40 − 12 = 28 Ω
Example 3.35 A voltmeter of resistance 994 Ω is connected
I
E across a cell of emf 1V and internal resistance 6 Ω. Find the
P + – Q potential difference across the voltmeter, that across the
r terminals of the cell and percentage error in the reading of
Fig. 3.13 the voltmeter.
(iii) Charging of cell when cell is taking current, Sol. Given, E = 1 V, r = 6 Ω
∴ V = VP − VQ = E + Ir , i.e.V > E and resistance of voltmeter, R = 994 Ω
Source Cell
1V 6 W

E I Voltmeter I
P I + – Q

Fig. 3.14
R = 994 W
(iv) When the cell is short circuited, i.e. external Current in the circuit is
resistance is zero.
E 1
⇒ I = E /r I= = = 1 × 10−3A
R + r (994 + 6)
∴ V = VP − VQ = E − Ir = 0, i.e.V = 0
The potential difference across the voltmeter is
V = IR = 1 × 10−3 × 994

E = 9.94 × 10−1V
+ –
P Q The same will be the potential difference across the terminals
I r
of the cell. The voltmeter used to measure the emf of the cell
Fig. 3.15
will read 0.994 V. Hence, the percentage error is
Example 3.33 The reading on a high resistance voltmeter E −V 1 − 0.994
× 100 = × 100 = 0.6%
when a cell is connected across it is 3 V. When the E 1
terminals of the cell are also connected to a resistance of
4 Ω, then the voltmeter reading drops to 1.2 V. Find the Example 3.36 Find the current drawn from a cell of emf 2 V
internal resistance of the cell. and internal resistance 2 Ω connected to the network given
below.
Sol. Given, E = 3 V, R = 4 Ω and V = 12
. V
A B
As, internal resistance, 1W
E −V   3 − 1.2 1W 1W
r = R  =4  Ω = 6Ω 1W 1W
 V   12 . 
1W
D C
Example 3.34 A battery of emf 2V and internal resistance r is
connected in series with a resistor of 10 Ω through an
ammeter of resistance 2 Ω. The ammeter reads 50 mA.
Draw the circuit diagram and calculate the value of r. 2 V, 2 W
for More Books Click HERE -> NEETpassionate.com
196 OBJECTIVE Physics Vol. 2

Sol. The equivalent circuit is shown below If external resistance is much higher than the total internal
resistance, then cells should be connected in series to get
1W
A B the maximum current.
1W 1W Note If dissimilar plates of cells are connected together, then their
P Q emf’s are added to each other while, if their similar plates are
1W 1W connected together their emf’s are subtracted. While their
internal resistances are always additive.
1Ω
I C I
D
E1 E2 E1 E2
2V r=2W
Eeq = E1 + E2 Eeq = E1 - E2 (when, E2 < E1)
req = r1 + r2 req = r1 + r2
Resistance in arm AB = 1 Ω Fig. 3.17
1×1 1×1 1 1
Resistance in arm PQ = + = + =1Ω Parallel grouping
1+1 1+1 2 2
Resistance in arm DC = 1 Ω In parallel grouping of cells, all anodes of cells are
These three resistances are connected in parallel. connected at one point and all cathodes of cells are
connected together at other point as shown below.
Their equivalent resistance R is given by
E, r
1 1 1 1 3
= + + =
R 1 1 1 1 E, r
1
or R= Ω E, r
3 I
Current drawn from the cell, R
E 2
I= =
R + r 1  Fig. 3.18
 + 2
3 
If n identical cells of emf E and internal resistance r each
2×3 6 are connected in parallel, then
= = A
7 7 (i) equivalent emf of the combination, E eq = E .
(ii) equivalent internal resistance, R eq = r /n.
Grouping of cells (iii) main current, I =
E
.
R + r /n
Series grouping I
In series grouping, anode of one cell is connected to (iv) current from each cell, I ′ = .
n
cathode of other cell and so on as shown below.
(v) potential difference across external resistance
E, r E, r E, r E, r = potential difference across each cell = V = IR.
If external resistance is much smaller than the total
I internal resistance, then cells should be connected in
R parallel to get the maximum current.

Fig. 3.16 Mixed grouping


If n identical cells of emf E and internal resistance r each, If n identical cells of emf E and internal resistance r each,
are connected in series, then are connected in a row and such m rows are connected in
(i) equivalent emf of the combination, E eq = nE . parallel as shown in Fig. 3.19, then
E, r E, r E, r
(ii) equivalent internal resistance of the combination,
1
r eq = nr . 1 2 n
2
nE
(iii) main current = Current from each cell = I = .
R + nr I m

(iv) potential difference across external resistance,V = IR . V

V
(v) potential difference across each cell,V ′ = . R
n Fig. 3.19
for More Books Click HERE -> NEETpassionate.com
Current Electricity 197

(i) Equivalent emf of the combination, Example 3.38 Find the emf and internal resistance of a
single battery which is equivalent to a combination of three
E eq = nE
batteries as shown in figure.
(ii) Equivalent internal resistance, 10 V 2 Ω
nr
r eq = 6V1Ω
m
(iii) Main current flowing through the load,
nE mnE
I = = 4V 2Ω
nr mR + nr
R+ Sol. The given combination consists of two batteries in parallel
m and resultant of these two in series with the third one.
(iv) Potential difference across load, For parallel combination, we can apply,
V = IR E1 E 2 10 4
(v) Current from each cell, – –
r1 r2
I E eq = = 2 2 =3V
I′ = 1 1 1 1
n + +
(vi) Potential difference across each cell, r1 r2 2 2
1 1 1 1 1
V Further, = + = + =1
V′ = req r1 r2 2 2
n
(vii) Total number of cells = mn ∴ req = 1 Ω
In mixed grouping of cells, the current through the Now, this is in series with the third one, i.e.
external resistance would be maximum, if the external 6V 3V
resistance is equal to the total internal resistance of the 1W 1W
nr
cells, i.e. R = . The equivalent emf of these two is (6 – 3) V or 3 V and the
m
internal resistance will be (1 + 1) Ω or 2 Ω, i.e.
Example 3.37. In the circuit shown in figure, E=3V
E1 = 10V , E 2 = 4V , r1 = r2 = 1 Ω and R = 2 Ω.
Find the potential difference across battery 1 and battery 2. r=2W

E1 r1 E2 Example 3.39 Two identical cells of emf 1.5 V each joined in


r2
parallel provide supply to an external circuit consisting of
two resistances of 17 Ω each joined in parallel. A very high
resistance voltmeter reads the terminal voltage of cells to be
R 1.4 V. Calculate the internal resistance of each cell.
Sol. Given, E = 1.5 V andV = 1.4 V
Sol. Net emf of the circuit = E1 + E 2 = 14 V
Resistance of external circuit = Total resistance of two
E1 r1 E2 r2
resistances of 17 Ω connected in parallel
RR 17 × 17
⇒ R= 1 2 = = 8.5 Ω
V1 V2 R1 + R 2 17 + 17
I 17Ω
R

17Ω
Total resistance of the circuit,
= R + r1 + r2 = 4 Ω I
I 1.5V
∴ Current in the circuit, r
Net emf 14
I= = = 3.5 A
Total resistance 4 1.5V r
Now, potential difference across battery 1,
V1 = E1 + Ir1 = 10 + (3.5)(1) = 13.5 V Let r ′ be the total internal resistance of the two cells.
and potential difference across battery 2, Then,
V2 = E 2 + Ir2 = 4 + (3.5)(1) = 7.5 V E −V  1.5 − 1.4
r′ = R  = 8.5   Ω = 0.6 Ω
 V   1.4 
for More Books Click HERE -> NEETpassionate.com
198 OBJECTIVE Physics Vol. 2

As the two cells of internal resistance r each have been Multiplying Eqs. (i) and (ii), we get
connected in parallel, therefore 0.5 n 2 = 72 or n 2 = 144
1 1 1 1 2
= + or = 36
r′ r r 0.6 r ∴ n = 12 and m = =3
12
or r = 0.6 × 2 = 12
. Ω
Thus, for maximum current there should be three rows in
Example 3.40 Find the minimum number of cells required to parallel, each containing 12 cells in series.
produce an electric current of 1.5 A through a resistance of mnE 36 × 1.5
30 Ω. Given that the emf of each cell is 1.5V and internal ∴ Maximum current = = = 4.5 A
mR + nr 3 × 2 + 12 × 0.5
resistance is 1.0 Ω .
nr Example 3.42 12 cells, each of emf 1.5V and internal
Sol. As, =R
m resistance of 0.5 Ω, are arranged in m rows each containing
n ×1 n cells connected in series, as shown. Calculate the values of
∴ = 30 or n = 30 m …(i)
m n and m for which this combination would send maximum
nE n × 1.5 current through an external resistance of 1.5 Ω.
Current, I = or 1.5 = or n = 60 …(ii)
2R 2 × 30 R =1.5Ω

From Eqs. (i) and (ii), we get


m = 60/30 = 2
m rows
and mn = 120
Example 3.41 36 cells, each of internal resistance 0.5 Ω and
emf of 1.5 V, are used to send maximum current through an (n cells in each row)
external circuit of 2 Ω resistance. Find the best mode of Sol. For maximum current through the external resistance,
grouping them and the maximum current through the
external resistance = total internal resistance of cells
external circuit. nr
or R=
Sol. Given, E = 1.5 V, r = 0.5 Ω and R = 2 Ω m
Total number of cells, mn = 36 …(i) n × 0.5
∴ 1.5 = (Q mn = 12)
For maximum current in the mixed grouping, 12
nr n × 0.5 n
= R or =2 … (ii)
m m or 36 = n 2 or n = 6 and m = 2

CHECK POINT 3.2


1. Calculate the amount of charge flowed in 2 min through a 5. The potential difference between points A and B of the
wire of resistance 10 Ω, when a potential difference of 20 V following figure is
is applied across its ends.
(a) 120 C (b) 240 C (c) 20 C (d) 4 C 5Ω 5Ω
A B
2. If e, τ and m respectively, represent the electron density,
relaxation time and mass of the electron, then the
2V
resistance R of a wire of length l and area of cross-section A 5Ω 5Ω
will be
ml mτ A
2
ne τ A
2 2
ne A
(a) (b) (c) (d) 5Ω 5Ω
ne 2τ A ne 2l 2ml 2mτ l D C

3. Four wires are made of the same material and are at the 2 8 4
(a) V (b) V (c) V (d) 2 V
same temperature. Which one of them has highest electrical 3 9 3
resistance?
(a) Length = 50 cm, diameter = 0. 5 mm 6. In the figure given below, the current passing through
(b) Length = 100 cm, diameter = 1 mm 6 Ω resistor is
(c) Length = 200 cm, diameter = 2mm
6Ω
(d) Length = 300 cm, diameter = 3mm
4. Carbon resistors, used in electronic circuits are marked for
their value of resistance and tolerance by a colour code. A 1.2A 4Ω
given carbon resistor has colour scheme brown, red, green
and gold. Its value in ohm is
(a) 0.40 A (b) 0.48 A
(a) 52 × 106 ± 10% (b) 24 × 105 ± 5%
(c) 0.72 A (d) 0.80 A
(c) 12 × 104 ± 10% (d) 12 × 105 ± 5%
for More Books Click HERE -> NEETpassionate.com
Current Electricity 199

7. In the figure given below, current passing through the 3 Ω 12. In the circuit shown, the point B is earthed. The potential at
resistor is 0.8 A, then potential drop through 4 Ω resistor is the point A is
3Ω 5Ω 7Ω B
4Ω A
6Ω
10 Ω

50 V
+ –
3Ω
(a) 9.6 V (b) 2.6 V (c) 4.8 V (d) 1.2 V
8. Current i as shown in the circuit will be
(a) 14 V (b) 24 V
(c) 26 V (d) 50 V
3Ω
13. A current of 2 A passes through a cell of emf 1.5 V having
internal resistance of 0.15 Ω. The potential difference
3Ω measured in volt, across both the ends of the cell will be
(a) 1.35 (b) 1.50
(c) 1.00 (d) 1.20
i
14. If V AB is 4V in the given figure, then resistance X will be
3Ω 10 V 10 Ω 5V

10
(a) 10 A (b) A (c) zero (d) infinite
3
A B
9. In the figure given below, the current flowing through 10 Ω 2V X
resistance is

20 V (a) 5 Ω (b) 10 Ω
(c) 15 Ω (d) 20 Ω
+ –
15. Two batteries of emf 4 V and 8 V with internal resistances
10 Ω 1 Ω and 2 Ω are connected in a circuit with a resistance of
3Ω 0.5 Ω 9 Ω as shown in figure. The current and potential difference
between the points P and Q are
15 Ω
1Ω 4V 8V 2Ω
P Q
(a) 12 A (b) 1.2 A (c) 0.8 A (d) 0.4 A r1 r2
10. If all the resistors shown have the value 2 Ω each, the
equivalent resistance over AB is 9Ω

A
B 1 1
(a) A and 3 V (b) A and 4 V
3 6
1 1
(c) A and 9 V (d) A and 12 V
9 2
2 2
(a) 2 Ω (b) 4 Ω (c) 1 Ω (d) 2 Ω 16. The current flowing through 5 Ω resistance is
3 3
5Ω
11. The current drawn from the battery in circuit diagram
shown is
2Ω A 7Ω
8V 4V
6V
15 V

6Ω 1Ω
0.5 Ω 9Ω

8Ω B 10 Ω (a) 10 A (b) 1 A
(c) 2.5 A (d) 0.4 A
(a) 1 A (b) 2 A (c) 1.5 A (d) 3 A
for More Books Click HERE -> NEETpassionate.com
200 OBJECTIVE Physics Vol. 2

KIRCHHOFF’S LAWS Kirchhoff’s loop rule


Many electric circuits cannot be reduced to simple This law states that the algebraic sum of change in
series-parallel combinations. Kirchhoff’s laws (or rules) potential around any closed loop involving resistors and
are used to solve these complicated electric circuits. cells in the loop, is zero. It means that, in any closed part
These rules are basically the expressions of of an electrical circuit, the algebraic sum of the emfs is
conservation of electric charge and of energy. e.g. two equal to the algebraic sum of the products of the
circuits that cannot be broken down are shown in resistances and currents flowing through them. It is also
Fig. 3.20. known as loop rule. i.e. Σ ∆V = 0
R1 E1
A closed loop
A B C D This law is also known as Kirchhoff’s Voltage Law (KVL).
B
R1 R2 R3 R4
R2 E2
The loop rule is based on conservation of energy.
D C E3 Sign conventions for the application of
E1 E2
Kirchhoff’s laws
R3 F
F E I
E G For the application of Kirchhoff’s laws, the following sign
(a)
R5
(b)
conventions are to be considered.
Fig. 3.20 (i) The change in potential in traversing a resistance in
the direction of current is –IR while in the opposite
However, it is always possible to analyse such circuits by direction is +IR.
applying two rules, derived by Kirchhoff in 1845 and 1846. A I R B A I R B
The two terms related to Kirchhoff’s laws are given below
–IR +IR
Junction Fig. 3.22
A junction in a circuit is a point, where three or more (ii) The change in potential in traversing an emf source
conductors meet. Junctions are also called nodes or from negative to positive terminal is +E while in the
branch points. For example, in Fig. 3.20 (a) points D and C opposite direction is −E, irrespective of the direction
are junctions. Similarly, in Fig. 3.20 (b) points B and F are of current in the circuit.
junctions. A E B A E B
Loop
–E +E
A loop is any closed conducting path. For example, in
Fig. 3.23
Fig. 3.20 (a) ABCDA, DCEFD and ABEFA are loops.
Similarly, in Fig. 3.20 (b), CBFEC, BDGFB are loops. To calculate potential difference between two
points by using Kirchhoff’s law
Kirchhoff’s junction rule While using Kirchhoff’s laws to calculate the potential difference, the
The algebraic sum of the currents meeting at a point or at following points should be considered
a junction in an electric circuit is always zero. (i) Start from a point on the loop and go along the loop, either
anti-clockwise or clockwise, to reach the same point again, but
i.e. ∑ i=0 balance currents at junction as per KCL.
(ii) If moving along the direction of the current, there will be
junction
potential drop across a resistance and if moving in the opposite
This law can also be written as, “the sum of all the currents direction, there will be potential gain.
directed towards a point (node) in a circuit is equal to the sum (iii) The net sum of all these potential differences should be zero,
of all the currents directed away from that point (node)”. using the KVL rule.
Now, let us consider a circuit as shown in Fig. 3.24.
This law is also known as Kirchhoff’s Current Law (KCL). E1 E2
A + – B – + C
I2 i1 i2
I1
R1 R3 R4
i1 + i2 E3
I3 i1
I4 F R2 E + − D

Fig. 3.21 Fig. 3.24


In closed loop ABEFA,
Thus, in Fig. 3.21 according to KCL, I 1 + I 2 = I 3 + I 4 . − E1 − (i1 + i 2) R 3 − i1R 2 − i1R1 = 0
The junction rule is based on conservation of electric In closed loop BCDEB,
charge. E 2 + i 2R 4 + E 3 − (i1 + i 2) R 3 = 0
for More Books Click HERE -> NEETpassionate.com
Current Electricity 201

Example 3.43 Find currents in different branches of the (a) Find the potential difference between the points A and B.
electric circuit shown in figure. (b) Find current through 20 V cell, if points A and B are
4Ω B 2Ω connected.
A C
Sol. (i) 9V 8V 7V
3A
2V 4V 6V A B
1Ω 2Ω
When we move from A to B, using KVL,
F D
2Ω E 4Ω VA − 9 − 3 × 1 − 8 − 3 × 2 − 7 = VB
Sol. Applying Kirchhoff’s first law ( junction law) at junction B, ⇒ VA − VB = 33 V
i1 = i2 + i3 …(i) (ii) (a) 2Ω i A 1Ω
P Q
Applying Kirchhoff ’s second law in loop 1 (ABEFA),
A
− 4 i1 + 4 − 2 i1 + 2 = 0 …(ii) 20 V
B
10 V
Applying Kirchhoff ’s second law in loop 2 (BCDEB ), 5V
− 2 i3 − 6 − 4 i3 − 4 = 0 …(iii) 2Ω 2Ω
1Ω
Solving Eqs. (i), (ii) and (iii), we get i
S R
i1 = 1 A 1Ω C 2Ω
4W 2W
A B
C No current flows in the branch CB as AB is not connected.
i1 i3
i2
Let current in the circuit APSCRQA be i.
+ 1 - 2 + Using KVL,
2V 4V 6V
- + - VA − i × 2 − 20 − i × 2 − i × 1 − i × 2 − i × 2+ 10 − i × 1 = VA
i1 i3 ⇒10i = − 10 ⇒ i = − 1 A
F D i.e., direction of i is opposite.
2W E 4W
5 8 Now, A to B path will be APSCB,
i3 = −
A ⇒ i2 = A VA − i × 2 − 20 − i × 2 − i × 1 + 5 = VB
3 3
Here, negative sign of i3 implies that current i3 is in opposite VA − VB = 15 + 5i = 15 + 5 × (−1) = 10 V
direction of what we have assumed. (b) Now, if the points A and B are connected.
P 2Ω A 1Ω Q
Example 3.44 In above example, find the potential difference
between points F and C. 20 V 10 V
5V
Sol. Let us reach from F to C via A and B,
VF + 2 − 4 i1 − 2 i3 = VC 2Ω 1Ω 2Ω
∴ VF − VC = 4 i1 + 2 i3 − 2
Substituting, i1 = 1 A S 1Ω C 2Ω R
and i3 = − (5 /3) A, we get
Resistance 2 Ω, 2 Ω and 1 Ω are in series along APSC
VF − VC = − (4 /3) V and resistance 1 Ω, 2 Ω and 2 Ω are also in series
Here, negative sign implies thatVF < VC . along AQRC.
A A A
Example 3.45 (i) Find the potential difference between the points A
A and B. i1
9V 8V 7V 20 V 5V 10 V
3A
A B 55
V
1W 2W 1W 7
5W 5W
(ii) 2Ω 1Ω
B B B B
20 V A 10 V
B
So, the equivalent voltage will be
5V 20 5 10
+ +
2Ω 2Ω
E= 5 1 5 = 4 + 5 + 2 = 11 × 5 = 55 V
1Ω 1 1 1 1+ 5 +1
+ + 7 7
5 1 5 5
1Ω 2Ω
for More Books Click HERE -> NEETpassionate.com
202 OBJECTIVE Physics Vol. 2

∴ Potential difference between points A and B, When some potential differenceV is applied across a
55 resistance R, charge q flows through the circuit in time t,
VA − VB = V
7 then the heat absorbed or produced is given by
For cell of emf 20 V, V 2t
W = qV = Vit = i 2Rt = joule
VA − VB = 20 − 5 i1 R
55 55 Vit i 2Rt V 2t
= 20 − 5 i1 ⇒ 5 i1 = 20 − −~ 12 A or W= = = cal
7 7 J J JR
∴ Current through cell of 20 V, i1 = 2.4 A where, J is the joule’s mechanical equivalent of heat
(4.21 J/cal).
Electrical energy and power These relations are also called Joule’s law of heating.
Electrical energy and power in electrical circuits or
components are described below. Example 3.46 In the given network of resistors, find the heat
developed across each resistance in 2s.
Electrical energy 6Ω
It is defined as the total work doneW by the source of emf 3Ω
V in maintaining the electric current I in the circuit for a
specified time t. 3Ω
5Ω
According to Ohm’s law, we haveV = IR
Total charge that crosses the resistor is given by q = It 20 V
Energy gained is given by E = W = Vq =VIt Sol. The 6 Ω and 3 Ω resistances are in parallel. So, their
= (IR ) It = I 2R t [QV = IR ] combined resistance is
1 1 1 1
2 2 = + = or R = 2 Ω
V  V t  V R 6 3 2
=   Rt = Q I = 
R R  R The equivalent simple circuit can be drawn as shown.
The SI unit of electrical energy is joule (J), where 3Ω 2Ω

1 joule = 1volt × 1ampere × 1second = 1watt × 1second. 5Ω

Electrical Power 20 V
It is defined as the rate of electrical energy supplied
per unit time to maintain the flow of electric current Current in the circuit,
through a conductor. Net emf 20
i= = =2A
W V2 Total resistance 3 + 2 + 5
Mathematically, P = = VI = I 2R = V = iR = ( 2) ( 2) = 4 V
t R
The SI unit of power is watt (W), where i.e. potential difference across 6 Ω and 3 Ω resistances same
as 4 V. Now,
1 watt = 1 volt × 1 ampere =1ampere-volt. 3W 2W
It can be defined as, the power of an electric circuit is
called one watt, if one ampere current flows in it against V
a potential difference of one volt. The bigger units of i 5W
electrical power are kilowatt (kW) and megawatt (MW)
where, 1kW = 1000 W and 1 MW = 10 6 W. 20 V
Commercial unit of electrical power is horse power (HP),
where, 1HP = 74 6 W. H 3 Ω = i 2Rt = (2)2 (3) (2) = 24 J
V2 (4)2 16
H6Ω = t= (2) =
Heating effects of current R 6 3
J

An electric current through a resistor increases its thermal V2 (4)2 (2) 32


energy. Also, there are other situations in which an H3Ω = t= = J
R 3 3
electric current can produce or absorb thermal energy.
and H 5 Ω = i 2Rt = (2)2 (5) (2) = 40 J
This effect is called heating effect of electric current.
for More Books Click HERE -> NEETpassionate.com
Current Electricity 203

Electricity consumption Power consumption in a


To measure the electrical energy consumed commercially, combination of bulbs
joule is not sufficient. So, a special unit kilowatthour is
used in place of joule. It is also called 1 unit of electrical Series combination of bulbs
energy. 1 kilowatt hour or 1 unit of electrical energy is
the amount of energy dissipated in 1 hour in a circuit, (i) Total power consumed is given by
when the electric power in the circuit is 1 kilowatt. 1 1 1
= + + ....
1 kilowatt hour (kWh) = 3.6 × 10 6 joule (J) Peq P1 P2
P1 P2
Note Resistance of electrical appliance On electrical appliances
(bulbs, geysers, heaters, etc.) wattage, voltage printed are called
rated values. The resistance of any electrical appliance can be
calculated by rated power and rated voltage by using R = VR2 / PR.

Example 3.47 In the following figure, find V


Fig. 3.25 Series combination of two bulbs
10 V 4V
1
(ii) Pconsumed (brightness) ∝ V ∝ R ∝ , i.e. in series
i Prated
combination bulb of lesser wattage will give more
3Ω bright light and potential difference appearing across
it will be more.
(i) the power supplied by 10 V battery,
(ii) the power consumed by 4 V battery and Parallel combination of bulbs
(iii) the power dissipated in 3 Ω resistance.
(i) Total power consumed is given by
Sol. Net emf of the circuit = (10 – 4) = 6 V
Ptotal = P1 + P2 + ....
Total resistance of the circuit = 3 Ω
Net emf 6 P1, V
∴ Current in the circuit, i = = = 2A
Total resistance 3
(i) Power supplied by 10 V battery = Ei = (10) (2) P2, V
= 20 W
(ii) Power consumed by 4 V battery = Ei = (4) (2) = 8 W
V
(iii) Power dissipated in 3 Ω resistance = i 2R = (2)2 (3) = 12 W
Fig. 3.26 Parallel combination of two bulbs
Example 3.48 In the following figure, each of the three 1
resistances, has rating of 24 W and resistance of 6 Ω. Find (ii) Pconsumed (brightness) ∝ Prated ∝ I ∝, i.e. in parallel
the maximum power rating of the circuit. R
6Ω combination, bulb of greater wattage will give more
B 6Ω C
bright light and more current will pass through it.
A
6Ω
Applications of heating effects of current
Sol. To find maximum current i, (i) Filament of electric bulb is made up of tungsten
P 24 which has high resistivity and high melting point.
we use, P = i 2R ⇒ i 2 =
= = 4 ⇒ i = 2A
R 6 (ii) Electric devices having heating elements like heater,
1 1 1 1 geyser or press are made up of nichrome which has
Resistance between A and B is = + = ⇒ R = 3Ω
R1 6 6 3 high resistivity and high melting point.
and resistance between A and C , R eq = 3 + 6 = 9 Ω (iii) Fuse wire is made up of tin-lead alloy, since it
If we make 2 A current flow through the given circuit, 1 A should have low melting point and high resistivity.
will flow through each of 6 Ω in parallel and 2 A through It is used in series as a safety device in an electric
6 Ω in series. This is the maximum current the circuit can circuit and is designed, so as to melt and thereby open
hold. the circuit, if the current exceeds a predetermined
So, power of circuit = i 2R = 2 × 2 × 9 = 36 W value due to some fault.
for More Books Click HERE -> NEETpassionate.com
204 OBJECTIVE Physics Vol. 2

Example 3.49 Two bulbs having rating of 60W-220 V and Example 3.52 Two coils of power 60 W and 100 W and both
100 W-220 V are joined (i) in series and (ii) in parallel. operating at 220 V takes time 2 min and 1.5 min separately
Which of the two will glow brighter in each case? to boil certain amount of water. If they are joined (i) in series
Sol. Suppose both are used at 220 V supply in both the cases. In and (ii) in parallel, then find the ratio of time taken by them
case of parallel combination, same voltage will be appear on to boil the same water in the two cases.
each bulb, hence heat produced will be proportional to1/R . Sol. When they are joined in series, then total power,
As, bulb of greater wattage will glow more. Hence, 100 W PP 60 × 100
bulb will glow brighter. In case of series combination, same P= 1 2 = = 37.5 W
P1 + P2 160
current flows through each bulb, hence heat produced will be
proportional to R (P = i 2R ). As we know, higher the wattage, When they are joined in parallel, then total power
= 60 + 100 = 160 W
lower the resistance, then 60 W bulb will have higher
Time taken to boil the water will be inversely proportional
resistance. That means more heat will be produced in 60 W
to power, so ratio of time taken in the two cases will be
bulb in this case, so this bulb will glow brighter. 160 : 37.5.
Example 3.50 Two bulbs having rating of 40 W-220 V and Example 3.53 Figure shows three identical bulbs A, B and C,
100 W-220 V are joined in series and alternately, (i) 300 V which are connected to a battery of supply voltage V. When
and (ii) 440 V is applied. Find out which bulb will fuse in the switch S is closed, then discuss the change in
each case.
(i) the illumination of the three bulbs.
Sol. We first have to find out the maximum current each bulb (ii) the power dissipated in the circuit.
can bear. This can be calculated from rating of the bulb,
A B C
40
P = VI ⇒ 40 = 220 I ⇒ I = = 0.18 A
220
This is the maximum current 40 W bulb can bear.
100
Similarly, 100 W bulb can bear = 0.45 A. S
220 V

V2
Now, find out the resistance of each bulb, R =
P Sol. When the switch S is open,
220 VA = VB = VC = V /3
⇒ Resistance of 40 W bulb = × 220 = 1210 Ω
40 and PA = PB = PC
220 × 220
Resistance of 100 W bulb = = 484 Ω ( /3)2 V 2
V
100 = = = P (say)
R 9R
These are joined in series, so total resistance = 1694 Ω
(i) When the switch S is closed, then the bulb C is short
(i) Current in each bulb when joined with 300 V circuited and hence there will be no current through C.
300 So, PC = 0
i.e. I= = 0.177 A
1694 V
VA = VB =
This current will flow in each, so no bulb will fuse as it 2
is less than their maximum permissible current.
(V /2)2 V 2 9
(ii) When they are joined with 440 V in series then current So, PA = PB = = = P
440 R 4R 4
will be = = 0.26 A.
1694 Therefore, the intensity of illumination of each of the
This current is less than maximum permissible current bulb A and B become 9/4 times of the initial value but
of 100 W bulb but more than that of 40W bulb. Hence, the intensity of the bulb C becomes zero.
40 W bulb will be fused and 100 W bulb will remain (ii) The power dissipated in the circuit before closing the
safe. switch is
Pi = PA + PB + PC = 3P
Example 3.51 In above example, if we join the bulbs in The power dissipated after closing the switch is
parallel and 300 V is applied, which of the two bulbs will
Pf = PA + PB + PC
fuse?
9 9
Sol. When they are joined in parallel and 300 V is applied on = P+ P+0
4 4
them, then both will get 300 V. Since, their rating is 220 V,
9
naturally, current flowing through them will be more than = P
maximum possible value. Hence, both will fuse out. 2
for More Books Click HERE -> NEETpassionate.com
Current Electricity 205

CHECK POINT 3.3


1. The figure shows a network of currents. The current i will 9. The electric bulb have tungsten filaments of same length. If
be one of them gives 60 W and other 100 W, then
3A
(a) 100 W bulb has thicker filament
15 A
(b) 60 W bulb has thicker filament
(c) Both filaments are of same thickness
8A
(d) it is not possible to get different wattage unless the
i lengths are different
5A
10. How many calories of heat will be produced approximately
(a) 3 A (b) 13 A in 210 W electric bulb in 5 min?
(c) 23 A (d) – 3 A (a) 800000 cal (b) 63000 cal
2. In the circuit element given here, if the potential difference (c) 1050 cal (d) 15000 cal
at point B , VB = 0, then the potential difference between A 11. If in the circuit, power dissipation is 150 W, then R is
and D are
R
1A 1.5 Ω 2.5 Ω 2V
A B C D
(a) V A = − 1.5 V, VD = + 2 V (b) V A = − 1.5 V, VD = + 0.5 V
2Ω
(c) V A = + 1.5 V, VD = + 0.5 V (d) V A = + 1.5 V, VD = − 0.5 V
3. Three resistances are connected to form a T-shape as shown
in the figure. Then, the current i in the 2 Ω resistor is 15 V
2W 4W
(a) 2 Ω (b) 6 Ω (c) 5 Ω (d) 4 Ω
20 V 5V
12. A wire when connected to 220 V mains supply has power
i
dissipation P1 . Now, the wire is cut into two equal pieces,
2W which are connected in parallel to the same supply. Power
dissipation in this case is P2. Then, P2 : P1 is
(a) 1 (b) 4 (c) 2 (d) 3
13. Two electric bulbs, one of 200 V-40 W and other 200 V-100
0V
W are connected in series to a 200 V line, then the potential
(a) 0.93 A (b) 4.5 A drop across
(c) 2.5 A (d) 1.57 A (a) the two bulbs is zero (b) both the bulbs is 200 V
4. How much work is required to carry a 6 µC charge from the (c) 40 W bulb is more (d) 100 W bulb is more
negative terminal to the positive terminal of a 9 V battery?
14. Three identical bulbs are connected in series and these
(a) 54 × 10−3 J (b) 54 × 10−6 J together dissipate a power P. Now, if the bulbs are connected
(c) 54 × 10−9 J (d) 54 × 10−12 J in parallel, then the power dissipated will be
P P
5. Two resistors R and 2R are connected in series in an electric (a) (b) 3P (c) 9P (d)
3 9
circuit. The thermal energy developed in R and 2R are in the
ratio 15. A and B are two bulbs connected in parallel. If A is glowing
(a) 1 : 2 (b) 2 : 1 (c) 1 : 4 (d) 4 : 1 brighter than B, then the relation between R A and RB is
(a) R A = RB (b) RB > R A
6. The resistor of resistance R is connected to 25 V supply and
−1 (c) R A > RB (d) None of these
heat produced in it is 25 Js . The value of R is
(a) 225 Ω (b) 1 Ω (c) 25 Ω (d) 50 Ω 16. Some electric bulbs are connected in series across a
220 V supply in a room. If one bulb is fused, then remaining
7. Just as electricity is supplied at 220 V for domestic use in bulbs are connected again in series (after removing the
India, it is supplied at 110 V in USA. If the resistance of fused bulb) across the same supply. The illumination in the
60 W bulb for use in India is R, then that of 60 W bulb for room will
use in USA will be
(a) increase (b) decrease
(a) R /4 (b) R /2
(c) remain the same (d) not continuous
(c) R (d) 2R
8. If R1 and R2 are respectively, the filament resistances of 17. Electric bulbs of 50 W-100 V glowing at full power are to be
used in parallel with battery 120 V, 10 Ω. Maximum
200 W bulb and 100 W bulb designed to operate on the
number of bulbs that can be connected, so that they glow in
same voltage, then
full power is
(a) R1 is two times R2 (b) R2 is two times R1
(a) 2 (b) 8
(c) R2 is four times R1 (d) R1 is four times R2
(c) 4 (d) 6
for More Books Click HERE -> NEETpassionate.com
206 OBJECTIVE Physics Vol. 2

MEASURING INSTRUMENTS
FOR CURRENT AND VOLTAGE
There are various instruments like galvanometer, Voltmeter
ammeter and voltmeter which can be used to detect It is an instrument used to measure potential difference and
current and voltage, depending on the range. is always connected in parallel with the circuit element
across which potential difference is to be measured.
Galvanometer Greater the resistance of voltmeter, more accurate will be
It is an instrument used to detect small current passing its reading, as only small amount of current pass through it,
through it by showing deflection. It can be converted into by not changing the circuit current. A voltmeter is said to
voltmeter (for measuring voltage) and ammeter (for be ideal, if its resistance is infinite.
measuring current). Conversion of galvanometer into voltmeter
A galvanometer can be converted into voltmeter by connecting
Ammeter a large resistance R in series with the galvanometer.
It is an instrument used to measure current and is always
R
connected in series with the circuit element through G
which current is to be measured. Smaller the resistance of
an ammeter, more accurate will be its reading, as it will ig
not change the circuit current. An ammeter is said to be
ideal, if its resistance r is zero. V

Conversion of galvanometer into ammeter Fig. 3.28 Voltmeter


A galvanometer can be converted into an ammeter by According to Ohm’s law,V = i g (G + R )
connecting a low resistance (called shunt S) in
V
parallel to the galvanometer of resistance G. or required resistance, R = −G
Hence, only a small amount of current pass through ig
galvanometer and remaining will pass through the shunt.
Example 3.55 How can we make a galvanometer with
G and S are parallel and hence have equal potential G = 20 Ω and i g = 1 mA into a voltmeter with a maximum
difference, i.e. i g G = (i − i g ) S. range of 10 V?
S V
Sol. Using, R = –G
ig
i–ig
10
i We have, R = − 20 = 9980 Ω
ig
G 10−3
Thus, a resistance of 9980 Ω is to be connected in series with the
Fig. 3.27 Ammeter galvanometer to convert it into the voltmeter of desired range.
ig
∴ Required shunt resistance, S = G Wheatstone bridge
(i − i g )
It is an arrangement of four resistances used to measure one
Example 3.54 What shunt resistance is required to make the of them, in terms of the other three as shown in Fig. 3.29.
1 mA, 20 Ω galvanometer into an ammeter with a range of B
0 to 50 mA? i1
Ig=0

Sol. Given, ig = 1mA = 10 –3 A , G = 20 Ω


P Q i1
A C
i = 50 × 10 –3 A G
i2 i2
 ig  R S
Substituting in S =  G i
 i – ig 
D
(10−3 ) (20)
E
=
(50 × 10−3 ) − (10−3 )
Fig. 3.29 A Wheatstone bridge
= 0.408 Ω
for More Books Click HERE -> NEETpassionate.com
Current Electricity 207

The bridge is said to be balanced, when deflection in Sol. The given network is equivalent to the circuit shown in
galvanometer is zero (I g = 0 ), i.e. no current flows through figure below
the galvanometer (branch BD). In the balanced condition, B

P R 1Ω 2Ω
=
Q S A 5Ω C

On mutually changing the position of cell and galvanometer, 2Ω 4Ω


this condition will not change. I D I
Note Different forms of Wheatstone bridge are shown below
P 4V

Q Now, the circuit represents a balanced Wheatstone bridge.


G 1 2
R ∴ =
2 4
S
The resistance of 5 Ω in arm BD is ineffective. The
equivalent circuit reduces to the circuit shown in figure.
3W
P R
Q 6W
P G
G I I
R Q S

S 4V
Fig. 3.30
6×3
Equivalent resistance, R eq = =2Ω
Example 3.56 Find out the magnitude of resistance X in the 6+3
circuit shown in figure, when no current flows through the ∴ Current drawn from the battery,
5 Ω resistor. V 4
I= = = 2A
R eq 2
6V
X 18 W Meter bridge
5W A meter bridge is slide wire bridge or Carey Foster
2W 6W bridge. It is an instrument that works on the principle of
Wheatstone bridge.
Sol. As no current flows through the middle 5 Ω resistor, the It consists of a straight and uniform wire along a meter
circuit represents a balanced Wheatstone bridge. scale (AC) and by varying the taping point B as shown in
X 2 Fig. 3.31, the bridge is balanced.
∴ =
18 6 E K
2
or X = × 18 = 6 Ω
6 S
R D
Example 3.57 Calculate the current drawn from the battery by
the network of resistors shown in figure.
0 Meter scale 100
2Ω
B
A 1Ω 5Ω D 4Ω P Q
B A C
C
(100–l)
2Ω (l) G

Fig. 3.31 Measuring unknown resistance using meter bridge


4V
for More Books Click HERE -> NEETpassionate.com
208 OBJECTIVE Physics Vol. 2

∴ At balancing situation of bridge, Example 3.59 The given figure shows the experimental set up
P R of a meter bridge. The null point is found to be 60 cm away
= from the end A with X andY in position as shown. When a
Q S resistance of 15 Ω is connected in series withY, then the
l R null point is found to shift by 10 cm towards the end A of the
⇒ = wire. Find the position of null point, if a resistance of
100 − l S 30 Ω were connected in parallel withY.
100 − l
⇒ S= ×R X Y
l
B
Applications of meter bridge
(i) It is used to measure an unknown resistance by G
R (100 − l )
using, S = A C
l
(ii) To compare the two unknown resistances by using,
R l
=
S 100 − l Sol. In first case,
X 60 X 3
Example 3.58 In the following circuit, a meter bridge is = or = …(i)
shown in its balanced state. The meter bridge wire has a Y 40 Y 2
resistance of 1 Ω-cm. Calculate the value of the unknown In second case,
resistance X and the current drawn from the battery of X 50
negligible internal resistance. = =1 …(ii)
Y + 15 50
6W
Dividing Eq. (i) by Eq. (ii), we get
X
G X Y + 15 3
× = ×1
40 cm 60 cm Y X 2
A B 15 3
J or 1+ =
Y 2
or Y = 30 Ω
6V
3 3
X = Y = × 30 = 45 Ω
Sol. In balanced condition, no current flows through the 2 2
galvanometer. When a resistance of 30 Ω is connected in parallel withY ,
Here, P = resistance of wire AJ = 40 Ω then the resistance in the right gap becomes
Q = resistance of wire BJ = 60 Ω 30Y 30 × 30
Y′ = = = 15 Ω
R = X, S = 6 Ω 30 + Y 30 + 30
In the balanced condition, Suppose the null point occurs at l cm from end A.
P R Then,
=
Q S X l
=
40 X 15 100 − l
or =
60 6 45 l
or X =4Ω or =
15 100 − l
Total resistance of wire AB = 100 Ω
or 300 − 3l = l
Total resistance of resistances X and 6 Ω connected in series
or 4l = 300
= 4 + 6 = 10 Ω
or l = 75 cm
This series combination is in parallel with wire AB.
10 × 100 100
∴ Equivalent resistance = =
10 + 100 11
Ω Potentiometer
The potentiometer is an instrument that can be used to
emf of the battery = 6 V
measure the emf or the internal resistance of an unknown
∴ Current drawn from the battery, source. It is a device which does not draw any current
emf 6 through the circuit to measure the potential difference.
I= = = 0.66 A
resistance 100 / 11
for More Books Click HERE -> NEETpassionate.com
Current Electricity 209

Hence, it is equivalent to an ideal voltmeter. It also has a rheostat R h and 1-way key K are connected between the
number of other useful applications. terminals A and B of the potentiometer.
K K
E
– + + – – + +
E

A A
Rh Rh
400 400
B B
300 J 300

200 200

100
100
A 0 10 20 30 40 50 60 70 80 90
A 0 10 20 30 40 50 60 70 80 90
+ – 1
Meter rod R E1 Meter rod
G R
+ – 3
+ G − E2 2
Fig. 3.32 Potentiometer Fig. 3.33 Circuit for comparing emf’s of two cells

Principle of potentiometer Now, to compare emfs of two cells having a constant


The potentiometer works on the principle that, when a current passing through the wire between terminals A and
constant amount of current flows through a wire of B, the current is kept constant by using rheostat. If the
uniform cross-section and composition, then the potential plug is put in the gap between terminals 1 and 3 of 2-way
drop across the wire is directly proportional to its length, key, then the emf E 1 of the cell is given by
i.e. E 1 = (xl1 ) I …(i)
V ∝ l ⇒ V = kl …(i) where, x = resistance per unit length of potentiometer wire
where, k is the constant of proportionality. (Q l1 = balancing length)
Also, by Ohm’s law, Now, when the key is put in the gap between terminals 2
and 3 after removing it from the gap between 1 and 3,
l
V = IR = ρ I …(ii) then the emf E 2 is given by
A
E 2 = (xl 2 ) I (Q l 2 = balancing length) …(ii)
ρI
If the current I is constant, then would be constant. From Eqs. (i) and (ii), we have
A
E 1 l1
So, comparing Eqs. (i) and (ii), we will have =
I E 2 l2
⇒ k =ρ
A (ii) To measure internal resistance of a cell
k is also known as potential gradient, which is the Now, to find the internal resistance r of a cell of emf E, let
potential drop per unit length of the potentiometer wire. E ′ be emf of the battery. A constant current I is maintained
V through the potentiometer wire with the help of rheostat.
i.e. k=
l K1
E′
The SI unit of potential gradient is Vm−1 and CGS unit – + + –
A
is Vcm −1. Rh
400
Applications of potentiometer B
J 300
Some important applications of potentiometer are given
below. 200

(i) To compare the emfs of two cells


0 10 20 30 40 50 60 70 80 90
Consider two cells of emfs E 1 and E 2 is to be compared. A
E
The positive terminals of both the cells are connected to G R
terminal A of potentiometer and the negative terminals of
both cells are connected to terminals 1 and 2 of a 2-way S
K2
key, while its common terminal is connected to a jockey J Fig. 3.34 Circuit for determining internal resistance of cell
through a galvanometer G. A battery of emf E, ammeter A,
for More Books Click HERE -> NEETpassionate.com
210 OBJECTIVE Physics Vol. 2

Now, the plug in key K 2 is kept out and the jockey J is Example 3.62 In a potentiometer arrangement, a cell of emf
moved on the potentiometer wire to balance the emf E of 2.25V gives a balance point at 30.0 cm length of the wire.
the cell, whose internal resistance r is to be determined. If the cell is replaced by another cell and the balance point
shifts to 60.0 cm, then what is the emf of the second cell?
Suppose l1 be the balancing length of the potentiometer
wire between point A and jockey J. If x is resistance per Sol. Given, E1 = 2.25 V, l1 = 30.0 cm,
unit length of wire, then emf of cell is given by l 2 = 60.0 cm and E 2 = ?
E = xl1I …(i) As we know that in case of potentiometer, the potential
gradient remains constant.
Introduce some resistance, says S from the resistance box S
So, E ∝l
and now put in the plug key K 2 . The potential difference E1 l1
V between the two terminals of the cell is given by ∴ =
E2 l2
V = xl 2I …(ii) 2.25 30.0
[Q l 2 = balancing length] ⇒ =
E2 60.0
On dividing Eq. (i) by Eq. (ii), we have
2.25 × 60
E l1 ∴ E2 = = 4.5 V
= 30
V l2
Example 3.63 AB is 1 m long uniform wire of 10 Ω
E  resistance. Other data are as shown in figure. Calculate (i)
The internal resistance of the cell is given by r =  − 1 S
V  potential gradient along AB and (ii) length AO, when
galvanometer shows no deflection.
Now, substituting values of E /V in above equation, we get
l  2V 15W
⇒ r =  1 − 1 S O
 l2  A B
1.2W 1.5V
where, S is the resistance of the resistance box. G
Note 0.3W
(i) The sensitiveness of potentiometer means the smallest potential
difference that can be measured with its help. Sol. (i) Total resistance of the primary circuit
(ii) A potentiometer can also be used to compare unknown resistances
and to calibrate a voltmeter or an ammeter. = 15 + 10 = 25 Ω and emf = 2 V
(iii) A balance point is obtained on the potentiometer wire, if the fall of ∴ Current in the wire AB,
potential along the potentiometer wire due to driving cell is greater 2
than the emf of the cell to be balanced. I= = 0.08 A
25
Potential difference across the wire AB
Example 3.60 A potentiometer wire is 10 m long and has a = Current × Resistance of wire AB
resistance of 18 Ω. It is connected to a battery of emf 5V
= 0.08 × 10 = 0.8 V
and internal resistance 2 Ω. Calculate the potential gradient
along the wire. Potential gradient
Potential difference 0.8
Sol. Given, l = 10 m , R = 18 Ω, E = 5 V and r = 2 Ω = = = 0.008 V cm−1
Length 100
Current through the potentiometer wire,
E 5 5 1 (ii) Resistance of secondary circuit = 1.2 + 0.3 = 1.5 Ω
I= = = = A
R + r 18 + 2 20 4 emf = 1.5 V
IR 1 18 1.5
∴ Potential gradient = = × = 0.45 Vm−1 Current in the secondary circuit = = 1.0 A
l 4 10 1.5
The same current is flowing in 0.3 Ω resistor.
Example 3.61 A cell can be balanced against 110 cm and
100 cm of potentiometer wire respectively, when in open Potential difference between points A and O
circuit and when short circuited through a resistance of = Potential difference across 0.3 Ω resistor in the zero
10 Ω. Find the internal resistance of the cell. deflection condition
Sol. Given, l1 = 110 cm, l 2 = 100 cm, R = 10 Ω and r = ? = Current × Resistance = 1.0 × 0.3 = 0.3 V
l − l  Potential difference
∴ r =  1 2R Length AO =
 l2  Potential gradient

110 − 100 0.3 V


⇒ r= × 10 = 1 Ω = = 37.5 cm
100 0.008 V cm−1
for More Books Click HERE -> NEETpassionate.com
Current Electricity 211

CHECK POINT 3.4


1. In the circuit shown, A and V are ideal ammeter and (a) 3 V (b) 2 V
voltmeter, respectively. Reading of the voltmeter will be (c) 5 V (d) 4 V
7. A galvanometer of 25 Ω and having full scale deflection for
2V
a current of 10 mA is changed into voltmeter of range 100 V
by connecting a resistance R in series with the
A V galvanometer. The resistance R (in ohm) is
(a) 10000 (b) 975
1Ω 1Ω (c) 10025 (d) 9975
8. An ammeter and a voltmeter are joined in series to a cell.
(a) 2 V (b) 1 V (c) 0.5 V (d) zero Their readings are A and V, respectively. If a resistance is
now joined in parallel with the voltmeter, then
2. The net resistance of a voltmeter should be large to ensure
(a) both A and V will decrease
that
(b) both A and V will increase
(a) it does not get overheated (c) A will decrease, V will increase
(b) it does not draw emmissive current (d) A will increase, V will decrease
(c) it can measure large potential difference 9. In the circuit shown in the figure, the voltmeter reading is
(d) it does not appreciably change the potential difference 6V
to be measured
3. Two galvanometers A and B require 3 mA and 6 mA
respectively, to produce the same deflection of 10 divisions.
Then, 60 Ω
V
(a) A is more sensitive than B
(b) B is more sensitive than A
(c) both A and B are equally sensitive 40 Ω
(d) sensitiveness of B is twice that of A
4. An ammeter A, a voltmeter V and (a) 2.4 V (b) 3.4 V
+ V (c) 4.0 V (d) 6.0 V
a resistance R are connected as
shown in the figure. If the 10. To send 10% of the main current through a moving coil
voltmeter reading is 1.6 V and + galvanometer of resistance 99 Ω, the shunt required is
the ammeter reading is 0.4 A, A
(a) 9.9 Ω (b) 10 Ω (c) 11 Ω (d) 9 Ω
then R is R
(a) equal to 4 Ω 11. The potential difference across the 100 Ω resistance in the
(b) greater than 4 Ω circuit is measured by a voltmeter of 900 Ω resistance.
(c) less than 4 Ω
The percentage error made in reading the potential
difference is
(d) between 3 Ω and 4 Ω
900 Ω
5. In the following circuit, the emf of the cell is 2 V and the V
internal resistance is negligible. The resistance of the
voltmeter is 80 Ω. The reading of the voltmeter will be
10 Ω 100 Ω
2V
− +
80 Ω
10
V (a) (b) 0.1 (c) 1.0 (d) 10.0
9
20 Ω 80 Ω 12. A microammeter has a resistance of 100 Ω and a full scale
range of 50 µA. It can be used as a voltmeter or as a higher
range ammeter provided a resistance is added to it. Pick the
(a) 0.80 V (b) 1.60 V (c) 1.33 V (d) 2.00 V
correct range and resistance combinations.
6. What is the reading of voltmeter in the figure? (a) 50 V range with 10 k Ω resistance in series
10 V
(b) 10 V range with 200 k Ω resistance in series
(c) 10 mA range with 1Ω resistance in parallel
(d) None of the above
1000 Ω 13. The percentage error in measuring resistance with a meter
V bridge can be minimised by adjusting the balancing point
close to
500 Ω 500 Ω (a) 20 cm (b) 50 cm (c) 80 cm (d) 100 cm
for More Books Click HERE -> NEETpassionate.com
212 OBJECTIVE Physics Vol. 2

14. When an additional resistance of 1980 Ω is connected in 17. A potentiometer is used for the comparison of emf of two
series with a voltmeter, then the scale division reads cells E1 and E2. For cell E1 , the deflection point is obtained
100 times larger value. Resistance of the voltmeter is at 20 cm and for E2, the deflection point is obtained at
(a) 10 Ω (b) 20 Ω 30 cm. The ratio of their emfs will be
(c) 30 Ω (d) 40 Ω (a) 2/3 (b) 3/2
15. In the given circuit, it is observed that the current I is (c) 1 (d) 2
independent of the value of the resistance R6. Then, the 18. In a potentiometer experiment, the galvanometer shows no
resistance values must satisfy deflection, when a cell is connected across 60 cm of the
potentiometer wire. If the cell is shunted by a resistance of
R5
6 Ω, the balance is obtained across 50 cm of the wire. The
i
internal resistance of the cell is
R1 R3 (a) 0.5 Ω (b) 0.6 Ω
R6
(c) 1.2 Ω (d) 1.5 Ω

R2 R4 19. A resistance of 4 Ω and a wire of length 5 m and resistance


5Ω are joined in series and connected to a cell of emf 10 V
and internal resistance 1 Ω. A parallel combination of two
(a) R1 R2R3 = R3R4 R5 identical cells is balanced across 3 m of the wire. The emf E
1 1 1 1 of each cell is
(b) + = +
R5 R6 R1 + R2 R3 + R4 4W 10 V
(c) R1 R4 = R2R3
(d) R1 R3 = R2R4 = R5R6 5m 1W

16. AB is a wire of uniform resistance. The galvanometer G 3m


X
shows no current when the length AC = 20 cm and E
CB = 80 cm. The resistance R is equal to
G
R 80 Ω
E
(a) 1.5 V (b) 3.0 V
G (c) 0.67 V (d) 1.33 V
A B 20. Potentiometer wire of length 1 m is connected in series
C
with 490 Ω resistance and 2 V battery. If 0.2 mV/ cm is the
potential gradient, then the resistance of the potentiometer
wire is
(a) 320 Ω (b) 8 Ω (c) 20 Ω (d) 40 Ω
(a) 4.9 Ω (b) 7.9 Ω (c) 5.9 Ω (d) 6.9 Ω
for More Books Click HERE -> NEETpassionate.com

Chapter Exercises
(A) Taking it together
Assorted questions of the chapter for advanced level practice

1 The terminal potential difference of a cell is greater 8 A student has 10 resistors of resistance r each. The
than its emf when it is minimum resistance made by him from given
(a) being discharged resistors is
(b) open circuit r r r
(a) 10r (b) (c) (d)
(c) being charged 10 100 5
(d) being either charged or discharged 9 A wire has resistance 12 Ω. It is bent in the form of
2 For measurement of potential difference, a circle. The effective resistance between the two
potentiometer is preferred in comparison to points on any diameter is equal to
voltmeter because (a) 12 Ω (b) 6 Ω (c) 3 Ω (d) 24 Ω
(a) potentiometer is more sensitive than voltmeter 10 A steady current (i) is flowing through a conductor of
(b) the resistance of potentiometer is less than voltmeter
uniform cross-section. Any segment of the conductor
(c) potentiometer is cheaper than voltmeter
has
(d) potentiometer does not take current from the circuit
(a) zero charge
3 What is immaterial for an electric fuse wire? (b) only positive charge
(a) Its specific resistance (b) Its radius (c) only negative charge
(c) Its length (d) Current flowing through it (d) charge proportional to current i
4 Conductivity increases in the order of 11 There are n cells, each of emf E and internal
(a) Al, Ag, Cu (b) Al, Cu, Ag resistance r, connected in series with an external
(c) Cu, Al, Ag (d) Ag, Cu, Al resistance R. One of the cells is wrongly connected, so
5 By mistake, a voltmeter is connected in series and an that it sends current in the opposite direction. The
ammeter in parallel. When the circuit is switched on current flowing in the circuit is
(a) Only the ammeter will be damaged (n – 1)E (n – 1)E
(a) (b)
(b) Only the voltmeter will be damaged (n + 1) r + 2 nr + R
(c) Both ammeter and voltmeter will be damaged (n – 2)E (n – 2)E
(d) Neither the ammeter nor the voltmeter will be (c) (d)
nr + R (n – 2)r + R
damaged
6 If E is the emf of a cell of internal resistance r and 12 The maximum power dissipated in an external
external resistance R, then potential difference resistance R, when connected to a cell of emf E and
across R is given as internal resistance r, will be
(a) V = E /(R + r ) (b) V = E E2 E2 E2 E2
(a) (b) (c) (d)
(c) V = E /(1 + r /R ) (d) V = E /(1 + R /r ) r 2r 3r 4r
7 When n cells are joined in parallel combination as 13 The current-voltage graph for a given metallic wire
shown, the strength of the current i is given by at two different temperatures T1 and T 2 is shown in
the figure. The temperatures T1 and T 2 are related as
E, r

E, r
T1
E, r i T2

R K V

(a) T1 > T2 (b) T1 < T2


nE E E
(a) (b) (c) (d) None of these (c) T1 = T2 (d) T1 > 2T2
R + nr R + (r /n ) r + Rn
for More Books Click HERE -> NEETpassionate.com
214 OBJECTIVE Physics Vol. 2

14 Which of the following characteristics of electrons 21 A wire of resistance R is divided into 10 equal parts.
determines the current in a conductor? These parts are connected in parallel, the equivalent
[NCERT Exemplar] resistance of such connection will be
(a) Drift velocity only (a) 0.01 R (b) 0.1 R
(b) Thermal velocity only (c) 10 R (d) 100 R
(c) Both drift velocity and thermal velocity
(d) Neither drift nor thermal velocity 22 Three resistors each of 2 Ω are connected together
in a triangular shape. The resistance between any
15 An ammeter and a voltmeter of resistance R are two vertices will be
connected in series to an electric cell of negligible (a) 4/3 Ω (b) 3/4 Ω (c) 3 Ω (d) 6 Ω
internal resistance. Their readings are A and V,
respectively. If another resistance R is connected in 23 The effective resistance between the points A and B
parallel with the voltmeter, then potential across in the figure is
D
(a) Both A and V will increase
(b) Both A and V will decrease 3Ω
3Ω
(c) A will decrease and V will increase
(d) A will increase and V will decrease 6Ω
A C
16 The resistance of a wire is 10 Ω. Its length is
increased by 10% by stretching. The new resistance 3Ω 3Ω
will be
(a) 12 Ω (b) 1.2 Ω (c) 13 Ω (d) 11 Ω B

17 Four resistances are connected in a circuit as shown (a) 5 Ω (b) 2 Ω (c) 3 Ω (d) 4 Ω
in the given figure. The electric current flowing 24 Two resistances are joined in parallel whose
through 4 Ω and 6 Ω resistance is respectively 6
resultant is Ω. One of the resistance wire is broken
4Ω 6Ω 8
and the effective resistance becomes 2 Ω. Then, the
4Ω 6Ω resistance in ohm of the wire that got broken was
(a) 3/5 (b) 2 (c) 6/5 (d) 3
25 A wire of resistance 9 Ω is broken in two parts. The
length ratio being 1 : 2. The two pieces are
20 V connected in parallel. The net resistance will be
(a) 2 A and 4 A (b) 1 A and 2 A (a) 2 Ω (b) 3 Ω (c) 4 Ω (d) 6 Ω
(c) 1 A and 1 A (d) 2 A and 2 A 26 In the network shown, the equivalent resistance
18 Current through the 5 Ω resistor is between A and B is
3Ω

8Ω 4Ω
10 V 5Ω 10 V

A 2Ω 6Ω B
6Ω
(a) 2A (b) 4A
(c) zero (d) 1A 4 3 24 17
(a) Ω (b) Ω (c) Ω (d) Ω
19 A cell which has an emf of 1.5 V is connected in 3 4 17 24
series with an external resistance of 10 Ω. If the 27 To send 10% of main current through a moving coil
potential difference across the cell is 1.25 V, then galvanometer of resistance 99 Ω, shunt required is
the internal resistance of the cell (in Ω) is (a) 9 Ω (b) 11 Ω
(a) 2 (b) 0.25 (c) 1.5 (d) 0.3 (c) 10 Ω (d) 9.9 Ω
20 A piece of wire of resistance 4 Ω is bent through 28 A 1250 W heater operates at 115 V. What is the
180° at its mid-point and the two halves are twisted resistance of the heating coil?
together, then the resistance is (a) 1.6 Ω (b) 13.5 Ω
(a) 8 Ω (b) 1 Ω (c) 2 Ω (d) 5 Ω (c) 1250 Ω (d) 10.6 Ω
for More Books Click HERE -> NEETpassionate.com
Current Electricity 215

29 The resistance of a wire at 20°C is 20 Ω and at 300 cm. On connecting the same potentiometer
500°C is 60 Ω. At which temperature its resistance between A and C , the balancing length is 100 cm.
will be 25 Ω? E
The ratio 1 is
(a) 50°C (b) 60°C (c) 70°C (d) 80°C E2
30 A conducting wire of cross-sectional area 1 cm2 has (a) 3 : 1 (b) 1 : 3
3 × 10 23 charge carriers per metre 3 . If wire carries a (c) 2 : 3 (d) 3 : 2
current 24 mA, then drift velocity of carriers is 36 A voltmeter of resistance 998 Ω is connected across
(a) 5 × 10−2 ms−1 (b) 0.5 ms−1 a cell of emf 2 V and internal resistance 2 Ω. The
(c) 5 × 10−3ms−1 (d) 5 × 10−6ms−1 potential difference across the voltmeter is
31 At room temperature, copper has free electron (a) 1.99 V (b) 3.5 V
28 −3 (c) 5 V (d) 6 V
density of 8.4 × 10 m . The electron's drift
velocity in a copper conductor of cross-sectional area 37 A wire 50 cm long and 1 mm2 in cross-section
of 10 −6 m2 and carrying a current of 5.4 A, will be carries a current of 4 A when connected to a 2 V
−1 −1 battery. The resistivity of the wire is
(a) 4 ms (b) 0.4 ms
(a) 2 × 10−7 Ω-m (b) 5 × 10−7 Ω-m
(c) 4 cms−1 (d) 0.4 mms−1
(c) 4 × 10−6 Ω-m (d) 1 × 10−6 Ω-m
32 A metal rod of length 10 cm and a rectangular
cross-section of 1cm × (1/2) cm is connected to a 38 Three resistances P, Q, R each of 2 Ω and an
battery across opposite faces. The resistance will be unknown resistance S form the four arms of a
[NCERT Exemplar]
Wheatstone’s bridge circuit. When a resistance of
(a) maximum when the battery is connected across
1 cm × (1/2) cm faces 6 Ω is connected in parallel to S, the bridge gets
(b) maximum when the battery is connected across balanced. What is the value of S ?
10 cm × (1) cm faces (a) 2 Ω (b) 3 Ω
(c) maximum when the battery is connected across (c) 6 Ω (d) 1 Ω
10 cm × (1/2) cm faces 39 A 2 V battery, a 990 Ω resistor and a potentiometer
(d) same irrespective of the three faces
of 2 m length, all are connected in series. If the
33 Two cells of emfs approximately 5V and 10 V are to resistance of potentiometer wire is 10 Ω, then the
be accurately compared using a potentiometer of potential gradient of the potentiometer wire is
length 400 cm. [NCERT Exemplar] (a) 0.05 Vm−1 (b) 0.5 Vm−1
(a) The battery that runs the potentiometer should have (c) 0.01 Vm−1 (d) 0.1 Vm−1
voltage of 8V
(b) The battery of potentiometer can have a voltage of 40 The electron with charge q = (1.6 × 10 −19 C) moves in
15 V and R adjusted so that the potential drop across an orbit of radius 5 × 10 −11 m with a speed of
the wire slightly exceeds 10 V
2.2 × 10 6 ms −1, around an atom. The equivalent
(c) The first portion of 50 cm of wire itself should have a
potential drop of 10 V current is
(d) Potentiometer is usually used for comparing resistances (a) 1.12 × 10−6 A (b) 1.12 × 10−3 A
and not voltages (c) 1.12 × 10−9 A (d) 1.12 A
34 The resistivity of a potentiometer wire is 41 A potentiometer having the potential gradient of
40 × 10 −8 Ω-m and its area of cross-section is 2 mV/cm is used to measure the difference of
8 × 10 −6 m2 . If 0.2 A current is flowing through the potential across a resistance of 10 Ω in same circuit.
wire, the potential gradient will be If a length of 50 cm of the potentiometer wire is
required to get the null point, then the current
(a) 10−2 V /m (b) 10−1 V /m
passing through the 10 Ω resistor is (in mA)
(c) 3.2 × 10−2 V /m (d) 1 V/m (a) 1 (b) 2 (c) 5 (d) 10
35 Two cells of emfs E 1 and E 2 (E 1 > E 2 ) are connected 42 The n rows each containing m cells in series are
as shown in figure. joined in parallel. Maximum current is taken from
E1 E2 this combination across an external resistance of
A
B
C 3 Ω. If the total number of cells used are 24 and
internal resistance of each cell is 0.5 Ω, then
When a potentiometer is connected between A and (a) m = 8, n = 3 (b) m = 6, n = 4
B, the balancing length of the potentiometer wire is (c) m = 12, n = 2 (d) m = 2, n = 12
for More Books Click HERE -> NEETpassionate.com
216 OBJECTIVE Physics Vol. 2

43 A 100 V voltmeter of internal resistance 20 kΩ in 49 Equivalent resistance between the points A and B is
series with a high resistance R is connected to a (in Ω)
110 V line. The voltmeter reads 5 V, the value of R
is 1Ω 1Ω 1Ω 1Ω 1Ω
(a) 210 kΩ (b) 315 kΩ (c) 420 kΩ (d) 440 kΩ A B

44 A cell supplies a current i1 through a resistance R 1


and a current i 2 through a resistance R 2 . The 1 5 7 7
(a) (b) (c) (d)
internal resistance of this cell is 5 4 3 2
1 2 – i2R1
iR 50 In the circuit shown here, what is the value of the
(a) R 2 – R1 (b)
i1 – i2 unknown resistance R, so that the total resistance of
i R – iR i + i  the circuit between points P and Q is also equal to
(c) 2 2 1 1 (d)  1 2  R1R 2
i1 – i2  i1 – i2  R?
10 Ω
45 Out of five resistances of R Ω each, 3 are
connected in parallel and are joined to the rest 2 in 3Ω
P Q
series. Find the resultant resistance.
(a) (3 / 7) R Ω (b) (7/3) R Ω
3Ω R
(c) (7/ 8) R Ω (d) (8 / 7) R Ω
46 Two batteries A and B each of emf 2V are connected (a) 3 Ω (b) 39 Ω (c) 69 Ω (d) 10 Ω
in series to an external resistance R = 1 Ω. If the 51 Two wires of same metal have the same length but
internal resistance of battery A is 1.9 Ω and that of their cross-sections are in the ratio 3 : 1. They are
B is 0.9 Ω. What is the potential difference between joined in series. The resistance of the thicker wire is
the terminals of battery A? 10 Ω. The total resistance of the combination will be
A B 40 5
(a) 40 Ω (b) Ω (c) Ω (d) 100 Ω
3 2
52 If power dissipated in the 9 Ω resistor in the circuit
R
shown is 36 W, the potential difference across the
2 Ω resistor is
(a) 2 V (b) 3.8 V (c) zero (d) 4.8 V 9Ω
47 For a cell of emf 2 V, a balance is obtained for
50 cm of the potentiometer wire. If the cell is
shunted by a 2 Ω resistor, the balance is obtained 6Ω
across 40 cm of the wire. Find the internal
resistance of the cell.
(a) 0.25 Ω (b) 0.50 Ω
(c) 0.80 Ω (d) 1.00 Ω
2Ω
48 AB is a potentiometer wire of length 100 cm and its V
resistance is 10 Ω. It is connected in series with a (a) 8 V (b) 10 V (c) 2 V (d) 4 V
resistance R = 40 Ω and a battery of emf 2 V and 53 The reading of the ammeter in the following figure
negligible internal resistance. If a source of unknown will be
emf E is balanced by 40 cm length of the 6Ω
potentiometer wire, the value of E is
R 2V 4Ω 2V A

40 cm C
A B
3Ω
2Ω
E
(a) 0.8 V (b) 1.6 V (a) 0.8 A (b) 0.6 A (c) 0.4 A (d) 0.2 A
(c) 0.08 V (d) 0.16 V
for More Books Click HERE -> NEETpassionate.com
Current Electricity 217

54 A wire of length 100 cm is connected to a cell of 59 Two resistors 400 Ω and 800 Ω are connected in
emf 2 V and negligible internal resistance. The series with a 6 V battery. The potential difference
resistance of the wire is 3 Ω. The additional measured by voltmeter of 10 kΩ across 400 Ω
resistance required to produce a potential drop of resistor is
1 milli volt per cm is (a) 2 V (b) 1.95 V (c) 3.8 V (d) 4 V
(a) 60 Ω (b) 47 Ω
(c) 57 Ω (d) 35 Ω
60 A battery of emf E has an internal resistance r. A
variable resistance R is connected to the terminals of
55 Two uniform wires A and B are of same metal and the battery. A current i is drawn from the battery. V
have equal masses. The radius of wire A is twice is the terminal potential difference. If R alone is
that of wire B. The total resistance of A and B when gradually reduced to zero, which of the following
connected in parallel is best describes i and V ?
(a) 4 Ω when the resistance of wire A is 4.25 Ω (a) i approaches zero,V approaches E
(b) 5 Ω when the resistance of wire A is 4 Ω E
(c) 4 Ω when the resistance of wire B is 4.25 Ω (b) i approaches , V approaches zero
r
(d) 5 Ω when the resistance of wire B is 4 Ω E
(c) i approaches , V approaches E
56 In the given circuit, the resistances are given in r
ohm. The current through the 10 Ω resistance is 3 A (d) i approaches infinity, V approaches E
while that through the resistance X is 1 A. No current 61 Two resistances are connected in two gaps of a
passes through the galvanometer. The values of the meter bridge. The balance point is 20 cm from the
unknown resistances X and Y are respectively zero end. A resistance of 15 Ω is connected in series
(in ohm) with the smaller of the two. The null point shifts to
1A 40 cm. The value of the smaller resistance in ohm is
X 24 84 Y (a) 3 (b) 6
(c) 9 (d) 12
IB = 0
62 A battery of four cells in series, each having an emf
10 30 of 1.4 V and an internal resistance of 2 Ω is to be
3A used to charge a small 2 V accumulator of negligible
(a) 14 and 54 (b) 12 and 6 internal resistance. What is the charging current?
(c) 6 and 12 (d) 6 and 6 (a) 0.1 A (b) 0.2 A (c) 0.3 A (d) 0.45 A
57 The current i drawn from the 5 V source will be 63 The length of a wire of a potentiometer is 100 cm,
10 Ω and the emf of its cell is E volt. It is employed to
measure the emf of a battery whose internal
5Ω 10 Ω 20 Ω resistance is 0.5 Ω. If the balance point is obtained
at l = 30 cm from the positive end, the emf of the
10 Ω battery is
i 30E
(a)
100.5
30E
+ − (b)
100 − 0.5
5V 30(E − 0.5i )
(c) , where i is the current in the potentiometer
(a) 0.33 A (b) 0.5 A 100
(c) 0.67 A (d) 0.17 A wire
58 The current in the given circuit is 30E
(d)
10 W 5V 100
64 When a resistance of 100 Ω is connected in series
with a galvanometer of resistance R, then its range is
V. To double its range, a resistance of 1000 Ω is
connected in series. Find the value of R.
2V 20 W (a) 700 Ω (b) 800 Ω
(c) 900 Ω (d) 100 Ω
(a) 0.3 A (b) 0.4 A (c) 0.1 A (d) 0.2 A
for More Books Click HERE -> NEETpassionate.com
218 OBJECTIVE Physics Vol. 2

65 Two wires of the same material but of different 73 When a galvanometer is shunted by resistance S,
diameters carry the same current i. If the ratio of then its current capacity increases n times. If the
their diameters is 2 : 1, then the corresponding ratio same galvanometer is shunted by another resistance
of their mean drift velocities will be S′, then its current capacity will increase by n′,
(a) 4 : 1 (b) 1 : 1 which is given by
(c) 1 : 2 (d) 1 : 4 (n + 1)S S (n − 1) + S ′ n+S S (n − 1) − S ′
(a) (b) (c) (d)
66 Two bulbs consume same power when operated at S′ S′ S′ S′
200 V and 300 V, respectively. When these bulbs 74 The tungsten filaments of two electric bulbs are of
are connected in series across a DC source of 500 V, the same length. If one of them gives 25 W power
then and the other 60 W power, then
(a) ratio of potential differences across them is 3/2 (a) Both the filaments are of same thickness
(b) ratio of potential differences across them is 9/4 (b) 25 W bulb has thicker filament
(c) ratio of powers consumed across them is 4/9 (c) 60 W bulb has thicker filament
(d) ratio of powers consumed across them is 2/3 (d) Both the filaments have same cross-sectional area
67 A factory is served by a 220 V supply line. In a 75 Three unequal resistors in parallel are equivalent to
circuit protected by a fuse marked 10 A, the a resistance 1 Ω. If two of them are in the ratio 1 : 2
maximum number of 100 W lamps in parallel that and if no resistance value is fractional, then the
can be turned on, is largest of the three resistances (in ohms) is
(a) 11 (b) 22 (a) 4 (b) 6 (c) 8 (d) 12
(c) 33 (d) 66
76 In the circuit given here, the points A, B and C are at
68 A tap supplies water at 22°C, a man takes 1 L of 70 V, zero, 10 V, respectively. Then,
water per min at 37°C from the geyser. The power
of geyser is B
20 W
(a) 525 W (b) 1050 W (c) 1775 W (d) 2100 W
69 The mean free path of electrons in a metal is A
4 × 10 −8 m. The electric field which can give an 10 W D
30 Ω
average 2 eV energy to an electron in the metal will C
be in unit of Vm−1?
(a) the point D will be at a potential of 60 V
(a) 8 × 107 (b) 5 × 10−11 (c) 8 × 10−11 (d) 5 × 107
(b) the point D will be at a potential of 20 V
70 You are given two resistances R 1 and R 2 . By using (c) currents in the paths AD, DB and DC are in the ratio of
1:2 :3
them singly, in series and in parallel, you can obtain (d) currents in the paths AD, DB and DC are in the ratio of
four resistances of 1.5 Ω, 2 Ω, 6 Ω and 8 Ω. The 3 : 2 :1
values of R 1 and R 2 are
(a) 1 Ω, 7 Ω (b) 1. 5 Ω, 6. 5 Ω 77 The current in the resistance R will be zero if
E1 r1
(c) 3 Ω, 5 Ω (d) 2 Ω, 6 Ω
71 A potentiometer having the potential gradient of
2 mVcm −1 is used to measure the difference of
E2 r2
potential across a resistance of 10 Ω. If a length of
50 cm of the potentiometer wire is required to get R
the null points, then the current passing through E1 E 2
10 Ω resistor is (in mA) r = E 2r2
(a) E11 (b) =
r1 r2
(a) 1 (b) 2 (c) 5 (d) 10
(c) (E1 + E 2 )r1 = E1r2 (d) (E1 – E 2 )r1 = E 2r1
72 A galvanometer of resistance 50 Ω is connected to a
78 The magnitude and direction of the current in the
battery of 3 V along with a resistance of 2950 Ω in
series. A full scale deflection of 30 divisions is circuit shown will be
1Ω e 2Ω
obtained in the galvanometer. In order to reduce this a b
deflection to 20 divisions, the resistance in series 10 V 4 V
should be
(a) 4450 Ω (b) 5050 Ω 3Ω
d c
(c) 5550 Ω (d) 6050 Ω
for More Books Click HERE -> NEETpassionate.com
Current Electricity 219

7 7 83 In the below circuit, the battery E 1 has an emf of 12 V


(a) A from a to e (b) A from b to e
3 3
and zero internal resistance. While the battery E 2 has
(c) 1A from b to e (d) 1A from a to e
an emf of 2 V.
79 Consider a current carrying wire (current I) in the
500 Ω
shape of a circle. Note that as the currrent progresses G
along the wire, the direction of J (current density)
changes in an exact manner, while the current I E1 XΩ E2
remains unaffected. The agent that is essentially
responsible for [NCERT Exemplar]
(a) source of emf If the galvanometer G reads zero, then the value of
(b) electric field produced by charges accumulated on the the resistance X in ohms is
surface of wire (a) 250 (b) 100
(c) the charges just behind a given segment of wire which (c) 50 (d) 200
push them just the right way by repulsion
(d) the charges ahead 84 The potential drop across the 3 Ω resistor is
80. Two batteries of emf ε 1 and ε 2 (ε 2 >ε 1 ) and internal 3Ω

resistances r1 and r 2 respectively are connected in 4Ω


parallel as shown in figure. [NCERT Exemplar]
6Ω
ε1
r1

A B
3V
ε2 r2
(a) 1 V (b) 1.5 V
(a) The equivalent emf εeq of the two cells is between ε1 (c) 2 V (d) 3 V
and ε 2, i.e. ε1 < εeq < ε 2
85 Equivalent resistance between A and B will be
(b) The equivalent emf εeq is smaller than ε1
(c) The εeq is given by εeq = ε1 + ε 2 always
(d) εeq is independent of internal resistances r1 and r2 3Ω 3Ω
81 A resistance R is to be measured using a meter
bridge, student chooses the standard resistance S to
be 100 Ω. He finds the null point at l1 = 2.9 cm. He
3Ω 3Ω
is told to attempt to improve the accuracy. 3Ω 3Ω
Which of the following is a useful way?
[NCERT Exemplar]
(a) He should measure l1 more accurately A B
(b) He should change S to 1000 Ω and repeat the 3Ω 3Ω
experiment
(a) 2 Ω (b) 18 Ω
(c) He should change S to 3 Ω and repeat the experiment (c) 6 Ω (d) 3.6 Ω
(d) He should given up hope of a more accurate
measurement with a meter bridge 86 In the network of resistors shown in the figure, the
equivalent resistance between A and B is
82 The current drawn from the battery shown in the
figure is
R 3Ω 3Ω 3Ω 3Ω 3Ω 3Ω
A B
V
R
3Ω 3Ω 3Ω 3Ω 3Ω 3Ω
R
(a) 54 Ω (b) 18 Ω
V V 2V 3V (c) 36 Ω (d) 9 Ω
(a) (b) (c) (d)
R 2R R 2R
for More Books Click HERE -> NEETpassionate.com
220 OBJECTIVE Physics Vol. 2

87 In the figure shown, the total resistance between A 92 Each of the resistance in the network shown in the
and B is figure is equal to R. The resistance between the
2Ω 1Ω 1Ω 1Ω 1Ω 1Ω terminals A and B is
C
A L

8Ω 8Ω 4Ω
R
R R
B
2Ω D 1Ω 1Ω 1Ω 1Ω 1Ω
R A
B
(a) 12 Ω (b) 4 Ω (c) 6 Ω (d) 8 Ω R
K
88 The potential difference in volt
across the resistance R 3 in the (a) R (b) 5R (c) 3R (d) 6R
circuit shown in figure, is 50 V 93 Find the equivalent resistance across AB.
R1
(R 1 = 15 Ω, R 2 = 15 Ω, A
R3
R 3 = 30 Ω, R 4 = 35 Ω)
R2 2Ω 2Ω
(a) 5
(b) 7.5 2Ω
(c) 15 R4
2Ω 2Ω
(d) 12.5
89 In the given circuit, the voltmeter records 5 V. The B
resistance of the voltmeter in Ω is (a) 1 Ω (b) 2 Ω (c) 3 Ω (d) 4 Ω
V 94 The equivalent resistance between P and Q in the
100 Ω 50 Ω
figure is approximately
P
Q
8Ω
10 V

6Ω 8Ω
(a) 200 (b) 100 (c) 10 (d) 50
10 Ω
90 In given figure, the potentiometer wire AB has a
resistance of 5 Ω and length 10 m. The balancing
(a) 6 Ω (b) 5 Ω (c) 7.5 Ω (d) 20 Ω
length AM for the emf of 0.4 V is
R = 45 Ω 95 In the given network of resistances, the effective
resistance between A and B is
5V

M R R
A B R

A R R R B
0.4 V
(a) 0.4 m (b) 4 m (c) 0.8 m (d) 8 m R R

91. In the circuit shown below, the readings of the


ammeter and voltmeter are 5 8
(a) R (b) R (c) 5R (d) 8R
6V1Ω 3 3
96 A source of emf E = 15 V and having negligible
internal resistance is connected to a variable
V
6Ω
A resistance, so that the current in the circuit increases
4Ω with time as i = 1.2 t + 3. Then, the total charge that
will flow in first five second will be
(a) 6 A, 60 V (b) 0.6 A, 6 V (a) 10 C (b) 20 C
(c) 6 A, 6 V (d) (6/11) A, (6/11) V (c) 30 C (d) 40 C
for More Books Click HERE -> NEETpassionate.com
Current Electricity 221

97 The effective resistance between points A and C for 100 An electric immersion heater of 1.08 kW is
the network shown in figure is immersed in water. After the water has reached a
R temperature of 100°C, how much time will be
A B
R
required to produce 100 g of steam?
R (a) 50 s (b) 420 s
R R (c) 105 s (d) 210 s
R O R
101 A moving coil galvanometer is converted into an
ammeter reading upto 0.03 A by connecting a shunt
D C
R of resistance 4r across it and into an ammeter
2 3 1 reading upto 0.06 A when a shunt of resistance r is
(a) R (b) R (c) 2R (d) connected across it. What is the maximum current
3 2 2R
which can be sent through this galvanometer, if no
98 Three resistances 5 Ω, 5 Ω and 6 Ω are connected as shunt is used?
shown in figure. If the point S divides the resistance (a) 0.01 A (b) 0.02 A
6 Ω into two equal halves, the resistance between (c) 0.03 A (d) 0.04 A
points P and S is
102 B1, B 2 and B 3 are the three identical bulbs
connected to a battery of steady emf with key K
closed. What happens to the brightness of the bulbs
5Ω P 5Ω B1 and B 2 when the key is opened?
S
B1
K
Q R B2
6Ω
B3
(a) 11 Ω (b) 8 Ω (c) 6 Ω (d) 4 Ω
99 A potentiometer circuit is set up as shown. The (a) Brightness of the bulb B1 increases and that of B 2
potential gradient across the potentiometer wire, is decreases
k volt/cm and the ammeter, present in the circuit, (b) Brightness of the bulbs B1 and B 2 increases
(c) Brightness of the bulb B1 decreases and B 2 increases
reads 1.0 A when two way key is switched off. The
(d) Brightness of the bulbs B1 and B 2 decreases
balance points, when the key between the terminals
(i) 1 and 2 (ii) 1 and 3, is plugged in, are found to be 103 The scale of a galvanometer of resistance 100 Ω
at lengths l1 cm and l 2 cm, respectively. The contains 25 divisions. It gives a deflection of one
magnitudes of the resistors R and X in ohm, are division on passing a current of 4 × 10 −4 A. The
then, respectively equal to resistance (in ohm) to be added to it, so that it may
become a voltmeter of range 2.5 V is
+ − (a) 150 (b) 170
(c) 110 (d) 220
104 Three electric bulbs of 200 W, 200 W and 400 W
are connected as shown in figure. The resultant
power of the combination is
A B
G 200 W
1
2
3 400 W

R X

− 200 W
A
+
~

(a) k (l 2 − l1) and kl 2 (b) kl1 and k (l 2 − l1) (a) 800 W (b) 400 W
(c) k (l 2 − l1) and kl 2 (d) kl1 and kl 2 (c) 200 W (d) 600 W
for More Books Click HERE -> NEETpassionate.com
222 OBJECTIVE Physics Vol. 2

105 Two electric bulbs rated 50 W and 100 V are What is the value of current i in circuit?
glowing at full power, when used in parallel with a (a) 15 A (b) 10 A
battery of emf 120 V and internal resistance 10 Ω. (c) 20 A (d) 5 A
The maximum number of bulbs that can be 110 The charge flowing in a conductor varies with time
connected in the circuit when glowing at full as Q = at – bt 2 , then the current
power, is (a) reaches a maximum and then decreases
(a) 6 (b) 4 (c) 2 (d) 8 a
(b) falls to zero after t =
106 The equivalent resistance between points A and B of 2b
an infinite network of resistances, each of 1 Ω, (c) changes at a rate of (– 2b)
connected as shown, is (d) Both (b) and (c)
A 1Ω 1Ω 1Ω 111 In the circuit shown in figure, ammeter and
voltmeter are ideal. If E = 4 V, R = 9 Ω and r = 1 Ω,
then readings of ammeter and voltmeter are
1Ω 1Ω 1Ω
B V

R R
(a) infinite (b) 2 Ω E, r R
1+ 5
(c) Ω (d) zero A
2
107 In the given figure, the current through the 20 V (a) 1 A, 3 V (b) 2 A, 3 V
battery is (c) 3 A, 4 V (d) 4 A, 4 V
112 In the circuit shown, the current in 3 Ω resistance
16 V 16 V 20 V
is
2Ω 3Ω
2Ω 4Ω

10 V 2Ω 3Ω
(a) 11 A (b) 12 A (c) 7 A (d) 14 A
108 The current in resistance R 3 in the given circuit is
1
2V 3V 4V (a) 1 A (b) A
7
5 15
(c) A (d) A
R1 = 2 Ω R2 = 2 Ω R3 = 3 Ω 7 7
113 Under what conditions, current passing through the
resistance R can be increased by short circuiting the
2V 3V 2V battery of emf E 2 ? The internal resistances of the
two batteries are r1 and r 2, respectively.
(a) 1 A (b) 2/3 A (c) 0.25 A (d) 0.50 A
109 In the circuit shown in figure, the resistance R has a E1 E2
value that depends on the current. Specifically R is
20 Ω when i is zero and the amount of increase in
resistance is numerically equal to one-half of the
current.
R
i
(a) E 2r1 > E1(R + r2 )
250 V R (b) E1r2 < E 2 (R + r1)
(c) E 2r2 < E1(R + r2 )
(d) E1r2 > E 2 (R + r1)
for More Books Click HERE -> NEETpassionate.com
Current Electricity 223

114 In the arrangement shown, the magnitude of each (a) 3R (b) 4R


resistance is 1 Ω. The equivalent resistance between O 5R 7R
(c) (d)
and A is given by 2 2
C 119 All resistances shown in circuit are 2 Ω each. The
current in the resistance between D and E is
A B

B D D
O C E
10 V

F H
G

A (a) 5 A (b) 2.5 A (c) 1 A (d) 7.5 A


14 3 2 5 120 A battery of emf 10 V is connected to a group of
(a) Ω (b) Ω (c) Ω (d) Ω
13 4 3 6 resistances as shown in figure. The potential
115 Find the reading of the ideal ammeter connected in the differenceVA − VB between the points A and B is
given circuit. Assume that the cells have negligible 1Ω A 3Ω
3Ω
internal resistance.
3Ω 1Ω
10 V 4V 8V 6 V
B
10 V

10 Ω 8Ω 4Ω 2Ω A
(a) –2 V (b) 2 V
20
(c) 5 V (d) V
11
(a) 0.8 A (b) 0.25 A (c) 1.95 A (d) 1.0 A 121 In the circuit shown, when key K 1 is closed, then
116 A moving coil galvanometer has 150 equal divisions. Its the ammeter reads I 0 whether K 2 is open or
current sensitivity is 10 divisions per milliampere and closed. But when K 1 is open the ammeter reads
voltage sensitivity is 2 divisions per millivolt. In order I 0 /2, when K 2 is closed.
that each division reads 1 V, the resistance (in ohms)
needed to be connected in series with the coil will be K1 K2 100 Ω
3 5
(a) 99995 (b) 9995 (c) 10 (d) 10
117 It takes 16 min to boil some water in an electric kettle. A
R1 R2 = 100 Ω
Due to some defect it becomes necessary to remove
10% turns of heating coil of the kettle. After repairs,
how much time will it take to boil the same mass of
E, r
water?
(a) 17.7 min (b) 14.4 min (c) 20.9 min (d) 13.9 min Assuming that ammeter resistance is much less
than R 2 , the values of r and R 1 in ohms are
118 Equivalent resistance between points A and B is (a) 100, 50 (b) 50, 100
R R
(c) 0, 100 (d) 0, 50
122 In the circuit shown in figure, V must be
R
+
4Ω 20 Ω 100 Ω 25 Ω
R
R R R
V
A B 6Ω
R 4A

R
(a) 50 V (b) 80 V
R (c) 100 V (d) 1290 V
for More Books Click HERE -> NEETpassionate.com
224 OBJECTIVE Physics Vol. 2

123 Current through wire XY of circuit shown is 124 In the circuit shown in figure, the ratio of currents
1Ω X 2Ω i1 /i 2 is
2Ω 4Ω 3Ω

2Ω 8Ω 1Ω
3Ω Y 4Ω
16 V
i2 i1
8Ω 8Ω
50 V
8V
(a) 1 A (b) 4 A
(c) 2 A (d) 3 A (a) 2 (b) 8 (c) 0.5 (d) 4

(B) Medical entrance special format questions


Assertion and reason
Directions (Q. Nos. 1-5) These questions consist of two 5 Assertion Two resistance wires shown in
statements each linked as Assertion and Reason. While answering figure are of same material. They have equal
these questions you are required to choose any one of the following length. More heat is generated in wire A.
four responses. B
(a) If both Assertion and Reason are true and Reason is the correct A
i i
explanation of Assertion.
(b) If both Assertion and Reason are true but Reason is not correct
explanation of Assertion.
(c) If Assertion is true but Reason is false.
Reason In series H ∝ R and resistance of
(d) If Assertion is false but Reason is true. wire A is more.
1 Assertion If a current flows through a wire of non-uniform
cross-section, potential difference per unit length of wire is
Statement based questions
same throughout the length of wire. 1 For the circuit shown in the figure, which of
the following statement is incorrect?
Reason Current through the wire is same at all
cross-sections. R N
2 Assertion In our houses when we start switching on 0.5 A
different light buttons, main current goes on increasing. R1 20 Ω
10 Ω
Reason Different connections in houses are in parallel.
1A
When we start switching on different light buttons, then
P
net resistance of the circuit decreases. Therefore, main 69 V
current increases.
(a) The current through NP is 0.5 A.
3 Assertion Resistance of an ammeter is less than the (b) The value of R1 = 40 Ω.
resistance of a milliammeter. (c) The value of R = 14 Ω.
Reason Value of shunt required in case of ammeter is (d) The potential difference across R = 49 V.
more than a milliammeter. 2 Two non-ideal batteries are connected in
series. Consider the following statements.
4 Assertion In the circuit shown in figure, battery is ideal.
If a resistance R 0 is connected in parallel with R, then I. The equivalent emf is larger than either of
the two emfs.
power across R will increase.
II. The equivalent internal resistance is smaller
than either of the two internal resistances.
R
(a) Each of I and II is correct
(b) I is correct but II is wrong
E (c) II is correct but I is wrong
Reason Current drawn from the battery will increase. (d) Each of I and II is wrong
for More Books Click HERE -> NEETpassionate.com
Current Electricity 225

3 Consider the following statements regarding the


network shown in the figure. Match the columns
R R 1 In the circuit diagram shown in figure, match the
following two columns when switch S is closed.
A B Choose the correct option from codes given below.
G
R
i1
S
2R 2R A B
i2 R

+ – C r D
E
E
I. The equivalent resistance of the network between Column I Column II
points A and B is independent of value of G.
A. Current (i1) (p) will increase
II. The equivalent resistance of the network between
points A and B is 4 / 3 R. B. Current (i 2 ) (q) will decrease
III. The current through G is zero. C. Potential difference (r) will remain same
Which of the above statement(s) is/are true? across A and B
(a) Only I (b) Only II
D. Potential difference (s) may increase or decrease. It
(c) Both II and III (d) I, II and III across C and D will depend on value of r
4 Two non-ideal unidentical batteries are connected in
parallel with positive terminals. Consider the Codes
A B C D A B C D
following statements.
(a) p q q q (b) q p r r
I. The equivalent emf is smaller than either of the two
(c) p s r q (d) s s r p
emfs.
II. The equivalent internal resistance is smaller than 2 In the circuit diagram shown in figure, potential
either of the two internal resistances. difference across 3 Ω resistance is 20 V. Then,
(a) Both I and II are correct match the following two columns and choose the
(b) I is correct but II is wrong correct option from codes given below.
(c) II is correct but I is wrong 12 Ω 3Ω
(d) Both I and II are wrong
5 IfVA − VB = V0 and the value of each resistance is 8Ω
4Ω 6Ω
R, then
C D

A E F B

Column I Column II

R A. Potential difference across 6 Ω resistance (p) 30 V


I. net resistance between AB is .
2 B. Potential difference across 4 Ω resistance (q) 40 V
3R
II. net resistance between AB is . C. Potential difference across 12 Ω resistance (r) 20 V
5
V0 D. Potential difference across 8 Ω resistance (s) 80 V
III. current through CD is .
R Codes
2V0 A B C D
IV. current through EF is .
3R (a) p r s q
(b) p r r s
Which of the statement(s) is/are correct? (c) r p p s
(a) I and II (b) I and III (c) II and III (d) All of these (d) r q s p
for More Books Click HERE -> NEETpassionate.com
OBJECTIVE Physics Vol. 2

(C) Medical entrances’ gallery


Collection of questions asked in NEET and various medical entrance exams
1 A charged particle having drift velocity of 6 Two solid conductors are made up of same material,
7.5 × 10 −4 ms −1 in an electric field of have same length and same resistance. One of them has
3 × 10 −10 Vm −1, has a mobility (in m 2 V −1s −1) of a circular cross-section of area A1 and the other one
[NEET 2020]
has a square cross-section of area A2 . The ratio
(a) 2.5 × 106 (b) 2.5 × 10−6
A1 / A2 is [NEET 2020]
(c) 2.25 × 10−15 (d) 2.25 × 1015 (a) 1.5 (b) 1 (c) 0.8 (d) 2
2 The color code of a resistance is given below 7 For the circuit given below, the Kirchhoff’s loop rule
for the loop BCDEB is given by the equation
[NEET 2020]
i1 R2 i2
Yellow Violet Brown Gold A B C

The values of resistance and tolerance respectively, R1


are [NEET 2020] E1 E2
(a) 47 kΩ, 10% (b) 4.7 k Ω, 5% i3
(c) 470 Ω, 5% (d) 470 kΩ, 5% F E D
E3
3 A resistance wire connected in the left gap of a
metre bridge balances a 10 Ω resistance in the right (a) − i2R 2 + E 2 − E 3 + i3R1 = 0
gap at a point which divides the bridge wire in the (b) i2R 2 + E 2 − E 3 − i3R1 = 0
ratio 3 : 2. If the length of the resistance wire is (c) i2R 2 + E 2 + E 3 + i3R1 = 0
1.5 m, then the length of 1 Ω of the resistance wire (d) −i2R 2 + E 2 + E 3 + i3R1 = 0
is [NEET 2020] 8 The equivalent resistance between A and B for the
(a) 1.0 × 10−1 m (b) 1.5 × 10−1 m mesh shown in the figure is [NEET 2020]
(c) 1.5 × 10−2 m (d) 1.0 × 10−2 m
4Ω 8Ω
4 Which of the following graph represents the
variation of resistivity (ρ) with temperature (T ) for
copper? [NEET 2020] 6Ω
4Ω 8Ω

ρ ρ
A B
(a) (b)
. Ω
(a) 72 (b) 16 Ω (c) 30 Ω (d) 4.8 Ω

T T 9 Six similar bulbs are connected as shown in the


ρ ρ
figure with a DC source of emf E and zero internal
resistance.
(c) (d) The ratio of power consumption by the bulbs when
(i) all are glowing and (ii) in the situation when two
T T from section A and one from section B are glowing,
will be [NEET 2019]
5 For the circuit shown in the figure, the current I will A B
be [NEET 2020]
2V 1Ω 4V 1Ω

I 4Ω
E
(a) 0.75 A (b) 1 A
(c) 1.5 A (d) 0.5 A (a) 9 : 4 (b) 1 : 2 (c) 2 : 1 (d) 4 : 9
for More Books Click HERE -> NEETpassionate.com
Current Electricity 227

10 Which of the following acts as a circuit protection 15 A current of 10 A is passing through a metallic wire
device? [NEET 2019] of cross-sectional area 4 × 10 −6 m 2 . If the density of
(a) Inductor (b) Switch (c) Fuse (d) Conduct the aluminium conductor is 2.7 g/cc, considering
11 In the circuits shown below, the readings of aluminium gives 1 electron per atom for conduction,
voltmeters and the ammeters will be [NEET 2019] then find the drift velocity of the electrons if
molecular weight of aluminium is 27 g. [AIIMS 2019]
10 Ω i1 10 Ω i2
(a) 1.6 × 10−4 m/s (b) 3.6 × 10−4 m/s
10 Ω (c) 2.6 × 10−4 m/s (d) 1.5 × 10−4 m/s
V1 V2 A2
A1
16 A circuit contain two resistances R 1and R 2 are in
series. Find the ratio of input voltage to voltage of
10 V 10 V R 2. [JIPMER 2019]
R2 R1 + R 2
(a) (b)
(a)V1 = V2 and i1 > i2 (b)V1 = V2 and i i = i2 R1 + R 2 R2
(c)V2 > V1 and i1 > i2 (d)V2 > V1 and i1 = i2 R1 + R 2 R1
(c) (d)
12 The meter bridge shown in the balance position with R1 R1 + R 2
P l1
= . If we now interchange the positions of 17 In the given circuit, find voltage across 12 Ω
Q l2
resistance. [JIPMER 2019]
galvanometer and cell, will the bridge work? If yes,
8Ω
that will be balanced condition? [NEET (Odisha) 2019]
8Ω 4Ω

P Q 4Ω
12A
G
12Ω

l1 l2
(a) 12 V (b) 36 V (c) 72 V (d) 48 V
P l −l 18 Find the current in the 8Ω resistance in the given
(a) Yes, = 2 1 (b) No, no null point
Q l 2 + l1 circuit. [JIPMER 2019]
P l P l1
(c) Yes, = 2 (d) Yes, =
Q l1 Q l2
3W 3W
13 The reading of an ideal voltmeter in the circuit 3W 3W
shown is [NEET (Odisha) 2019]
20 Ω 30 Ω
3W

30 Ω 20 Ω 8W
24 V

2V (a) 2 A (b) 3A
(c) 4 A (d) 5 A
(a) 0.6 V (b) 0 V (c) 0.5 V (d) 0.4 V
. × 10 −8 Ω-m and
19 If resistivity of copper is 172
R 1
14 For a wire = and length of wire is l = 5 cm. number of free electrons in copper is
l 2
8.5 × 10 28 / m3 . Find the mobility. [JIPMER 2019]
If potential difference of 1 V is applied across it, −3 2 −3 2
(a) 4. 25 × 10 m / CΩ (b) 6.8 × 10 m / CΩ
then current through wire will be [AIIMS 2019] −3
(c) 8.5 × 10 m / CΩ 2
(d) 3.4 × 10−3m2 / CΩ
(a) 40 A (b) 4 A (c) 25 A (d) 2.5 A
for More Books Click HERE -> NEETpassionate.com
228 OBJECTIVE Physics Vol. 2

20 In the given circuit, if power rating of heater is (a) 0.5 A (b) 0.2 A
1000 W at 100 V, find the resistance R in figure so (c) 1 A (d) 2 A
that heater produces power of 62.5 W. [JIPMER 2019] 25 Assertion Terminal voltage of a cell is greater than
Heater emf of cell during charging of the cell. [AIIMS 2018]
10W Reason The emf of a cell is always greater than its
R terminal voltage.
(a) Both Assertion and Reason are correct and Reason is the
correct explanation of Assertion.
(b) Both Assertion and Reason are correct but Reason is not
100 V
the correct explanation of Assertion.
(a) 5 Ω (b) 7 Ω (c) 10 Ω (d) 8 Ω (c) Assertion is correct but Reason is incorrect.
21 A carbon resistor of (47 ± 4.7) kΩ is to be marked (d) Assertion is incorrect but Reason is correct.
with rings of different colours for its identification. 26 The effective resistance between P and Q in given
The colour code sequence will be [NEET 2018] figure is [AIIMS 2018]
(a) Yellow - Green - Violet - Gold 6W 8W
(b) Yellow - Violet - Orange - Silver 3W 8W
P Q
(c) Violet - Yellow - Orange - Silver 10W
(d) Green - Orange - Violet - Gold 10W 5W
4W W
20

4W
22 A set of n equal resistors, of value R each , are
connected in series to a battery of emf E and internal 4W
resistance R. The current drawn is I. Now, the n (a) 2Ω (b) 3Ω (c) 5Ω (d) 6Ω
resistors are connected in parallel to the same
battery. Then, the current drawn from battery 27 Assertion Bulb generally get fused when they are
becomes 10I. The value of n is [NEET 2018]
switched ON or OFF.
(a) 20 (b) 11 (c) 10 (d) 9 Reason When we switch ON or OFF a circuit,
current changes in it rapidly. [AIIMS 2018]
23 A battery consists of a variable number n of identical (a) Both Assertion and Reason are correct and Reason is the
cells (having internal resistance r each) which are correct explanation of Assertion.
connected in series. The terminals of the battery are (b) Both Assertion and Reason are correct but Reason is not
short-circuited and the current I is measured. Which the correct explanation of Assertion.
of the graphs shows the correct relationship between (c) Assertion is correct and Reason is incorrect.
I and n ? [NEET 2018] (d) Assertion is incorrect and Reason is correct.
I I 28 A current i is flowing through the wire of diameter
d having drift velocity of electrons v d in it. What
(a) (b) will be new drift velocity when diameter of wire is
made d/4? [JIPMER 2018]
O O n
n vd vd
(a) 4v d (b) (c) 16 v d (d)
I I 4 16
(c) (d) 29 The resistance of a wire is R ohm. If it is melted and
stretched to n times its original length, its new
O
O resistance will be [NEET 2017]
n n
R R
(a) nR (b) (c) n 2R (d)
24 Find current (i ) in circuit shown in figure. [NEET 2018] n n2
B
30 Find the value of R net between A and B. [NEET 2017]
5W 10W 10W 10W 10W
A
A D
10W
40W 40W 20W
10W 20W
i
C B
10W 10W 10W
5V (a) 60 Ω (b) 40 Ω (c) 70 Ω (d) 20 Ω
for More Books Click HERE -> NEETpassionate.com
Current Electricity 229

31 FindVP − VQ in the circuit shown in below figure. (b) Both Assertion and Reason are correct but Reason is not
[NEET 2017] the correct explanation of Assertion.
(c) Assertion is correct Reason is incorrect.
10W
(d) Assertion is incorrect but Reason incorrect.
10W Q
P R 36 You are given resistance wire of length 50 cm and a
18W
battery of negligible resistance. In which of the
8W 25W following cases is largest amount of heat generated?
[JIPMER 2017]
+ –
(a) When the wire is connected to the battery directly.
(b) When the wire is divided into two parts and both the
10 V parts are connected to the battery in parallel.
(c) When the wire is divided into four parts and all the
(a) 6.68 V (b) 8 V (c) 4.65 V (d) 7 V
four parts are connected to the battery in parallel.
32 Find the value of i in shown below figure. [NEET 2017] (d) When only half of the wire is connected to the
battery.
30 Ω
37 The current passing through the ideal ammeter in
i1 the circuit given below is [KCET 2017]
60 Ω 2Ω
i2 4V 2Ω
1Ω
i

2V 2Ω

(a) 0.2A (b) 0.1A (c) 0.3A (d) 0.4A 4Ω


A
33 N lamps each of resistance r are fed by a machine
of resistance R. If light emitted by any lamp is (a)1.25A (b) 1A (c) 0.75A (d) 0.5 A
proportional to the square of the heat produced, 38 A potentiometer wire is 100 cm long and a constant
prove that the most efficient way of arranging them potential difference is maintained across it. Two
is to place them in parallel arcs, each containing n cells are connected in series first to support one
lamps, where n is the integer nearest to [NEET 2017] another and then in opposite direction. The balance
 r 
3/ 2
 NR 
1/ 2 points are obtained at 50 cm and 10 cm from the
(a)   (b)   positive end of the wire in the two cases. The ratio
 NR   r 
of emfs is [NEET 2016]
(c) (NRr )3/ 2 (d) (NRr )1/ 2
(a) 5 : 4 (b) 3 : 4 (c) 3 : 2 (d) 5 : 1
34 Two batteries, one of emf 18V and internal 39 The charge flowing through a resistance R varies
resistance 2Ω and the other of emf 12V and internal
with time t as Q = at − bt 2 , where a and b are
resistance 1Ω are connected as shown in figure. The
voltmeterV will record a reading of [AIIMS 2017] positive constants. The total heat produced in R is
[NEET 2016]
V a 3R a 3R a 3R a 3R
(a) (b) (c) (d)
3b 2b b 6b
2Ω
40 The potential difference (VA −VB ) between the
18V points A and B in the given figure is [NEET 2016]
1Ω
3V
VA 2W + – 1W VB
12V
I=2A B
(a) 14 V (b) 15 V (c) 18 V (d) 30 V (a) –3 V (b) +3 V (c) +6 V (d) +9 V
35 Assertion A potentiometer is preferred over that of 41 A filament bulb (500 W, 100 V) is to be used in a
a voltmeter for measurement of emf of a cell. 230 V main supply. When a resistance R is
Reason Potentiometer does not draw any current connected in series, it works perfectly and the bulb
from the cell. [AIIMS 2017] consumes 500 W. The value of R is [NEET 2016]
(a) Both Assertion and Reason are correct and Reason is the (a) 230 Ω (b) 46 Ω
correct explanation of Assertion. (c) 26 Ω (d) 13 Ω
for More Books Click HERE -> NEETpassionate.com
230 OBJECTIVE Physics Vol. 2

42 A potentiometer wire has length 4 m and resistance 48 The range of voltmeter is 10 V and its internal
8 Ω. The resistance that must be connected in series resistance is 50 Ω. To convert it to a voltmeter of
with the wire and an accumulator of emf 2 V, so as range 15 V, how much resistance is to be added?
to get a potential gradient of 1 mV per cm on the (a) Add 25 Ω resistor in parallel [EAMCET 2015]
wire is [CBSE AIPMT 2015] (b) Add 25 Ω resistor in series
(a) 32 Ω (b) 40 Ω (c) 44 Ω (d) 48 Ω (c) Add 125 Ω resistor in parallel
(d) Add 125 Ω resistor in series
43 A, B and C are voltmeters of resistance R, 1.5 R and
3R respectively, as shown in the figure. When some 49 Identify the wrong statement. [Kerala CEE 2015]
potential difference is applied between X andY, then (a) Charge is a vector quantity
the voltmeter readings areVA, VB andVC , (b) Current is a scalar quantity
respectively. Then, [CBSE AIPMT 2015]
(c) Charge can be quantised
(d) Charge is additive in nature
B (e) Charge is conserved
A
X Y 50 When the rate of flow of charge through a metallic
C
conductor of non-uniform cross-section is uniform,
(a) VA = VB = VC (b) VA ≠ VB = VC then the quantity that remains constant along the
(c) VA = VB ≠ VC (d) VA ≠ VB ≠ VC conductor is [Kerala CEE 2015]
44 Across a metallic conductor of non-uniform (a) current density (b) electric field
cross-section, a constant potential difference is (c) electric potential (d) drift velocity
applied. The quantity which remain(s) constant along (e) current
the conductor is [CBSE AIPMT 2015] 51 The resistance of a carbon resistor of colour code
(a) current density (b) current Red-Red-Green-Silver is (in kΩ) [Kerala CEE 2015]
(c) drift velocity (d) electric field (a) 2200 ± 5% (b) 2200 ± 10%
45 Consider the diagram shown below [AIIMS 2015] (c) 220 ± 10% (d) 220 ± 5%
(e) 2200 ± 1%
B
A C
100 Ω 100 Ω 52 The slope of the graph showing the variation of
potential differenceV on X-axis and current on
Y-axis gives conductor [Kerala CEE 2015]
50 V (a) resistance (b) resistivity
A voltmeter of resistance 150 Ω is connected across (c) reciprocal of resistance (d) conductivity
(e) impedance
A and B. The potential drop across B and C measured
by voltmeter is 53 Two wires of equal length and equal diameter and
(a) 29 (b) 27 V (c) 31 V (d) 30 V having resistivities ρ1 and ρ 2 are connected in series.
46 Each resistor shown in the figure has a resistance of The equivalent resistivity of the combination is
[Guj. CET 2015]
10 Ω and the battery has the emf 6 V. What will be ρ + ρ2
the current supplied by the battery? [UK PMT 2015] (a) 1 (b) ρ1 + ρ2
2
ρ1 ρ2
10 Ω 10 Ω (c) (d) ρ1 ρ2
10 Ω ρ1 + ρ2
54 A galvanometer of resistance 50 Ω is connected to a
10 Ω Y 10 Ω
battery of 8 V along with a resistance of 3950 Ω in
series. A full scale deflection of 30 divisions is
6V obtained in the galvanometer . In order to reduce
this deflection to 15 divisions, the resistance in
(a) 0.6 A (b) 1.2 A (c) 1.8 A (d) 0.3 A
series should be … Ω. [Guj. CET 2015]
47 A 1 Ω resistance in series with an ammeter is (a) 1950 (b) 7900
balanced by 75 cm of potentiometer wire. A (c) 2000 (d) 7950
standard cell of emf 1.02 V is balanced by 50 cm. 55 Choose the correct statement. [CG PMT 2015]
The ammeter shows a reading of 1.5 A. Then, the
(a) Kirchhoff’s first law of electricity is based on
error in ammeter reading is [EAMCET 2015]
conservation of charge while the second law is based
(a) 0.03 A (b) 3A (c) 1.3 A (d) 0.3 A on conservation of energy.
for More Books Click HERE -> NEETpassionate.com
Current Electricity 231

(b) Kirchhoff’s first law of electricity is based on 62 Three resistances 2 Ω, 3 Ω and 4 Ω are connected in
conservation of energy while the second law is based
on conservation of charge.
parallel. The ratio of currents passing through them
when a potential difference is applied across its ends
(c) Kirchhoff’s both laws are based on conservation of
charge. will be [KCET 2015]
(d) Kirchhoff’s both laws are based on conservation of (a) 5 : 4 : 3 (b) 6 : 3 : 2
energy. (c) 4 : 3 : 2 (d) 6 : 4 : 3

56 A metal plate weighing 750 g is to be electroplated 63 Four identical cells of emf ε and internal resistance r
with 0.05% of its weight of silver. If a current of are to be connected in series. Suppose, if one of the
0.8 A is used, find the time (approx.) needed for cell is connected wrongly, then the equivalent
depositing the required weight of silver (ECE of emf and effective internal resistance of the
silver is 11.8 × 10 −7 kgC −1 ) combination is [KCET 2015]
[CG PMT 2015]
(a) 2E and 4r (b) 4E and 4r
(a) 5 min 32 s (b) 6 min 37 s (c) 2E and 2r (d) 4E and 2r
(c) 4 min 16 s (d) 6 min 10 s
64 In the circuit shown alongside, the ammeter and the
. Ω and its current
57 A DC ammeter has resistance 01 voltmeter readings are 3A and 6V, respectively.
ranges 0-100 A. If the range is to be extended to Then, the value of the resistance R is [KCET 2015]
0-500, then the following shunt resistance will be
R
required [ CG PMT 2015] A
(a) 0.010 Ω (b) 0.011 Ω
(c) 0.025 Ω (d) 0.25 Ω
58 The current I shown in the circuit is [WB JEE 2015] V

2V + (a) < 2 Ω (b) 2 Ω
2Ω
(c) ≥ 2 Ω (d) > 2 Ω
2Ω 2Ω
2V
+
2V + 65 The resistance of a bulb filament is 100 Ω at a
– I –
temperature of 100° C. If its temperature coefficient
(a) 1.33 A (b) zero (c) 2 A (d) 1 A of resistance be 0.005 per ° C, then its resistance will
become 200 Ω at a temperature [KCET 2015]
59 A metal wire of circular cross-section has a (a) 500° C (b) 300° C (c) 200° C (d) 400° C
resistance R 1. The wire is now stretched without
breaking, so that its length is doubled and the 66 A and B are the two points on a uniform ring of
density is assumed to remain the same. If the radius r. The resistance of the ring is R and
resistance of the wire now becomes R 2 , then R 2 : R 1 ∠AOB = θ as shown in the figure. The equivalent
is [WB JEE 2015] resistance between points A and B is [Guj. CET 2015]
(a) 1 : 1 (b) 1 : 2 (c) 4 : 1 (d) 1 : 4 A
60 Consider the combination of resistor, B
r θ
r
a b
R R R O

The equivalent resistance between a and b is


[UP CPMT 2015]
R 2R R
(a) (b) (c) (d) 3R R (2π − θ ) Rθ
6 3 3 (a) (b)
4π 2π
61 A potentiometer wire of length 100 cm has a  θ R
(c) R 1 −  (d) (2π − θ )θ
resistance of 10 Ω. It is connected in series with a  2π  4π 2
resistance and a cell of emf 2V having negligible
internal resistance. A source of emf 10 mV is 67 The resistance in the two arms of the meter bridge
balanced against a length of 40 cm of the are 5 Ω and R Ω, respectively. When the resistance
potentiometer wire. The value of external resistance R is shunted with an equal resistance, then the new
is [UP CPMT 2015] balance point is at 1.6 l1. The resistance R is
[CBSE AIPMT 2014]
(a) 760 Ω (b) 640 Ω (c) 790 Ω (d) 840 Ω (a) 10 Ω (b) 15 Ω (c) 20 Ω (d) 25 Ω
for More Books Click HERE -> NEETpassionate.com
232 OBJECTIVE Physics Vol. 2

67 A potentiometer circuit has been set up for finding the R. By mistake one of the cells is connected in
internal resistance of a given cell. The main battery, reverse. Then, the current in the external circuit is
used across the potentiometer wire, has an emf of [WB JEE 2014]
2E 3E 3E 2E
2.0 V and a negligible internal resistance. The (a) (b) (c) (d)
potentiometer wire itself is 4m long. When the 4r + R 4r + R 3r + R 3r + R
resistance R connected across the given cell, then 75 A circuit consists of three batteries of emf E 1 = 1 V,
has values of E 2 = 2 V and E 3 = 3 V and internal resistance
(i) infinity (ii) 9.5 Ω 1 Ω, 2 Ω and 1 Ω respectively which are connected in
The balancing lengths of the potentiometer wire are parallel as shown in figure. The potential difference
found to be 3 m and 2.85 m, respectively. between points P and Q is [WB JEE 2014]
The value of internal resistance of the cell is E1=1 V
[CBSE AIPMT 2014]
(a) 0.25 Ω (b) 0.95 Ω E2=2 V
(c) 0.5 Ω (d) 0.75 Ω
E3=3 V
69 In an ammeter, 0.2% of main current passes through P Q
the galvanometer. If resistance of galvanometer is G,
then the resistance of ammeter will be (a) 1.0 V (b) 2.0 V
[CBSE AIPMT 2014] (c) 2.2 V (d) 3.0 V
1 499 1 500 76 Two resistors of resistances 2 Ω and 6 Ω are
(a) G (b) G (c) G (d) G
499 500 500 499 connected in parallel. This combination is then
70 A carbon film resistor has colour code green, black, connected to a battery of emf 2 V and internal
violet, gold. The value of the resistor is [KCET 2014] resistance 0.5 Ω. What is the current flowing
(a) 50 MΩ (b) 500 MΩ through the battery? [KCET 2014]
(c) 500 ± 5% MΩ (d) 500 ± 10% MΩ 4
(a) 4 A (b) A
71 A uniform wire of resistance 9 Ω is joined 3
4
end-to-end to form a circle. Then, the resistance of (c) A (d) 1 A
17
the circular wire between any two diametrically
points is [Kerala CEE 2014] 77 The dimensions of mobility of charge carriers are
9 3 [Kerala CEE 2014]
(a) 6 Ω (b) 3 Ω (c) Ω (d) Ω (a) [M − 2T2A] (b) [M −1T2A]
4 2
(e) 1 Ω (c) [M − 2T3A] (d) [M −1T3A]
72 The equivalent resistance of two resistors connected (e) [M −1T3A−1]
in series is 6 Ω and their parallel equivalent 78 The temperature coefficient of resistance of an alloy
4 used for making resistor is
resistance is Ω. What are the values of [Kerala CEE 2014]
3 (a) small and positive (b) small and negative
resistances? [KCET 2014] (c) large and positive (d) large and negative
(a) 4 Ω, 6 Ω (b) 8 Ω, 1 Ω (e) zero
(c) 4 Ω, 2 Ω (d) 6 Ω, 2 Ω 79 A wire of resistance 4 Ω is stretched to twice its
73 Six resistances are connected as shown in figure. If original length. In the process of stretching, its area
total current flowing is 0.5 A, then the potential of cross-section gets halved. Now, the resistance of
differenceVA −VB is [EAMCET 2014] the wire is [EAMCET 2014]
6Ω 6Ω 6Ω (a) 8 Ω (b) 16 Ω (c) 1 Ω (d) 4 Ω

A 6Ω 80 In Wheatstone bridge, three resistors P, Q and R are


B
0.5 A connected in three arms in order and 4th arm of
resistance s, is formed by two resistors s1 and s 2
6Ω 12 Ω connected in parallel. The condition for bridge to be
(a) 8 V (b) 6 V (c) 2 V (d) 4 V P
balanced is,
Q [MHT CET 2014]
74 Four cells, each of emf E and internal resistance r,
are connected in series across an external resistance R (s1 + s2 ) s1s2 R s1s2 (s1 + s2 )
(a) (b) (c) (d)
s1s2 R (s1 + s2 ) (s1 + s2 ) R s1s2
for More Books Click HERE -> NEETpassionate.com
Current Electricity 233

81 An electron in potentiometer experiences a force 89 The internal resistance of a 2.1 V cell which gives a
2.4 × 10 −19 N. The length of potentiometer wire is current of 0.2 A through a resistance of 10 Ω is
[NEET 2013]
6m. The emf of the battery connected across the
(a) 0.2 Ω (b) 0.5 Ω (c) 0.8 Ω (d) 1.0 Ω
. × 10 −19 C)
wire is (electronic charge = 16
[MHT CET 2014] 90 The resistances of the four arms P, Q, R and S in a
(a) 6 V (b) 9 V Wheatstone’s bridge are 10 Ω, 30 Ω, 30 Ω and
(c) 12 V (d) 15 V 90 Ω, respectively. The emf and internal resistance
82 A galvanometer having internal resistance 10 Ω of the cell are 7V and 5 Ω, respectively. If the
galvanometer resistance is 50 Ω, then the current
requires 0.01 A for a full scale deflection. To
drawn from the cell will be [NEET 2013]
convert this galvanometer to a voltmeter of full scale
(a) 1.0 A (b) 0.2 A (c) 0.1 A (d) 2.0 A
deflection at 120 V, we need to connect a resistance
of [UK PMT 2014] 91 An electron revolves in a circle at the rate of 10 19
(a) 11990 Ω in series (b) 11990 Ω in parallel rounds per second. The equivalent current is
(c) 12010 Ω in series (d) 12010 Ω in parallel . × 10 −19 C )
(e = 16 [J & K CET 2013]
83 In potentiometer experiment, a cell of emf 1.25 V (a) 1.0 A (b) 1.6 A (c) 2.0 A (d) 2.6 A
gives balancing length of 30 cm. If the cell is 92. A silver wire of radius 0.1 cm carries a current of
replaced by another cell, then balancing length is 2A. If the charge density in silver is
found to be 40 cm. What is the emf of second cell? 5.86 × 10 28 m−3 , then the drift velocity is
[KCET 2014]
[J & K CET 2013]
(a) —~ 1.5 V (b) —~ 1.67 V −3 −1 −4
(c) —~ 1.47 V (d) —~ 137
. V (a) 0.2 × 10 ms (b) 0.4 × 10 ms−1

84 Potentiometer measures the potential difference (c) 0.68 × 10−4 ms−1 (d) 7 × 10−4 ms−1
more accurately than a voltmeter because 93 A 1 m long wire of diameter 0.31 mm has a resistance
[UK PMT 2014] of 4.2 Ω. If it is replaced by another wire of same
(a) it does not draw current from external circuit material of length 1.5 m and diameter 0.155 mm, then
(b) it draws a heavy current from external circuit the resistance of wire is [J & K CET 2013]
(c) it has a wire of high resistance
(a) 25.2 Ω (b) 0.6 Ω (c) 26.7 Ω (d) 0.8 Ω
(d) it has a wire of low resistance
94 24 cells of emf 1.5 V each having internal resistance
85 In a potentiometer experiment, the balancing with a
of 1 Ω are connected to an external resistance of
cell is at length 240 cm. On shunting the cell with a
1.5 Ω. To get maximum current, [J & K CET 2013]
resistance of 2 Ω, the balancing becomes 120 cm.
(a) all cells are connected in series combination
The internal resistance of the cell is [UK PMT 2014]
(b) all cells are connected in parallel combination
(a) 1 Ω (b) 0.5 Ω (c) 4 cells in each row are connected in series and 6 such
(c) 4 Ω (d) 2 Ω rows are connected in parallel
86 A galvanometer has a coil of resistance 100 Ω and (d) 6 cells in each row are connected in series and 4 such
rows are connected in parallel
gives full scale deflection for 30 mA current. If it is
to work as a voltmeter of 30 V, the resistance 95 The temperature coefficient of the resistance of a
required to be added is [UK PMT 2014] wire is 0.00125 per ° C. At 300 K its resistance is
(a) 500 Ω (b) 900 Ω 1 Ω. The resistance of wire will be 2 Ω at
(c) 1000 Ω (d) 1800 Ω [J&K CET 2013]
(a) 1154 K (b) 1100 K (c) 1400 K (d) 1127 K
87 When 4 A current flows for 2 min in an electroplating
experiment, then m gram of silver is deposited. 96 The emf of a cell E is 15 V as shown in the figure
Then, the amount (in gram) of silver deposited by with an internal resistance of 0.5 Ω. Then, the value
6 A current flowing for 40 s is [NEET 2013]
of the current drawn from the cell is [EAMCET 2013]
2Ω 7Ω
m m
(a) 4m (b) (c) 2m (d)
2 4
15 V
88 A wire of resistance 4 Ω is stretched to twice its 6Ω 1Ω
original length. The resistance of stretched wire 0.5 Ω
would be [NEET 2013]
8Ω 10 Ω
(a) 2 Ω (b) 4 Ω
(c) 8 Ω (d) 16 Ω (a) 3 A (b) 2 A (c) 5 A (d) 1 A
for More Books Click HERE -> NEETpassionate.com
234 OBJECTIVE Physics Vol. 2

97 Copper and carbon wires are connected in series and 104 The drift speed of electrons in copper wire of
the combined resistor is kept at 0° C. Assuming the diameter d and length l is v. If the potential
combined resistance does not vary with temperature, difference across the wire is doubled, then the new
the ratio of the resistances of carbon and copper drift speed becomes [Kerala CET 2013]
wires at 0° C is (temperature coefficients of (a) v (b) 2v
resistivity of copper and carbon respectively are (c) 3v (d) v/2
4 ×10 −3 /° C and −0.5 × 10 −3 / ° C) [EAMCET 2013]
(e) v/4
(a) 4 (b) 8 (c) 6 (d) 2 105 A potentiometer wire of length 10 m and resistance
98 Three conductors draw currents of 1 A, 2 A snd 3 A
10 Ω per metre is connected in series with a resistance
respectively, when connected in turn across a box and a 2 volt battery. If a potential difference of
battery. If they are connected in series and the 100 mV is balanced across the whole length of
combination is connected across the same battery, potentiometer wire, then the resistance introduced in
then the current drawn will be [Karnataka CET 2013] the resistance box will be [MP PMT (2013)]
2 3 (a) 1900 Ω (b) 900 Ω
(a) A (b) A (c) 190 Ω (d) 90 Ω
7 7
4 5 106 If a wire is stretched to four times its length, then
(c) A (d) A
7 7 the specific resistance of the wire will [MP PMT 2013]
(a) become 4 times (b) become 1/4 times
99 Masses of three wires of copper are in the ratio of
(c) become 16 times (d) remain the same
1 : 3 : 5 and their lengths are in the ratio of 5 : 3 : 1.
The ratio of their electrical resistances is 107 For the circuit shown in figure given below, the
[Karnataka CET 2013] equivalent resistance between points A and B is
(a) 1 : 3 : 5 (b) 5 : 3 : 1 [MP PMT 2013]
(c) 1 : 15 : 125 (d) 125 : 15 : 1 7Ω
D C
100 In the circuit diagram, heat produces in R, 2R and
3Ω Ω
1.5 R are in the ratio of [Karnataka CET 2013] 10 5Ω
I1 R
A B
1.5 R 10 Ω
I I
(a) 10 Ω (b) 5 Ω
10
(c) Ω (d) 2 Ω
I2
2R 3
(a) 4 : 2 : 3 (b) 8 : 4 : 27 108 Two resistors of 6 Ω and 9 Ω are connected in series
(c) 2 : 4 : 3 (d) 27 : 8 : 4 to a 120 V source. The power consumed by 6 Ω
101 Which one of the following electrical meter has the resistor is [MP PMT 2013]
smallest resistance? [Kerala CET 2013] (a) 384 W (b) 616 W
(a) Ammeter (b) Milliammeter (c) 1500 W (d) 1800 W
(c) Galvanometer (d) Voltmeter 109 A current of 2A flows in the arrangement of
(d) Millivoltmeter conductors as shown in below figure. The potential
102 Two wires of the same material having equal area of difference between points A and B (VA −VB ) will be
cross-section have length L and 2L. Their respective [UP CPMT 2013]
resistances are in the ratio [Kerala CET 2013] A
(a) 2 : 1 (b) 1 : 1 2Ω 3Ω
(c) 1 : 2 (d) 1 : 3 D C
(e) 4 : 1 2A
103 Two bulbs 60 W and 100 W designed for voltage 3Ω 2Ω
220 V are connected in series across 220 V source. B
The net power dissipated is [Kerala CET 2013]
(a) + 1 V (b) −1 V
(a) 80 W (b) 160 W (c) 37.5 W (d) 60 W
(c) +2 V (d) −2 V
(e) 120 W
for More Books Click HERE -> NEETpassionate.com
Current Electricity 235

110 A cell of emf E and internal resistance r supplies 117 Two cells when connected in series are balanced on
current for the same time t through external 8 m on a potentiometer. If the cells are connected
resistance R 1 and R 2 separately. If the heat with polarities of one of the cells is reversed, then
developed in both the cases is the same, then the they balance on 2 m. The ratio of emfs of two cells
internal resistance r will be [UP CPMT 2013] is [BCECE (Mains) 2012]
(a) r = R1 + R 2 (b) r = R1 × R 2 (a) 3 : 4 (b) 4 : 3
(c) 3 : 5 (d) 5 : 3
R1 + R 2 1 1
(c) r = (d) r = + 118 In the given circuit diagram if each resistance is of
2 R1 R 2
10 Ω, then the current in arm AD will be
111 In the circuit shown, the cells A and B have [BCECE (Mains) 2012]
E
negligible resistances. ForVA = 12 V, R 1 = 500 Ω F i
and R = 100 Ω the galvanometer (G ) shows no
deflection. The value ofVB is [CBSE AIPMT 2012]
B
R1
G C
i
A
VA R VB
D

i 2i
(a) (b)
(a) 4 V (b) 2 V (c) 12 V (d) 6 V 5 5
3i 4i
112 A millivoltmeter of 25 mV range is to be converted (c) (d)
5 5
into an ammeter of 25 A range. The value (in ohm)
of necessary shunt will be [CBSE AIPMT 2012] 119 When current i is flowing through a conductor, the
(a) 0.001 (b) 0.01 (c) 1 (d) 0.05 drift velocity is v. If the value of current through the
conductor and its area of cross-section is doubled,
113 If voltage across a bulb rated 220 V-100 W drops by
then new drift velocity will be [BCECE Mains 2012]
2.5% of its rated value, then the percentage of the
v
rated value by which the power would decrease is (a) 4v (b)
[CBSE AIPMT 2012] 2
(a) 20% (b) 2.5% (c) 5% (d) 10% v
(c) (d) v
4
114 6 Ω and 12 Ω resistors are connected in parallel.
This combination is connected in series with a 10 V 120 A wire having resistance12 Ω is bent in the form of an
battery and 6 Ω resistor. What is the potential equilateral triangle. The effective resistance between
difference between the terminals of the 12 Ω any two corners of the triangle will be
[BCECE Mains 2012]
resistor? [AIIMS 2012]
8
(a) 4 V (b) 16 V (c) 2 (d) 8 V (a) 6 Ω (b) Ω
3
115 Charge passing through a conductor of cross-section (c) 9 Ω (d) 12 Ω
area A = 0.3 m2 is given by q = 3 t 2 + 5 t + 2 in 121 When a current of (2.5 ± 0.5) A flows through a
coulomb, where t is in second. What is the value of wire, it develops a potential difference of (20 ± 1) V,
drift velocity at t = 2 s ? (Given, n = 2 × 10 25 /m3 ) then the resistance of wire is [UP CPMT 2012]
[AIIMS 2012] (a) (8 ± 2) Ω (b) (8 ± 1.6) Ω
(a) 0.77 × 10−5m / s (b) 1.77 × 10−5m / s (c) (8 ± 1.5) Ω (d) (8 ± 3) Ω
(c) 2.08 × 105m / s (d) 0.57 × 105m / s 122 To draw the maximum current from a combination
116 A galvanometer having resistance of 50 Ω requires a of cells, how should the cells be grouped?
[UP CPMT 2012]
current of 100 µA to give full scale deflection. How (a) Parallel
much resistance is required to convert it into an (b) Series
ammeter of range of 10 A? [BCECE (Mains) 2012] (c) Depends upon the relative values of internal and
(a) 5 × 10−3 Ω in series (b) 5 × 10−4 Ω in parallel external resistance
5 (d) Mixed grouping
(c) 10 Ω in series (d) 105 Ω in parallel
for More Books Click HERE -> NEETpassionate.com
236 OBJECTIVE Physics Vol. 2

123 The variation betweenV-i has shown by graph for C respectively, then R 1 : R 2 : R 3 will be equal to
heating filament [UP CPMT 2012] [AMU 2012]
A
i i

(a) (b)
r
r
r
V V r

i i r
B C
(.c) (d) r
(a) 6 : 3 : 2 (b) 1 : 2 : 3 (c) 5 : 4 : 3 (d) 4 : 3 : 2
V V 129 The equivalent resistance between A and B of
124 Two bulbs when connected in parallel to a source network shown in figure is [UP CPMT 2012]
take 60 W each. The power consumed, when they R
are connected in series with the same source is R
[UP CPMT 2012]
R R B
(a) 15 W (b) 30 W
A R R
(c) 60 W (d) 120 W
125 A voltmeter of range 2 V and resistance 300 Ω 3R 4R
(a) (b) (c) 6 R (d) 2 R
cannot be converted into ammeter of range 4 3
[Manipal 2012]
(a) 1 A (b) 1 mA 130 Each resistance shown in figure is 2 Ω. The
(c) 100 mA (d) 10 mA equivalent resistance between A and B is [AFMC 2012]
126 In the Wheatstone network given, P = 10 Ω,
Q = 20 Ω, R = 15 Ω, S = 30 Ω, the current passing 2Ω 2Ω
through the battery (of negligible internal resistance)
is [Manipal 2012]
A B
2Ω
P R
2Ω 2Ω

G
(a) 2 Ω (b) 1 Ω (c) 4 Ω (d) 5 Ω
Q S 131 In the circuit shown, the potential difference
between x and y will be [JCECE 2012]
40 Ω x y
+ –
6V
(a) 0.36 A (b) zero
(c) 0.18 A (d) 0.72 A
127 A current of 5 A is passing through a metallic wire
of cross-sectional area 4 × 10 −6 m2 . If the density of 120 V 20 Ω
charge carriers of the wire is 5 × 10 26 m−3 , the drift
velocity of the electrons will be [Manipal 2012]
(a) zero (b) 120 V (c) 60 V (d) 20 V

(a) 1 × 10 ms 2 −1
(b) 1.56 × 10 −2
ms −1 132 For the circuit shown in figure, [JCECE 2012]
−3 −1 −2 −1 R
(c) 1.56 × 10 ms (d) 1 × 10 ms
128. Six resistances each of value r = 5 Ω are connected 0.5 A
between points A, B and C as shown in the figure. If 10 Ω 10 Ω 20 Ω
R 1, R 2 and R 3 are the net resistance between A and 25 V
B, between B and C and between A and C
for More Books Click HERE -> NEETpassionate.com
Current Electricity 237

(a) resistance R = 46 Ω (a) −1 V (b) +1 V


(b) current through 20 Ω resistance is 0.1A (c) −2 V (d) + 2 V
(c) potential difference across the middle resistance is 2 V 135 Two batteries of emfs 2 V and 1 V of internal
(d) All of the above are true resistances 1 Ω and 2 Ω respectively are connected in
133 An ammeter connected in the circuit as shown in parallel. The effective emf of the combination is
figure shows a reading of [BHU Screening 2012] [Kerala CEE 2011]
2Ω 3 5 3
(a) V (b) V (c) V (d) 2 V
2 3 5
2Ω 2V (e) 5 V
A 136 Two cells with the same emf E and different internal
2Ω resistances r1 and r 2 are connected in series to an
external resistance R. What is the value of R, if the
potential difference across the first cell is zero?
2Ω
[WB JEE 2011]

1 3 1 (a) rr
12 (b) r1 + r2
(a) A (b) A (c) A (d) 2 A r +r
2 4 8 (c) r1 − r2 (d) 1 2
2
134 If a current of 2 A flows through resistances
connected as shown in figure, the potential 137 In the circuit shown below, the currents i1 and i 2 are
differenceVA − VB is [KCET 2011]
[BHU Screening 2012] i1
B
12 Ω 2Ω
i2 4Ω
2Ω 3Ω

A C
12 V, 1 Ω

3Ω 2Ω
(a) 1.5 A, 0.5 A (b) 0.5 A, 1.5 A
(c) 1 A, 3 A (d) 3 A, 1 A
D
2A 2A

ANSWERS
CHECK POINT 3.1
1. (a) 2. (a) 3. (c) 4. (b) 5. (a) 6. (b) 7. (a)

CHECK POINT 3.2


1. (b) 2. (a) 3. (a) 4. (d) 5. (c) 6. (b) 7. (c) 8. (b) 9. (b) 10. (d)
11. (a) 12. (b) 13. (d) 14. (d) 15. (a) 16. (d)

CHECK POINT 3.3


1. (c) 2. (d) 3. (b) 4. (b) 5. (a) 6. (c) 7. (a) 8. (b) 9. (a) 10. (d)
11. (b) 12. (b) 13. (c) 14. (c) 15. (b) 16. (a) 17. (c)

CHECK POINT 3.4


1. (d) 2. (d) 3. (a) 4. (b) 5. (c) 6. (d) 7. (d) 8. (d) 9. (d) 10. (c)
11. (c) 12. (b) 13. (b) 14. (b) 15. (c) 16. (c) 17. (a) 18. (c) 19. (b) 20. (a)
for More Books Click HERE -> NEETpassionate.com
238 OBJECTIVE Physics Vol. 2

(A) Taking it together


1. (c) 2. (d) 3. (c) 4. (b) 5. (a) 6. (c) 7. (b) 8. (b) 9. (b) 10. (a)
11. (c) 12. (d) 13. (b) 14. (a) 15. (c) 16. (a) 17. (d) 18. (a) 19. (a) 20. (b)
21. (a) 22. (a) 23. (b) 24. (c) 25. (a) 26. (a) 27. (b) 28. (d) 29. (d) 30. (c)
31. (d) 32. (a) 33. (b) 34. (a) 35. (d) 36. (a) 37. (d) 38. (b) 39. (c) 40. (b)
41. (d) 42. (c) 43. (c) 44. (c) 45. (b) 46. (c) 47. (b) 48. (d) 49. (c) 50. (c)
51. (a) 52. (b) 53. (c) 54. (c) 55. (a) 56. (d) 57. (b) 58. (c) 59. (b) 60. (b)
61. (c) 62. (d) 63. (d) 64. (c) 65. (d) 66. (c) 67. (b) 68. (b) 69. (d) 70. (d)
71. (d) 72. (a) 73. (b) 74. (c) 75. (b) 76. (d) 77. (b) 78. (d) 79. (b) 80. (a)
81. (c) 82. (b) 83. (b) 84. (a) 85. (d) 86. (d) 87. (d) 88. (c) 89. (b) 90. (d)
91. (d) 92. (a) 93. (a) 94. (b) 95. (a) 96. (c) 97. (a) 98. (d) 99. (b) 100. (d)
101. (b) 102. (c) 103. (a) 104. (a) 105. (b) 106. (c) 107. (a) 108. (b) 109. (b) 110. (d)
111. (a) 112. (c) 113. (d) 114. (c) 115. (c) 116. (b) 117. (b) 118. (c) 119. (b) 120. (b)
121. (d) 122. (b) 123. (c) 124. (b)

(B) Medical entrance special format questions


l Assertion and reason
1. (d) 2. (a) 3. (b) 4. (d) 5. (a)

l Statement based questions


1. (a) 2. (d) 3. (d) 4. (c) 5. (b)

l Match the columns


1. (a) 2. (c)

(C) Medical entrances’ gallery


1. (a) 2. (c) 3. (a) 4. (b) 5. (b) 6. (b) 7. (b) 8. (b) 9. (a) 10. (c)
11. (b) 12. (d) 13. (d) 14. (a) 15. (c) 16. (b) 17. (d) 18. (b) 19. (a) 20. (a)
21. (b) 22. (c) 23. (c) 24. (a) 25. (c) 26. (b) 27. (a) 28. (c) 29. (c) 30. (b)
31. (c) 32. (b) 33. (b) 34. (a) 35. (a) 36. (c) 37. (d) 38. (c) 39. (d) 40. (d)
41. (c) 42. (a) 43. (a) 44. (b) 45. (c) 46. (a) 47. (a) 48. (b) 49. (a) 50. (e)
51. (b) 52. (c) 53. (a) 54. (a) 55. (a) 56. (b) 57. (c) 58. (a) 59. (c) 60. (c)
61. (c) 62. (d) 63. (a) 64. (a) 65. (b) 66. (d) 67. (b) 68. (c) 69. (a) 70. (c)
71. (c) 72. (c) 73. (d) 74. (a) 75. (b) 76. (d) 77. (b) 78. (a) 79. (b) 80. (a)
81. (b) 82. (b) 83. (b) 84. (a) 85. (d) 86. (b) 87. (b) 88. (d) 89. (b) 90. (b)
91. (b) 92. (c) 93. (a) 94. (d) 95. (d) 96. (d) 97. (b) 98. (*) 99. (d) 100. (b)
101. (a) 102. (c) 103. (c) 104. (b) 105. (c) 106. (d) 107. (b) 108. (a) 109. (a) 110. (b)
111. (b) 112. (a) 113. (c) 114. (a) 115. (b) 116. (b) 117. (d) 118. (b) 119. (d) 120. (b)
121. (a) 122. (c) 123. (a) 124. (b) 125. (b) 126. (a) 127. (b) 128. (c) 129. (a) 130. (a)
131. (b) 132. (d) 133. (b) 134. (d) 135. (b) 136. (c) 137. (b)
for More Books Click HERE -> NEETpassionate.com

Hints & Explanations


l CHECK POINT 3.1 3 (a) R = ρ
l
dq A
1 (a) Current, I = = 3t 2 + 2t + 5
dt Since, wires are made of same material, so resistivity ρ of all
t=2 wires will be same.
q =∫ (3t 2 + 2t + 5) dt l
t=0
Therefore, R ∝
Charge, q = [t 3 + t 2 + 5t]20 = [8 + 4 + 10] = 22 C A
l
q ne n × 1.6 × 10 −19 ⇒ R∝ 2
2 (a) Current, i =or i = ⇒ 16 × 10 −3 = πr
t t 1 l
R∝
∴ Number of electrons, n = 1017 d 
2
 
ne  2
3 (c) We have, current, i =
t For length = 50 cm and diameter = 0.5 mm
it 0.2 × 30 Resistance will be maximum.
∴ Number of electrons, n = = = 3.75 × 1019
e 1.6 × 10 −19
4 (d) Colour code for carbon resistor,
4 (b) Given, I = 4 − 0.08 t A B C D
dq Brown Red Green Gold
⇒ = 4 − 0.08 t
dt ↓ ↓ ↓ ↓

50
q = ∫ (4 − 0.08t ) dt 1 2 10 5 5%
0 R = AB × C ± D
50
 0.08t 2  = 12 × 10 5 ± 5%
⇒ ne = 4t − = 100
 2  0 5 (c) Given, circuit can be redrawn as follows
100 100
⇒ n= = = 6.25 × 10 20 electrons 5W 5W B 5W
e 1.6 × 10 −19 A C
2/3 V 2/3 V 2/3 V
eEτ
5 (a) Drift velocity, v d = I1 V1 V2 V3
m
I
∴ vd ∝ E I1 2V

6 (b) The order of drift velocity of electrons is A C


5W D 5W 5W
10 −4 ms −1 = 10 −2 cms −1
7 (a) Drift velocity, I V 2 2
Here, I1 = = = = A
3 2 2Req  15 15
vd =
i
= 2× 
 2
neA 8.5 × 10 28 × 1.6 × 10 −19 × 2 × 10 −6
So, potential across each resistance,
= 0.11 × 10 −3 ms −1
2 2
length of the wire 3 V ′ = I1 R = ×5= V
∴Time, t = = = 2.73 × 10 4 s 15 3
vd 0.11 × 10 −3
∴ Potential difference across,
2 2 4
l CHECK POINT 3.2 AB = + = V
V q 3 3 3
1 (b) We have, i = =
R t 6 (b) Potential difference across the circuit
Vt 20 × 2 × 60  R R   6 × 4
∴ Charge, q = = = 240 C = i  1 2  = 1.2 ×   = 2.88 V
R 10  R1 + R2   6 + 4
l m
2 (a) Resistance, R = ρ and resistivity, ρ = 2 So, current through 6 Ω resistance
A ne τ
ml 2.88
∴ R= 2 = = 0.48A
ne τA 6
for More Books Click HERE -> NEETpassionate.com
240 OBJECTIVE Physics Vol. 2

Alternative solution So, equivalent resistance across battery,


 R2  Req = 8.5 + 2 + 4.5 = 15 Ω
From current division rule, i1 = i  
 R1 + R2  Hence, current from the battery,
15
i1 R1 = 6 W i= = 1A
15
12 (b) As B is connected to the earth, so potential at B isVB = 0.
i i Now, current in the given circuit,
R2 = 4 W V 50
i= = = 2A
i2 Rnet 5 + 7 + 10 + 3
Potential difference between A and B is
 4   4
= 1.2   = 1.2   = 0.48 A VA − VB = 2 × 12
 6 + 4  10 
or VA − 0 = 24
7 (c) Current divides according to resistance, so current in 6 Ω VA = 24 V
0.8
resistance is = 0.4 A 13 (d) We have, V = E − ir
2
= 1.5 − 2 × 0.15
So, total current in circuit is 0.8 + 0.4 = 1.2 A
∴ Potential drop across 4 Ω = 1.2 × 4 = 4.8 V V = 1.20 V

8 (b) Two resistances are short circuited. 14 (d) Potential difference between A and B is given by
E r + E 2 r1
So, only third resistance will be considered and hence, VA − VB = 1 2
r1 + r2
i = V /R = 10 / 3 A
5X + 2 × 10
10 × 15 ∴ 4=
9 (b) Net resistance, Rnet = + 3 + 0.5 = 9.5 Ω X + 10
10 + 15
⇒ X = 20 Ω
V 20
∴ I= = = 2.1A
Rnet 9.5 15 (a) 1Ω E1 E2 2Ω
P i Q
According to current division rule, current through 10 Ω 4V 8V
resistance is
i
 15 
I1 =   × 2.1 = 1.2 A
 10 + 15
9Ω
R 2
10 (d) Resistance, RAB = +R= +2
3 3 E1 = 4 V and E 2 = 8 V
8 2 As, E 2 > E1, so current flows from Q to P.
RAB = =2 Ω
3 3 8−4 1
∴ i= = A
11 (a) The circuit can be reduced as follows 12 3
2Ω
1
∴ Potential difference across PQ = × 9 = 3 V
3
15 V Parallel
16 (d) The potentials of different points are as shown in below
figure
6Ω 18 Ω
0.5 Ω 2V 5Ω 4V

8Ω
Series 8V 4V
6V
0V
2Ω
10 V 9Ω

Current through 5 Ω resistance


15 V
Potential difference
=
4.5 Ω Resistance
8.5 Ω
(4 − 2)
= = 0.4 A
5
for More Books Click HERE -> NEETpassionate.com
Current Electricity 241

l CHECK POINT 3.3 V 2 25 × 25


6 (c) Resistance, R = = = 25 Ω
P 25
1 (c)
3A
15 A V2
7 (a) Resistance, R = or R ∝ V 2
A B P
2
R1 V12 R  220  R
8A ⇒ = ⇒ =  = 4 ⇒ R2 =
R2 V22 R2  110  4
D C V2
i 8 (b) Power, P =
5A R
1
∴ P ∝
Applying Kirchhoff’s first law at junction A, B , C, D R
At A, iAB = 15 + 8 = 23A P1 R2
So, =
At B, iBC = 23 + 3 = 26 A P2 R1
At D, iCD = 8 − 5 = 3A 200 R2
At C, iCD + i = iBC =
100 R1
or 3 + i = 26
R2 = 2R1
∴ i = 23 A
V2
2 (d) 1 A 1.5 Ω VB = 0 2.5 Ω 2V 9 (a) Power, P =
R
A B C D 1
Potential difference between A and B, ∴ P ∝
R
VA − VB = 1 × 1.5 or VA − 0 = 1.5 (QVB = 0, given) 1
∴ VA = 1.5 V Also, resistance of wire ∝
(radius of filament)2
Now, potential difference between B and C,
∴ P ∝ (radius of filament)2
VB − VC = 1 × 2.5 = 2.5 V
So, 100 W bulb has thicker filament.
∴ 0 − VC = 2.5 V
⇒ VC = − 2.5 V V 2t Pt 210 × 5 × 60
10 (d) H = = = = 15000 cal
Hence, potential difference between C and D, R×J J 4.2
VC − VD = − 2 V or − 2.5 − VD = − 2 or VD = − 0.5 V V2 (15)2
11 (b) P = ⇒ 150 =
3 (b) Let at junction C, potential isV. R  2R 
 
 2 + R
20 V 2W C 4W 5V
2+R 2
A i1 i3 i2 B ∴ =
2R 3
2W ∴ Resistance, R = 6 Ω
R V2
12 (b) R′ = and P =
4 R
0V P2
∴ P2 = 4P1 ⇒ =4
P1
VA − VC VB − VC VC − 0
∴ + =
2 4 2 13 (c) R40 > R100 . In series, potential difference distributes in
20 − V 5 − V V − 0 direct ratio of resistance.
+ =
2 4 2 V2
14 (c) When bulbs are in series, P = …(i)
V = 9V 3R
9 When bulbs are connected in parallel,
Current, i3 = i = = 4.5 A
2 V2 3V 2
P′ = = = 3 × 3P [from Eq. (i)]
4 (b) Work done,W = qV = 6 × 10 −6 × 9 = 54 × 10 −6 J (R / 3) R

5 (a) In series, i is same, so H = i 2Rt. Therefore, H ∝ R. = 9P

H1 R 1 15 (b) It is known that in parallel combination,


∴ = = 1
H 2 2R 2 Pconsumed ∝ Brigntness ∝
R
⇒ H1 : H 2 = 1: 2
According to question, PA > PB (given), therefore RB > RA
for More Books Click HERE -> NEETpassionate.com
242 OBJECTIVE Physics Vol. 2

V2 11 (c) Before connecting the voltmeter, let the potential


16 (a) P = . As, R net will decrease, so P will increase.
R difference across 100 Ω isV1.
17 (c) When each bulb is glowing at full power. 10 Ω 100 Ω
50 1
Current from each bulb, i = = A V1
100 2
n
So, main current, i = A (for parallel circuit)
2
Also, E = V + ir V
n
120 = 100 +   × 10  100  10
 2 ∴ V1 =   ×V = V
 100 + 10  11
n=4
After connecting the voltmeter across 100 Ω,
l CHECK POINT 3.4
900 Ω
1 (d) Ammeter is parallel with voltmeter, therefore its reading
will be zero.
1.6 10 Ω 100 Ω
4 (b) If voltmeter is ideal, then R should be = 4 Ω. If it is
0.4
V2
non-ideal, R should be greater than 4 Ω.
5 (c) Total resistance of given circuit V
80
= + 20 = 40 + 20 = 60 Ω
1+ 1
100 × 900
2 1 Equivalent resistance = = 90 Ω
∴ Main current, i = = A 100 + 900
60 30
Let this time potential difference isV2 .
Now, in parallel, there are two resistances of 80 Ω each  90  9
∴ V2 =  V = V
(one of voltmeter and other 80 Ω resistance). So, current is  90 + 10  10
1
equally distributed in 80 Ω resistance and voltmeter, i.e. A 10 9
60 V− V
of current flows through each. Magnitude of % error = 11 10 × 100 = 1.0
10
∴ Potential difference across 80 Ω resistance or voltmeter V
1 11
reading = × 80 = 1.33 V
60 12 (b) (i) 50 ≠ (50 × 10 −6 ) (100 + 10 4 ). Therefore, (a) is wrong.
6 (d) Equivalent resistance of circuit, (ii) 10 ≈ (50 × 10 −6 ) (100 + 200 × 10 3 ). Therefore, (b) is
1000 2500 correct.
Req = 500 + = Ω
3 3 50 × 10 −6 1
∴Current drawn from the cell, (iii) ≠ . Therefore, (c) is wrong.
10 × 10 −3 − 50 × 10 −6 100
10 3
i= = A
2500 / 3 250 14 (b) (ig RV ) (100 ) = ig (RV + 1980 )
3 1000 ∴ Resistance, RV = 20 Ω
Reading of voltmeter = × = 4V
250 3 R R
15 (c) 3 = 1 (balanced Wheatstone bridge)
7 (d) V = IR R4 R2
100 = (10 × 10 −3 ) (25 + R ) or R1 R4 = R2 R3
∴ Resistance, R = 9975 Ω 16 (c) The galvanometer shows no current, it means this is a
balanced Wheatstone bridge, so
8 (d) When some resistance is connected in parallel with R AC 20
voltmeter the effective resistance get decreased. So, A will = =
increase andV will decrease. 80 BC 80
∴ Resistance, R = 20 Ω
10 (c) We have, igG = (i − ig )S
17 (a) In potentiometer, the ratio of emfs is equal to the ratio of
10 × 99 = (90 )S no deflection lengths.
10 × 99 E1 l1 2
S= = =
90 E 2 l2 3
∴ Shunt, S = 11Ω
for More Books Click HERE -> NEETpassionate.com
Current Electricity 243

(l1 − l2 )  60 − 50  11 (c) Out of n cells, two cells will cancel out each other’s emf.
18 (c) r = ×R =   × 6 = 1.2 Ω
l2  50  So, net emf = (n − 2) E.
19 (b) For potentiometer, emf Total resistance = R + nr
E ∝ l or E = kl (n − 2)E
Current, i =
where, k is a constant. nr + R
V iR E
Also, E = = ×l 12 (d) Current, i =
l L r +R
E′ R E 2R
∴ E = × ×l Power, P = i 2R ⇒ P =
(R1 + R2 + r ) L (r + R )2
10 5 Power will be maximum, when r = R.
∴ E = × × 3 = 3V
5+ 4+1 5 E2
Pmax =
e R 4r
20 (a) Potential gradient, x = ⋅
(R + Rh + r ) L 13 (b) Slope of the V-i curve at any point is equal to reciprocal of
−3 resistance at that point.
0.2 × 10 2 R
⇒ = ×
10 −2 (R + 490 + 0 ) 1 From the curve, slope for T1 > slope for T2
⇒ R = 4.9 Ω RT1 < RT 2
⇒ Also at higher temperature resistance will be higher, so
(A) Taking it together T2 > T1
14 (a) The relationship between current and drift speed is given
1 (c) During the charging of battery terminal, potential difference by i = neAv d
is always greater than emf of circuit.
Here, I is the current and v d is the drift velocity.
V = E + ir
So, I ∝vd
2 (d) Potentiometer works on null deflection method. In balance Thus, only drift velocity determines the current in a conductor.
condition, no current flow in secondary circuit.
15 (c) Current in the circuit will increase because another
3 (c) We cannot increase the rating of fuse wire of lower value resistance is connected in parallel to the circuit and hence
just by increasing its length. For it, we shall have to make potential drop across the ammeter will decrease. So, the
fuse wire thick. potential difference over voltmeter will increase because total
4 (b) Specific resistance of silver, copper and aluminium are potential difference over ammeter and voltmeter is equal to
1.6 × 10 −8 Ω - m, 1.7 × 10 −8 Ω - m and 2.7 × 10 −8 Ω - m, emf (constant).
respectively. 16 (a) Resistance of wire, R = ρ L /A
1
Since, conductivity (σ ) = On stretching the wire, the volume of the wire remains same.
resistivity (ρ )
A′ l
⇒ =
Hence, σ Al < σ Cu < σ Ag A l′
Thus, correct sequence is Al, Cu, Ag. So, new resistance, R′ = ρl′ /A′
2 2
5 (a) Voltmeter has high resistance. So, most of the main R′  l′   A  l′ l′  l′   1.1l 
⇒ =    = ⋅ =  =  = 1.21
current will flow through ammeter which is in parallel. So, it R  l   A′  l l  l   l 
will burn out. No damage will occur to voltmeter.
∴ R′ = 1.21R = 1.21 × 10 = 12.1Ω −~ 12 Ω
 E   R  E
6 (c) V = E − ir = E −   r =E   = 17 (d) Circuit can be reduced as follows
R + r  R + r  1 + r /R
2Ω 3Ω
8 (b) The minimum resistance can be achieved when we
5Ω
connect all resistances in parallel.
r
So, equivalent resistance of combination = . ⇒
10
9 (b) When wire is bent in the form of a circle, then between 20 V
two points in any diameter, it is equivalent to two resistances 20 V Step (2)
12 Step (1)
in parallel, Req = = 6Ω
2 So, equivalent resistance of circuit, Req = 5 Ω
20
10 (a) As steady current is flowing through the conductor, hence ∴ Current in the circuit = = 4A
the number of electrons entering from one end and outgoing 5
from the other end of any segment is equal. Hence, charge As in parallel, current is divided according to resistance, so
will be zero. current flowing through each resistance = 2 A.
for More Books Click HERE -> NEETpassionate.com
244 OBJECTIVE Physics Vol. 2

18 (a) Potential difference = 10 V R1 R2 6


24 (c) Given, = …(i)
R1 + R2 8
10
So, i = = 2A When one resistance say R2 is broken, then
5
E   1.5  R1 = 2 Ω …(ii)
19 (a) Internal resistance, r = R  − 1 = (10)  − 1 = 2 Ω From Eqs. (i) and (ii), we get
V   1.25 
6
20 (b) For twisted wire, there are two halves each of resistance R2 = Ω
5
2 Ω in parallel.
l
2×2 25 (a) We have, R =ρ
So, Req = = 1Ω A
2+ 2
R ∝l
21 (a) When wire is divided in 10 equal parts, then each part
R1 l1 1
will have a resistance = R / 10 = r . = =
Let equivalent resistance be rR , then R2 l2 2
1 1 1 1 ⇒ R2 = 2R1 …(i)
= + + + ...10 times
rR r r r
1 10 10 100 Q R1 + R2 = 9 …(ii)
∴ = = = From Eqs. (i) and (ii), we get
rR r (R /10 ) R
R1 + 2R1 = 9
R
∴ rR = = 0.01R ⇒ R1 = 3 Ω
100
∴ R2 = 2R1 = 2 × 3 = 6 Ω
22 (a) Equivalent resistance between B and C. RR 3×6
Net resistance, R net = 1 2 = =2Ω
(2 + 2) × 2 8 4 R1 + R2 3 + 6
RBC = = = Ω
2+ 2+ 2 6 3
26 (a) The simplified circuit is as shown below
A 3Ω
2W 2W
8Ω 4Ω

2W 2W Þ C A B
B
2Ω 6Ω
6Ω
B C 2W 3W 3W
2W
23 (b) Given circuit can be reduced to
16/10 W 24/10 W 4W
6Ω A B ÞA B

6W 6W

1 1 1 1 4+ 3+ 2 9
6Ω ⇒ = + + = =
A C R eq 3 4 6 12 12

3Ω 4
R eq = Ω
3Ω 3
B
27 (b) Let resistance, R = 100 Ω
So, equivalent resistance between points A and B is equal to 10
∴ R′ = 100 + 100 × = 110 Ω
100
3Ω
A C ∴ Required shunt, ∆R = R′ − R = 110 − 99 = 11 Ω
28 (d) Given,V = 115 V
3Ω and P = 1250 W
V2
3Ω We know, R =
B P
(115)2
6×3 So, R= = 10.58 Ω
Req = = 2Ω 1250
6+ 3
Resistance, R = 10.6 Ω
for More Books Click HERE -> NEETpassionate.com
Current Electricity 245

29 (d) Resistance, Rt = R0 (1 + αt ) E2 2
⇒ =
E1 3
20 = R0 (1 + 20 α ) and 60 = R0 (1 + 500 α )
E1 3
From these equations, we can find ⇒ =
E2 2
R0 = 18.33 Ω
α = 4.54 × 10 −3 °C −1 36 (a) From Ohm’s law,
Rt = 25 Ω
2V
Again, Rt = R0 (1 + αt )
25 = 18.33(1 + 4.54 × 10 −3t ) 2Ω

We find, t = 80 ° C
30 (c) The current i crossing area of cross-section A, can be
expressed in terms of drift velocity v d and the moving charges
R = 998 Ω
as
i = nev dA E 2
i= = = 2 × 10 −3A
where, n is number of charge carriers per unit volume and R + r 998 + 2
e the charge on the carrier.
Potential difference across the voltmeter is
i 24 × 10 −3
∴ vd = = V = iR = (2 × 10 −3 ) × 998 = 1.996 V −
~ 1.99 V
neA (3 × 10 23 )(1.6 × 10 −19 )(10 −4 )
V 2
= 5 × 10 −3 ms −1 37 (d) Resistance, R = = = 0.5 Ω
i 4
31 (d) Drift velocity in a copper conductor, ρl ρ × 0.5
i 5.4 Resistance, R = ⇒ 0.5 =
vd = = A 1 × 10 −6
neA 8.4 × 10 28 × 1.6 × 10 −19 × 10 −6
Resistivity of wire,
= 0.4 × 10 −3 ms −1 = 0.4 mms −1
0.5 × 10 −6
l ρ= = 1 × 10 −6 Ω -m
32 (a) The resistance of wire is given by, R = ρ 0.5
A
For greater value of R, l must be higher and A should be lower 38 (b) In order to balance the bridge, the effective value of S
and it is possible only when the battery is connected across must be equal to 2 Ω.
S×6
 1 So, 2=
1cm ×   cm (area of cross-section A). S+6
 2
2S + 12 = 6S ⇒ 4S = 12 ⇒ S = 3 Ω
33 (b) In potentiometer experiment, the emf of a cell can be 2
measured, if the potential drop along the potentiometer wire 39 (c) Current in the potentiometer wire = = 2 × 10 −3 A
990 + 10
is more than the emf of the cell to be determined. Here value
of emfs of two cells are given as 5V and 10V, therefore the Potential drop over wire = 2 × 10 −3 × 10 = 2 × 10 −2V (QV = IR )
potential drop along the potentiometer wire must be more
2 × 10 −2  V
than 10 V. Potential gradient = = 0.01 V/m Q k = 
2  l
V iR i ρ L i ρ
34 (a) Potential gradient = = = = −19
L L LA A  v  1.6 × 10 (2.2 × 10 6 )
40 (b) Current, i = qf = q   =
0.2 × 40 × 10 −8  2πr  (2π ) (5 × 10 −11)
= = 10 −2 V/m
8 × 10 −6 = 1.12 × 10 −3A
35 (d) When potentiometer is connected between A and B, then 41 (d) We have, k × l = i × R
it measures only E1 and when connected between A and C,
(2 × 10 −3 ) (50 ) = 10 × i
then it measures E1 − E 2.
E1 l The current passing through the resistor,
∴ = 1 i = 10 × 10 −3 A = 10 mA
E1 − E 2 l2
E1 − E 2 l2 42 (c) Total cells = m × n = 24 ⇒ mn = 24 ...(i)
⇒ =
E1 l1 For maximum current in the circuit,
E 2 100 mr m
⇒ 1− = R= ⇒ 3 = × (0.5) ⇒ m = 6n ...(ii)
E1 300 n n
E2 1 On solving Eqs. (i) and (ii), we get m = 12 and n = 2
⇒ = 1−
E1 3
for More Books Click HERE -> NEETpassionate.com
246 OBJECTIVE Physics Vol. 2

VV 5 10 (3 + R )
43 (c) Current, IV = = = 0.25 × 10 −3A 50 (c) Resistance, R = 3 + 10 || (3 + R ) = 3 +
RV 20 × 10 3 13 + R
∴ V = IV (R + r ) Solving this equation, we get
110 = 0.25 × 10 −3 (R + 20 × 10 3 ) R = 69 Ω
R = 420 k Ω l
51 (a) Resistance, R = ρ ⋅
E E A
44 (c) We have, i1 = and i2 = R2 A1 3
r + R1 r + R2 For same material and same length, = =
R1 A2 1
From these two equations,we get
i R − iR ∴ R2 = 3R1
r= 2 2 11 Resistance of thick wire, R1 = 10 Ω (given)
i1 − i2
∴ Resistance of thin wire, R2 = 3 × 10 = 30 Ω
1 1 1 1 3
45 (b) = + + = Total resistance of series combination = 10 + 30 = 40 Ω
RP R R R R
R 52 (b) Electric power, P = i 2R
RP = Ω
3 P
∴ Current, i =
and RS = R + R = 2R Ω R
Rnet = RP + RS For resistance of 9 Ω,
R 7R 36
Rnet = 2R + = Ω i1 = = 4= 2 A
3 3 9
2+ 2 4 i ×R 2× 9
46 (c) i = = A i2 = 1 = = 3A
1 + 1.9 + 0.9 3.8 6 6
i = i1 + i2 = 2 + 3 = 5A
For cell A, E = V + ir
⇒ V2 = iR2 = 5 × 2 = 10 V
4
V = E − ir = 2 − × 1.9 53 (c) The equivalent circuit can be drawn as
3.8
6W
V = 0 (zero)
47 (b) Internal resistance, 2V A
4W
l   50 
r =  1 − 1 R =  − 1 × 2 = 0.50 Ω
 l2   40 
3W
e R
48 (d) E = ⋅ ×l
(R + Rh + r ) L
2W
2 10
= × × 0.4 = 0.16V
(10 + 40 + 0 ) 1 ß
4W 2V A
49 (c)
1Ω 1Ω 1Ω R 1Ω 1Ω 1W
P
A Q S B
ß 2V A

5W
In circuit, resistance between PQ, QR and RS are in parallel.
Now circuit reduces to 2
Current = = 0.4 A
5
1Ω
54 (c) Total potential drop across the given wire
1Ω 1Ω 1Ω = (1 × 10 −3 ) (10 2 ) = 0.1 V
A B Therefore, potential difference across R should be 1.9 V.
1Ω
2V
R
1Ω 1/3 Ω 1Ω
A B
100 cm, 3 W
1 7 1.9 R
∴ RAB = 1+ 1+ = Ω Now, = or R = 57 Ω
3 3 0.1 3
for More Books Click HERE -> NEETpassionate.com
Current Electricity 247

55 (a) We have, rA = 2rB Also since, cells are connected in opposite polarities, the
resultant emf is
∴ AA = 4 AB
l E = E1 − E 2 = 5 V − 2 V = 3 V
or lA = B E 3
4 From Ohm’s law, E = iR ⇒ i = = = 0.1 A
R 30
∴ RB = 16RA
 16 R 59 (b) Here, the resistances of 400 Ω and 10000 Ω are in
Rnet =   RA = B
 17 17 parallel, so their effective resistance RP will be
If Rnet = 4, then RA = 4.25 Ω or RB = 68 Ω. 400 × 10000 5000
RP = = Ω
400 + 10000 13
56 (d) Balanced condition for Wheatstone’s bridge,
R1 R3 Total resistance of the circuit,
=
R2 R4 5000 15400
R= + 800 = Ω
24 + X 10 1 13 13
We have, = = …(i) 6 39
84 + Y 30 3 Current in the circuit, i = = A
3 24 + 84 + X + Y 15400 / 13 7700
and = [QV = IR]
1 10 + 30 Potential difference across voltmeter,
108 + X + Y 39 5000
= …(ii) V = iRP = × = 1.95 V
40 7700 13
Solving Eqs. (i) and (ii), we get E E
60 (b) Current, i = . When R decreases to 0, i = and
X =Y = 6 Ω R+r r
V = IR = 0.
57 (b) The given circuit can be redrawn as
P l 20 20 1
10 Ω 20 Ω 61 (c) Initially, = = = =
Q 100 − l 100 − 20 80 4

10 Ω When a resistance of 15 Ω is connected in series with the


smaller of the two, i.e. with P, then
P + 15 40 P 15 2
i 5Ω 10 Ω = ⇒ + =
Q 60 Q Q 3
+ −
1 15 2 15 5
5V + = ⇒ =
4 Q 3 Q 12
It is a balanced Wheatstone bridge and hence, no current
flows in the middle resistor, so equivalent circuit would be as ⇒ Q = 36 Ω
shown in figure. 1
Resistance, P = × 36 = 9 Ω
10 W 20 W 4
62 (d) Effective voltage on charging
= 4 × 1.4 − 2 = 3 .6 V
Total resistance = 8 Ω
5W 10 W ⇒ i = V /R = 3.6 / 8 = 0.45 A
i
63 (d) Since, on balancing no current flows through the battery
so its internal resistance will not affected.
+ -
E
5V Potential gradient =
100
10 Ω and 20 Ω resistances are in series, E 30E
So, balancing emf = × 30 =
∴ R′ = 10 Ω + 20 Ω = 30 Ω 100 100
Similarly, 5 Ω and 10 Ω are in series, R′ ′ = 15 Ω
64 (c) When a resistance of 100 Ω is connected in series, then
15 × 30
Now, R′ and R′ ′ are in parallel, R = = 10 Ω the current flowing will be
15 + 30 V
V 5 i= …(i)
So, i= = = 0.5 A 100 + R
R 10
When a resistance of 1000 Ω is connected in series, the range
58 (c) We know that, when current flow is same then resistors of galvanometer gets doubled.
are connected in series, hence resultant resistance is 2V
Current, i= …(ii)
R′ = R1 + R2 = 10 Ω + 20 Ω = 30 Ω 1100 + R
for More Books Click HERE -> NEETpassionate.com
248 OBJECTIVE Physics Vol. 2

From Eqs. (i) and (ii), we get 2


∴ R× × 10 −3 = 3000 × 1 × 10 −3
V 2V 3
= ⇒ R = 900 Ω
100 + R 1100 + R 3000 × 3
⇒ R= = 4500 Ω
i i×4 2
65 (d) Drift velocity, v d = =
nAe nπD 2e So, resistance to be added = 4500 − 50 = 4450 Ω
1 G
i.e. vd ∝ 2 73 (b) We have, shunt, S = ...(i)
D (n − 1)
v d1 D22  1
2
1 where, G is resistance of galvanometer.
∴ = =  = G
vd2 D12  2 4 Again, S′ = ...(ii)
(n′ − 1)
2
V S (n′ − 1)
66 (c) Power, P = ∴ =
R S′ (n − 1)
V2
∴ R= or R ∝ V 2 S (n − 1)
P ⇒ = (n′ − 1)
2
S′
R1  200  4
i.e. = = S (n − 1) + S ′
 
R2  300  9 ⇒ n′ =
S′
When connected in series, then potential drop and power
consumed are in the ratio of their resistances. So, 74 (c) Higher the power, lower is the resistance and lower the
P1 V1 R1 4 resistance, thicker will be the element.
= = = R 1
P2 V2 R2 9 75 (b) Given, 1 =
R2 2
67 (b) Current required by each bulb,
Let, third resistance is R.
P 100
i= = A 1 1 1
V 220 So, + + =1
a 2a R
If n bulbs are joined in parallel, then ni = ifuse 3R
100 ∴ a=
or n× = 10 or n = 22 2 (R − 1)
220
As resistance is not fractional. (given)
68 (b) Rise in temperature = 15°C R
Amount of water = 1L = 1000 cc = 1000 g ∴ =2
R −1
Heat supplied in cal = 1000 × 15 × 1
So, R = 2Ω
= 1000 × 15 × 4.2 (in form of joule)
∴ R1 = a = 3 Ω and R2 = 2a = 6 Ω
1000 × 15 × 4.2
∴Power = = 1050 W or (J/s) 76 (d) Applying Kirchhoff’s law at point D, we get
60
i1 = i2 + i3
69 (d) Average energy = 2 eV
VA − VD VD − 0 VD − VC
⇒ eV0 = 2eV0 = +
10 20 30
⇒ V0 = 2 volt VD VD − 10
or 70 − VD = +
V0 2 2 3
∴Electric field, E = = = 5 × 10 7 V/m
Mean free path 4 × 10 −8 70 − 40
⇒ VD = 40 V ⇒ i1 = = 3A
10
70 (d) By joining 2 Ω and 6 Ω in parallel, we get 1.5 Ω and
40 − 0
joining them in series, we get 8 Ω. i2 = = 2A
20
So, values of R1 and R2 will be 2 Ω and 6 Ω. A
71 (d) 10 × i = 2 × 50 70 V i1
100
i= = 10 mA 10 W
10
72 (a) Initially, R1 = 50 + 2950 = 3000 Ω i3 i2
3 D
30 W 20 W
E = 3V ⇒ I1 = = 1 × 10 −3 A
3000
C B
To reduce deflection to 20, 10 V 0V
2 40 − 10
I2 = × 1 × 10 −3 A and i3 = = 1A
3 30
for More Books Click HERE -> NEETpassionate.com
Current Electricity 249

E1/r1 − E 2 /r2 85 (d) The given circuit can be reduced to


77 (b) Enet =
1/ r1 + 1/r2
3Ω
Current through R will be zero, if Enet = 0
E1 E 2
∴ =
r1 r2 3Ω 3Ω
Net emf 10 − 4
78 (d) i = = = 1A from a to e.
Net resistance 1 + 2 + 3 A B
6Ω
79 (b) Electric field produced by charges accumulated on the
surface of wire, provides force to the electrons to change the So, equivalent resistance between A and B,
direction of current density. RAB = (3 + 3 + 3) || 6 = 9 || 6
80 (a) We know that in the given circuit, 9 × 6 9 × 6 18
= = = Ω
ε r + ε 2r1 9+ 6 15 5
ε eq = 1 2
r1 + r2 ⇒ RAB = 3.6 Ω
Here, ε eq is weighted average of ε1 and ε 2. 86 (d) The network can be redrawn as follows
Naturally, its value will lie between the value of ε1 and ε 2, 6Ω 6Ω 6Ω
A B
i.e. ε eq will be less than ε 2 and more than ε1.
81 (c) We know that percentage of error can be minimised by
adjusting the balance point near the middle of the meter scale,
6Ω 6Ω 6Ω
i.e. around 50 cm. This can be achieved by adjusting the Step (1)
value of S. For balance point to be at 2.9 cm, we have the
equation, 3Ω 3Ω 3Ω
R 2.9
= A
Step (2)
B
S 971 .
2 .9 Now, there are three resistances in series.
⇒ R= × 100 = 3 Ω
971. So, equivalent resistance,
For balance point to be in the middle, the equation becomes Req = 3 + 3 + 3 = 9 Ω
R 1 87 (d) The circuit can be reduced as,
=
S 1
2Ω 2Ω 2Ω
R=S A
Here, R = 3Ω
⇒ S = 3Ω 8Ω 8Ω 4Ω
So, the value of S should be changed from 100 Ω to 3 Ω to
obtain reading in the mean position.
B
2Ω 2Ω 2Ω
82 (b) No current will flow through the grounded wire. Step (1)
V
∴ I= 2Ω 2Ω
2R A
83 (b) We have, IG = 0
∴ VX = E 2 = 2 V 8Ω 8Ω 8Ω
V500 Ω (12 − 2) 500
= = B
VX 2 X 2Ω 2Ω
∴ X = 100 Ω Step (2)
2Ω 2Ω
3×6 A
84 (a) Equivalent resistance of circuit, R = 4 + = 6Ω
3+ 6
3 8Ω 4Ω
and current through battery, i = = 0.5 A
6
Potential difference over 4 Ω = 0.5 × 4 = 2V B
2Ω 2Ω
Potential difference across the resistor of 3 Ω Step (3)
over 6 Ω = 3 − 2 = 1V
for More Books Click HERE -> NEETpassionate.com
250 OBJECTIVE Physics Vol. 2

2W
A RAB = (R + R ) || (R + R )
= 2R || 2R
8W 8W 2R × 2R
= =R
B 2R + 2R
2W
Step (4) 93 (a) The simple circuit is as shown below,
2W
A A
A
4W
2W
2W 2W 1W A
B
2W 2W
Step (5)
Þ Þ2W 2W
Now, equivalent resistance between A and B is
Req = 2 + 4 + 2 = 8 Ω 1W
2W 2W B
88 (c) Total resistance of the circuit,
(R + R2 ) × R3 (15 + 15) × 30 B
R = R4 + 1 = 35 +
(R1 + R2 ) + R3 (15 + 15) + 30 B

30 × 30 ∴ The equivalent resistance across AB,


= 35 + = 50 Ω
30 + 30 2× 2
Req = = 1Ω
50 2+ 2
Current in circuit, i = = 1A
50
94 (b) The simple circuit is as shown below,
1
Current through R3 , i′ = i2 = A
2 10 Ω 10 Ω
Potential difference across R3,
1
V3 = i′ × R3 = × 30 = 15 V 8Ω
2 8Ω 8Ω 24/7 Ω
Q P Q P
89 (b) Potential difference across 100 Ω resistance is 5V. As 6Ω
voltmeter and 100 Ω resistance are in parallel. It means
equivalent resistance of voltmeter and 100 Ω should be 50 Ω. 10 × (8 + 24 / 7)
R net =
So, resistance of voltmeter must be 100 Ω. 10 + (8 + 24 / 7)
iR = 5.3 Ω −
~ 5Ω
90 (d) Emf of cell, E = xil = ×l
L 95 (a) The given circuit consists of a balanced Wheatstone bridge.
e R
∴ E = × ×l 2R
(R1 + R2 + r ) L
5 5 R 2R A R 2/3R B
0.4 = × ×l ⇒
(5 + 45 + 0 ) 10 A B
∴ l =8m 2R
91 (d) Net resistance of circuit, R = 1 + 6 + 4 = 11Ω
2 5
6 ∴ Rnet = R + R= R
I= A 3 3
11 5 5
96 (c) q = ∫ idt = ∫ (1.2t + 3) dt = 30 C
6 6 0 0
∴ V = × 1= V
11 11 97 (a) As RAB = RAO and RBC = ROC , so points O and B will be at
92 (a) The given circuit forms a balanced Wheatstone bridge same potential and hence, resistance ROB becomes ineffective.
between points A and B. Similarly, as RAO = RAD and ROC = RDC , resistance ROD becomes
L ineffective.
So, excluding RBO and ROD , equivalent resistance Req of the
R R
given network between points A and C will be given by
1 1 1 1
B = + +
i.e. A Req 2R 2R 2R
2
R R ⇒ Req = R
3
K
for More Books Click HERE -> NEETpassionate.com
Current Electricity 251

98 (d) The circuit can be drawn as, If n be the number of bulbs possible, then total resistance of
200
circuit = + 10
n
Maximum current in the circuit = 0.5 × n
5Ω P 5Ω 120
⇒ P So, = 0.5n ⇒ n = 4
S 8Ω 8Ω 200
S + 10
n
106 (c) Let RAB = x. Then,
3Ω 3Ω ⇓
1Ω
A

P 1Ω x
4Ω
S B

x x
RAB = 1 + or x = 1+
Resistance between points P and S = 4 Ω 1+ x 1+ x
99 (b) In the first case, potential difference balances against ∴ x + x 2 = 1+ x + x or x 2 − x − 1 = 0
resistance R ,V1 = k l1 ⇒ R = kl1Ω (Q I = 1A) 1+ 1+ 4 1+ 5
In the second case, potential difference balances against R + X, x= = Ω
2 2
V2 = kl2 ⇒ R + X = kl2
107 (a) Potential difference across 4 Ω resistance = 20 + 16 = 36V
∴Potential difference balance over X = k (l2 − l1) Ω
36
100 (d) Power = 1.08 kW = 1080 W Current through 4 Ω resistance = = 9 A [from top to bottom]
4
P × t = E ⇒ 1080 × t = m × L
20 − 16
1080 × t = 100 × 540 × 4.2 ⇒ t = 210 s Similarly, current through 2 Ω resistance = = 2A
2
ig S
101 (b) For an ammeter, = ⇒ ig G = (i − ig )S Therefore, total current through 20 V battery will be 11 A.
i G+S
108 (b) Current through R1 and R2 comes out to be zero (potential
∴ ig G = (0.03 − ig ) 4r …(i)
difference = 0).
and ig G = (0.06 − ig )r …(ii)
Net emf
From Eqs. (i) and (ii), we get ∴ Current through R3 =
Total resistance
0.12 − 4ig = 0.06 − ig
(4 + 3 + 2) − (2 + 3 + 2) 2
Maximum current through the galvanometer, ig = 0.02 A = = A
3 3
102 (c) When key K is opened, bulb B3 will not draw any current
 i
from the source. So, that terminal voltage of source increases. 109 (b) According to the question, R =  20 +  Ω
 2
Hence, power consumed by bulb increases. So, light of the 250 250
bulb B2 becomes more. The brightness of bulb B1 decreases. Now, current, i = =
R 20 + (i / 2)
103 (a) Maximum current possible in galvanometer, ⇒ i 2 + 40i − 500 = 0
Imax = 25 × 4 × 10 − 4 A = 10 − 2 A Solving, we get
2.5 i = 10 A
So, 10 − 2 =
100 + R
110 (d) According to question,
100 + R = 250 dQ
Q = at − bt 2 ⇒ i= = a − 2bt
Resistance, R = 150 Ω dt
a
104 (a) Let, the bulb 400 W is having resistance value of R. For i = 0 at t =
2b
200 W, necessary value of resistance will be 2R. di
= − 2b
Total value of resistance in the circuit will be R + R = 2R dt
If I is the maximum current in the circuit, then I 2R = 400 W 111 (a) The given three resistors are in parallel,
Power of circuit as a whole = I 2 × 2R = 2 × I 2R = 2 × 400 i/3 R
= 800 W
50 R
105 (b) Maximum current possible in bulb = = 0.5 A
100 i R
V 2 100 × 100 V
Resistance of each bulb = = = 200 Ω
P 50 E
for More Books Click HERE -> NEETpassionate.com
252 OBJECTIVE Physics Vol. 2

E 4 Current sensitivity
∴ Current, i = = = 1A 116 (b) Voltage sensitivity =
r + R / 3 1+ 3 Resistance of galvanometer G
i
V = R = E − ir = 4 − 11
( ) = 3V Current sensitivity 10
3 G= = = 5Ω
Voltage sensitivity 2
6×2 7
112 (c) Resistance, Rnet = 2 + = Ω 150
6+ 2 2 Full scale deflection current, ig = = 15 mA
2Ω 3Ω 10
i Voltage to be measured,V = 150 × 1= 150 V
V 150
10 V 2Ω 3Ω Hence, R = − G = − 5 = 9995 Ω
ig 15 × 10 −3
V2
117 (b) We have, × 16 = H (Q H = P × t )
10 20 R
∴ i= = A
(7 / 2) 7 V2 V2 V2
×t = H ⇒ × 16 = ×t
According to current division rule, 0.9 R R 0.9R
 2  1 5 ∴Time, t = 16 × 0.9 = 14.4 min
i3 Ω =   i = ⋅i = A
 2 + 6 4 7 118 (c) Topmost and bottommost figures are short-circuited.
E1 + E 2 E1 Simplified circuit is shown below
113 (d) Current, i1 = and i2 =
r1 + r2 + R R + r1 R
Q i2 > i1
E1 E + E2
∴ > 1 or E1 r2 > E 2 (R + r1) R R B ⇒A
R 1/2R R B
R + r1 r1 + r2 + R A
R
114 (c) Both B and D are symmetrically located with respect to
points O. Hence, the figure can be folded as shown in figure. R 5R
Req = R + R + =
C 2 2
119 (b) Due to balanced Wheatstone bridge, resistance between A
1/2Ω and B can be removed.
1Ω A,C,F

2W 2W
1/2 Ω
B, D O
D 2W G
2W 2W
1Ω
1/2Ω
B,E,H
10 V
A
The given circuit forms Wheatstone bridge with DO or BO 2W
ineffective. The current between D and E,
1 1 10 10
∴ RS = 1 + + = 2 Ω IDE = = = 2.5 A
2 2 RDE + RHG 2 + 2
2×1 2
Resistance across AO = = Ω 120 (b) Resistance between upper branch and lower branch in
2+1 3
parallel part is same, so equal amount of current flows
115 (c) By finding potential difference across any resistance, we through them.
can find current through each resistance directly. Let main current is i.
10 V 4V 8V 6 V i/2 1Ω A 3Ω
3Ω
C
3Ω 1Ω
i
10 Ω 8Ω 4Ω 2Ω A i/2 B
10 V
i
0.8 A 0.25 A 0.5 A 3A
V
∴ i=
∴ i = 3 + 0.25 − 0.5 − 0.8 = 1.95 A Req
for More Books Click HERE -> NEETpassionate.com
Current Electricity 253

Equivalent resistance of circuit, Applying KVL in loop (1) and (2), we get
10 8i3 = 16
Req = 3 + 2 = 5 Ω ⇒ i = = 2A
5 ⇒ i3 = 2A …(i)
So, current in each branch = 1A and 4(i1 − i3 ) = 16 − 8 = 8
Now, VC − VA = 1 × 1 = 1V …(i) ⇒ i1 = 2 + i3 = 4 A [using Eq. (i)]
and VC − VB = 1 × 3 = 3 V …(ii) The current in a circuit is distributed depending on the value
Solving Eqs. (i) and (ii), we have of resistance as shown below.
VA − VB = 3 − 1 = 2 V 2 W 1A 4 W 2A 2A 3W

121 (d) When K1 is closed, R1 is short-circuited. 1A 4A


2W
E E
When K2 is open, I0 = = ...(i) 8W 1W
r + R2 r + 100 1A
— 1A
— 16 V
2 2
1 E  8W 8W
When K2 is closed, I0 = ...(ii)
2 r + 50 
From these two equations, we get r = 0 8V
I E 1
When K1 is open and K2 is closed, 0 = ...(iii) ∴ i2 = A
2 2(R1 + 50 ) 2
From Eqs. (i) and (iii), we have i1 4
⇒ = =8
R1 = 50 Ω i2 (1/ 2)
122 (b) In the circuit, we can see that 20 Ω, 100 Ω and 25 Ω are
in parallel. (B) Medical entrance special format
Net resistance of circuit = 4 + 6 +
1
= 20 Ω questions
1 1 1
+ +
20 100 25 l Assertion and reason
∴ V = iR = 4 × 20 = 80 V 1 (d) R1 and R2 are resistances per unit length.
123 (c) Let current through XY is i3. Applying Kirchhoff’s law to 1
2
loops (1) and (2),
1W X 2W
i1 (i1 – i3)
1 i3 2
(i2 + i3)
i2 3W Y 4W V1 = iR
1 1
i
and V2 = i2R2
 1
50 V R2 > R1  as R ∝ 
 A
i1 + 0 × i3 − 3i2 = 0
As, i1 = i2
∴ i1 = 3i2 …(i)
Hence, V2 > V1
and − 2(i1 − i3 ) + 4 (i2 + i3 ) = 0
3 (b) The resistance of milliammeter becomes high because of
So, 2i1 − 4i2 = 6i3 …(ii) increased number of turns of coil, so the torque produced in
Also, 50 = i1 + 2 (i1 − i3 ) the coil is not decreased due to low value of current.
∴ 3i1 − 2i3 = 50 …(iii) Milliammeter generally do not have shunt because the main
From Eqs. (i), (ii) and (iii), we get current is already very low. Even if there is shunt, its value is
kept high so that current diversion through it is least in case
i3 = 2 A of milliammeter.
124 (b) The simplified circuit can be drawn as, 4 (d) In both cases potential difference across R is E.
i3 (i1 – i3)
E2
∴ P =
16 V R
8Ω 2 4Ω i.e., power remains same.
1 i1
In second case, net resistance will decrease. Therefore, main
current will increase.
8V
for More Books Click HERE -> NEETpassionate.com
254 OBJECTIVE Physics Vol. 2

l 1 In this case, equivalent resistance of the network between


5 (a) R =ρ or R ∝
A A points A and B is independent of the value of G, as no current
Area of cross-section of wire A is less. Hence, its resistance is flows through it.
more. E1 / r1 + E 2 / r2
4 (c) E eq =
Also, H ∝ R, so more heat is generated in wire A. 1/ r1 + 1/ r2
l Statement based questions E1r2 + E 2r1 r + r (E / E ) 
= = E1  2 1 2 1 
1 (a) In parallel circuit, voltage remains same, so r1 + r2  r1 + r2 
VNP = 20 × 1 = 20 V Therefore, E eq can be greater than, less than or equal to E1
20 depending upon the condition whether
∴ INP = = 2A E 2 > E1 , E 2 = E1 or E 2 < E1
10
Current through R, 5 (b) C D
IR = 0.5 + 2 + 1= 3.5 A
VR = 69 − 20 = 49 V A E F B
49 V1 V1 V3 V3 V1 V1 V2 V2 V3 V3 V2 V2
∴ R= = 14 Ω
3.5
Also, VR1 = 20 = 0.5 × R1
⇒ R1 = 40 Ω R/2 C,D R/2
2 (d) When two non-ideal batteries of emfs E1 and E 2 are V1 V3 V3 V2
connected as given below.
A E F B
E1 r1 E2 r2
V1 R V2
R
∴ RAB =
2
V V
⇒ ICD = 0 , IEF = 0
R R R
Then, equivalent emf, E = E1 + E 2 l Match the columns
∴Hence, |E | > |E1| and |E | > |E 2|
1 (a) In parallel connections, net resistance will decrease.
Equivalent internal resistance, req = r1 + r2 Therefore, main current i1 will increase.
Again, when two non-ideal batteries of emfs E1 and E 2 are VAB = VCD = E − ir1
connected as given below. VAB
With increase in i1, VAB orVCD will decrease. Further, i2 = .
E1 r
1 E 2 r2 R
VAB is decreasing. Therefore, i2 will also decrease.
2 (c) The simple circuit is shown in below figure
V1 V2 V3
R 8Ω 3Ω 2Ω

Then, equivalent emf, E ′ = E1 − E 2 (Q E1 > E 2)


Hence, |E′ | may be greater than or equal to or less than the
V
individual value of E1 or E 2.
As, 3 Ω and 6 Ω are in parallel connection.
Equivalent internal resistance, r = r1 + r2
Hence, potential difference across 6 Ω is also 20 V.
Hence, each of statement I and II is wrong.
V 20 20 60
(P + Q )(R + S ) And, for 8 Ω, 1 = + = = 10
3 (d) Reff = 8 3 6 6
(P + Q + R + S ) V = 80 V
(R + R )(2R + 2R ) Similarly, for 12 Ω and 4 Ω,
=
(R + R + 2R + 2R ) V1 V2
=
The equivalent resistance of the network between points A 8 3
and B, 80 V2
⇒ =
4 8 3
Reff = R
3 ⇒ V2 = 30 V
for More Books Click HERE -> NEETpassionate.com
Current Electricity 255

6 (b) Given, R1 = R2, l1 = l2


(C) Medical entrances’ gallery
l R l A
1 (a) Given, drift velocity, v d = 7. 5 × 10 −4 m/s Since, resistance, R = ρ ⋅ ⇒ 1 = 1⋅ 2
A R2 l2 A1
Electric field, E = 3 × 10 −10 Vm −1
R1 l1 A2
Mobility, µ = ? ⇒ = ⋅
R1 l1 A1
v d 7. 5 × 10 −4
As, µ= = A2
E 3 × 10 −10 ⇒ 1=
A1
= 2.5 × 10 6 m2V −1s −1 A1
or =1
2 (c) According to the carbon colour code for resistors, A2
Code of yellow = 4
Code of violet = 7 7 (b) Applying KVL rule in loop BCDEB,
Code of brown, i.e. multiplier = 101 R2i2 + E 2 − E 3 − i3 R1 = 0

Code of gold, i.e. tolerance = ± 5% 8 (b) Equivalent resistance between points A and B is
Hence, resistance of resistor = 47 × 101Ω , 5% RAB = [(4 + 8) || 6] + 4 + 8 = [12 || 6] + 12
 12 × 6 
= 470 Ω , 5% =  + 12 = [4] + 12 = 16 Ω
12 + 6 
3 (a) According to the question, the metre bridge is shown
below, 9 (a) Case I When all bulbs are glowing, then the circuit can be
R 10 Ω realised as shown in the figure below.
A B
R R
G
R R
x1 x2
100 cm
R R
x1 3
Given, =
x2 2
At balance condition in metre bridge, E
R x
= 1 ∴The equivalent resistance of this circuit is
10 x 2
Req = RA + RB
x1 3
⇒ R= × 10 = × 10 = 15 Ω As, section A has three parallel resistance, so equivalent
x2 2 resistance,
Now, length of given wire of resistance 15 Ω is 1.5 m. R
RA =
Therefore, length of 1Ω resistance wire is 3
1. 5 1 R
= = = 0.1 or 1 × 10 −1 m Similarly, for section B, equivalent resistance, RB =
15 10 3
4 (b) Resistivity of copper (a metal) as a function of temperature R R 2R
∴ Req = + = … (i)
increases with the increase in temperature as shown below, 3 3 3
ρ Thus, power consumed in this circuit,
V 2 E 2 3E 2
P1 = = = [using Eq. (i)] …(ii)
R Req 2R
Case II When two from section A and one from section B
glow, the circuit can be realised as shown in the figure below.
O T (K)
A B
For copper at 0K, value of resistivity is 1.7 × 10 −8 Ω-m. R
R
Hence, correct option is (b). R

5 (b) Applying KVL in the loop, we get


4I + I ⋅ 1 − 4 + I ⋅ 1 − 2 = 0
⇒ 6I = 6 ⇒ I = 1A
E
for More Books Click HERE -> NEETpassionate.com
256 OBJECTIVE Physics Vol. 2

∴ Equivalent resistance of section A, 13 (d) The given circuit diagram can be drawn as shown below
R
RA = I1 20 Ω B 30Ω
2 A E
and of section B, V
RB = R 30 Ω 20 Ω
D F
Thus, equivalent resistance of the entire circuit becomes I I2 C
R 3R
Req = RA + RB = + R = … (iii)
2 2
∴Power consumed by this circuit, 2V
2 2 2 The equivalent resistance of circuit is given by
V E 2E
P2 = = = [using Eq. (iii)]… (iv)
R Req 3R 1 1 1
= +
Req RAE RDF
So, ratio of power in two cases is obtained from Eqs. (ii)
P1 3E 2 3R 9 1 1
and (iv), = × = or 9 : 4 = +
P2 2R 2E 2 4 (20 + 30 ) (30 + 20 )
1 1 2
10 (c) Among given devices fuse is used in electric circuit as a = + =
protection device. 50 50 50
It helps in preventing excessive amount of current to flow in ⇒ Req = 25 Ω
the circuit or from short circuiting. V 2
The current in circuit, I = = A
It has low melting point and low resistivity, so when excess R 25
amount of current flows in the circuit, then due to excessive As the resistance of two branches is same, i.e. 50 Ω.
amount of heat, it melts and breaks the circuit. So, the current I1 = I2
11 (b) For an ideal voltmeter, the resistance is infinite and for an ⇒ I = I1 + I2
ideal ammeter, the resistance is zero. 2
10 Ω
⇒ = 2I1
i1 25
1
⇒ I2 = I1 = A
V1
25
A1
∴The voltage across AB,
1
V1 = I1R1 = × 20 V
25
10 V Circuit 1
and voltage across CD,
So, the current in circuit 1 is 1
V2 = I2R2 = × 30 V
10 25
R × i = V or 10 i1 = 10 ⇒ i1 = = 1A
10 ∴Voltmeter reading
∴ V1 = i1 × R = 1 × 10 = 10 V 30 20 10
= V2 − V1 = − = = 0.4 V
Similarly, for circuit 2, the addition of 10 Ω to voltmeter does 25 25 25
not affect the current and hence R 1
10 Ω i2
14 (a) Given, for a wire, =
l 2
10 Ω Length of wire, l = 5 cm = 5 × 10 −2 m
V2 A2 l
∴ R= = 2 .5 × 10 −2 Ω
2
Potential difference,V = 1V or IR = 1
10 V Circuit 2
1 1 100
10 I= = = = 40 A
10 i2 = 10 ⇒ i2 = = 1A R 2 .5 × 10 −2 2 .5
10
15 (c) Given, current, I = 10 A
V2 = i2R = 1 × 10 = 10 V
∴ V1 = V2 and i1 = i2 Area of cross-section, A = 4 × 10 −6 m 2
12 (d) The balance condition still remains the same, if the jockey Density of conductors, ρ = 2 .7 g/cc = 2 .7 × 10 3 kg/m 3
points the same point P given in the initial condition, for
Molecular weight of aluminium, Mw = 27g = 27 × 10 −3 kg
which there is no deflection in the galvanometer or no
current will be drawn from the cell. Thus, the bridge will If n be the total number of electrons in the conductor per unit
work as usual and balance condition is same, P / Q = l1 / l2. volume, then
for More Books Click HERE -> NEETpassionate.com
Current Electricity 257

Total number of electrons So, current in 8 Ω resistor,


n=
Volume of conductor V ( ) V 24 − 0
I= = = 3A
Number of atoms per mole × Number of moles R 8
=
V 19 (a) Given, resistivity of copper, ρ = 1.72 × 10 −8 Ω-m
Avogadro number  M 
= ×  Electrons density, n = 8.5 × 10 28 / m3
V  Mw 
1 1
∴ Mobility (µ ) = =
23 ρ 2 .7 × 10 3 ρ ne 1.72 × 10 −8 × 8.5 × 10 28 × 1.6 × 10 −19
= 6 × 10 × = 6 × 10 23 ×
Mw 27 × 10 −3
~ 4.25 × 10 −3 m 2/C Ω

∴ n = 6 × 10 28
20 (a) Power rating of heater, P = 1000 W
We know that, drift velocity,
Voltage rating of heater,V = 100 V
I 10
vd = =
neA 6 × 10 28 × 1.6 × 10 −19 × 4 × 10 −6
V″ Heater
= 2 .6 × 10 −4 m/s + – + V′ –
10Ω
16 (b) The situation is shown in the circuit diagram.
R
R1 R2

V1 V2 100 V

∴Resistance of heater,
I Vi V 2 (100 )2
R1 = = = 10 Ω
Vi P 1000
Current flowing through the circuit, I = According to question, power dissipated in heater,
R1 + R2
P′ = 62 . 5 W
Voltage across R2, V2 = IR2
VR V R + R2 ∴Voltage V′
( ) across heater can be calculated as
V2 = i 2 ⇒ i = 1 ( ′ )2
V
R1 + R2 V2 R2 P′ =
R1
17 (d) If R1 be the equivalent resistance of parallel resistors
⇒ ( ′ )2 = P ′ × R1 = 62 . 5 × 10
V
8 Ω, 8 Ω and 4 Ω, then
1 1 1 1 4 ⇒ V′ = 25 V (across heater)
= + + =
R1 8 8 4 8 ∴ Voltage across 10 Ω resistor,
⇒ R1 = 2 Ω V ′′ = 100 − 25 = 75V
∴Total resistance of upper branch, Current in 10 Ω resistor
R2 = R1 + 4 = 2 + 4 = 6 Ω V ′′ 75
= = = 7.5 A
10 10
Now, circuit can be redrawn as
Current in heater resistor
I1 6W
V ′ 25
= = = 2. 5 A
10 10
12A
So, current in R, I = 7.5 − 2.5 = 5A
12 W Now,V = IR
25
I2 ⇒ R =V / I = =5Ω
5
By current division rule, current through 12 Ω resistor, 21 (b) Given, R = (47 ± 4.7) kΩ
12 × 6 1
I2 = = 12 × = 4A = 47 × 10 3 ± 10% Ω
12 + 6 3
∴Voltage across 12 Ω resistor, As per the colour code for carbon resistors, the colour
assigned to numbers, 4-Yellow, 7-Violet, 3-Orange.
V = I2R = 4 × 12 = 48 V
For ±10% accuracy, the colour is silver. Hence, the bands of
18 (b) As the other end of 8 Ω resistor is grounded, which is at colours on carbon resistor are in sequence Yellow, Violet,
zero potential, the potential difference across 8 Ω resistor Orange and Silver.
= 24 − 0 = 24 V
for More Books Click HERE -> NEETpassionate.com
258 OBJECTIVE Physics Vol. 2

22 (c) When n equal resistors of resistances R are connected in So, circuit becomes as shown below
series, then the current drawn is given as B
E 5W 10 W
I=
nR + r A D

where, nR = equivalent resistance of n resistors in series and 10 W 20 W


r = internal resistance of battery. i
C
Given, r =R
E E 5V
⇒ I= = …(i)
nR + R R (n + 1) Now, in the above circuit, RABD = 5 + 10 = 15 Ω
Similarly, when n equal resistors are connected in parallel, RACD = 10 + 20 = 30 Ω
then the current drawn is given as Resistance RABD and RACD are in parallel, so
E 15 × 30 450
I′ = Rnet = = = 10 Ω
R 30 + 15 45
+R
n E 5
R ∴ i= = = 0.5 A
where, = equivalent resistance of n resistors in parallel. Rnet 10
n
25 (c) During charging, E = V − ir (due to reversed current). In
Given, I′ = 10 I
case of charging, emf of a cell is less than its terminal voltage
E nE
⇒ 10I = = …(ii) while in case of discharging emf is greater than terminal
R
+R (n + 1)R voltage.
n Therefore, Assertion is correct but Reason is incorrect.
Substituting the value of I from Eq. (i) in Eq. (ii), we get
26 (b) Figure is equivalent to the one shown below. It is a
 E  nE Wheatstone’s bridge in which
10   =
 R (n + 1) R (n + 1) C

⇒ n = 10
6W

8W
Q
P
3W

8W
23 (c) If n identical cells are connected in series, then equivalent
emf of the combination, P 10W 10W Q
E eq = nE

W
S
4W

20
Equivalent internal resistance, R

req = nr
5W
4W

E eq D
nE
∴ Current, I = = 6×3
req nr P = = 2Ω
6+ 3
E
or I= = constant 8 × 8 64
r Q= = = 4Ω
8 + 8 16
Thus, current (I ) is independent of the number of cells (n )
4 × 4 16
present in the circuit. R= = = 2Ω
4+ 4 8
Therefore, the graph showing the relationship between I and
20 × 5
n would be as shown below and S = = 4Ω
20 + 5
P 2 R
I We find that, = =
Q 4 S
i.e., the bridge is balanced and resistance of arm CD is
ineffective. Effective resistance between P and Q,
(P + Q ) (R + S ) (2 + 4) (2 + 4) 6 × 6
RPQ = = = = 3Ω
O n P +Q +R+S 2+ 4+ 2+ 4 12

24 (a) Given circuit satisfies the Wheatstone bridge condition, so 27 (a) Switching results in high decay/growth rate of current
no current flows in the branch BC and it behaves like an open which results in a high current when bulb is turned ON or
circuit. OFF (due to back emf). So, a bulb is most likely to get fused
when it is just turned ON or OFF.
for More Books Click HERE -> NEETpassionate.com
Current Electricity 259

28 (c) Q Current, I = nAev d 31 (c) In the given circuit, net resistance between P and Q points
1 is RPQ and net resistance between Q and R points is RQR , then
or vd ∝ 10 × 8 80 40
A RPQ = = = Ω
If diameter of wire is d /4, then area will be A / 16, so new 10 + 8 18 9
drift velocity = 16v d . 10 × 18 × 25 4500 225
RQR = = = Ω
10 × 18 + 18 × 25 + 25 × 10 880 44
29 (c) Volume of material remains same in stretching.
As volume remains same, Al11 = A2l2 So, circuit will be
40 225 Ω
Given, l2 = nl1 Ω
9 Q 44
P R
∴ New area, A2 = Al11/l2 = A1/n
Resistance of wire after stretching,
l2 nl  l    l1  
R2 = ρ = ρ 1 = ρ 1  n 2 = n 2R Q R = ρ A  
A2 A1 / n  A1  1 
+ –

10 W C 10 W 10 W G 10 V
30 (b) E
A In the series circuit, voltages will be divide in the ratio of
their resistance.
40 W 40 W 20 W
RPQ VPQ
∴ =
RQR VQR
B F
10 W D 10 W 10 W H
 40 
  V
In the circuit, the branch EGHF have three resistances of 10  9 PQ 1760 VPQ
∴ = or =
Ω, 20 Ω and 10 Ω respectively, which are connected in series  225 VQR 2025 VQR
 
combination.  44 
So, their equivalent resistance is given by 1760 VPQ
⇒ =
R1 = 10 + 20 + 10 = 40 Ω 1760 + 2025 VPQ + VQR
This R1 resistance is in parallel with 40 Ω resistance which is
1760 VPQ
connected in the branch EF. So, their equivalent resistance, ⇒ =
3785 10
40 × 40
R2 = = 20 Ω ⇒ VPQ = 4.65 V
40 + 40
Now, circuit becomes 32 (b) Q 60 Ω and 30 Ω resistors are connected in parallel.
10 W C 10 W E So, their net resistance
A 30 × 60 1800
Rnet = = = 20 Ω
40 W 20 W 30 + 60 90
2 2
∴ i= = = 0.1 A
B F Rnet 20
10 W D 10 W

Now, in the branch CEFD 10 Ω, 20 Ω and 10 Ω resistance 33 (b) As each arc containing n lamps, hence resistance of each
are connected in series combination and their equivalent arc = nr, number of arcs = N /n
resistance is given by Equivalent resistance S is given by
R3 = 10 + 20 + 10 = 40 Ω 1 1 N  1
=Σ =  
This R3 is in parallel with 40 Ω resistance which is in branch S nr n  nr 
40 × 40 n 2r
CD. Their equivalent resistance, R4 = = 20 Ω S=
40 + 40 N
Now, circuit becomes n 2r
10 Ω ∴ Total resistance = R + S = R +
C N
A
If E is the emf of the machine, current entering the arcs is
20 Ω E / (R + S ) and in each arc is nE / (R + S )N .
Hence, current passing through each lamp,
B −1
10 Ω D nE E R nr 
I= = +
The net resistance between A and B, N (R + n 2r / N ) N  n N 
Rnet = 10 + 20 + 10 = 40 Ω Now, heat produced per second in the lamps is H = NrI 2.
for More Books Click HERE -> NEETpassionate.com
260 OBJECTIVE Physics Vol. 2

Since, light emitted is proportional to H 2, therefore light Hence, largest amount of heat will be generated in case of
produced is maximum when H 2 and hence H is maximum or four parts connected in parallel.
R nr  37 (d) Here, 2 Ω and 2 Ω are in parallel.
+ is minimum.
 n N  1 1 1
2
R nr  R 
1/ 2
 nr  
1/ 2
 Rr 
1/ 2 ∴ = +
Hence, we can write, + =   −    + 2   R 2 2
n N  n  N

N
2×2
⇒ R= = 1Ω
R nr 2+ 2
This is minimum, when − =0
n N Now, resistance 1 Ω, 2 Ω, 4 Ω and 1Ω are in series.
1/ 2
or very small or n is closely equal to (NR /r ) . ∴ Rnet = 1 + 2 + 4 + 1= 8 Ω
34 (a) It is clear that the two cells oppose each other, hence the V 4
Hence, current, I = = = 0.5 A
effective emf in closed circuit is 18 − 12 = 6V and net R 8
resistance is 1 + 2 = 3 Ω (because in the closed circuit the 38 (c) According to question, emf of the cell is directly
internal resistance of two cells are in series). proportional to the balancing length, i.e., E ∝ l …(i)
The current in circuit will be in direction of arrow shown in Now, in the first case, cells are connected in series to support
figure one another, i.e. net emf = E1 + E 2
effective emf From Eq (i), E1 + E 2 = 50 cm (given) …(ii)
I= = 6/ 3 = 2A
total resistance Again cells are connected in series but in opposite direction,
The potential difference across V will be same as the terminal i.e. net emf = E1 − E 2
voltage of either cell. From Eq. (i), E1 − E 2 = 10 cm (given) …(iii)
From Eqs. (ii) and (iii),
V
E1 + E 2 50
=
I E1 − E 2 10
18V 2Ω
E1 5 + 1 6 3
⇒ = = =
E2 5 − 1 4 2
12V 1Ω
39 (d) Given, charge, Q = at − bt 2
dQ
Q We know that current, I = …(i)
Since, current is drawn from the cell of 18V, hence dt
V1 = E1 − Ir1 = 18 − (2 × 2) So, Eq. (i) can be written as
= 18 − 4 = 14V d
I = (at − bt 2 )
dt
35 (a) When a voltmeter is connected across the two terminals of
a cell, it draws a small current from the cell, so it measures ⇒ I = a − 2bt …(ii)
terminal potential difference between the two terminals of For maximum value of t,
the cell, which is always less than the emf of the cell. On the dQ
=0
other hand, when a potentiometer is used for the dt
measurement of emf of cell, it does not draw any current d
from the cell. Hence, it accurately measures the emf of cell. or (at − bt 2 ) = 0
dt
Thus, a potentiometer is preferred over a voltmeter.
⇒ a − 2bt = 0
36 (c) Let R be the resistance of the wire, then a
∴ t= …(iii)
V 2t 2b
(i) The heat generated is H1 = .
R Q The total heat produced (H ) can be given as
t a / 2b
(ii) Resistance of each part will be R / 2. When they are H = ∫ I 2R dt = ∫ (a − 2bt )2 R dt
connected in parallel, the resistance will be R /4. 0 0
a / 2b
Hence, H 2 = 4V 2t /R. =∫ (a 2 + 4b 2t 2 − 4abt ) Rdt
0
(iii) In case of four wires connected in parallel, the resistance a / 2b
will be R / 8.  t 3 4abt 2 
H = a 2t + 4b 2 − R
8V 2t  3 2  0
∴ H3 =
R Solving above equation, we get
2 2
V t 2V ⋅ t a 3R
(iv) H 4 = = ⇒ H=
R /2 R 6b
for More Books Click HERE -> NEETpassionate.com
Current Electricity 261

2W + – 1W It is clear from Eq. (i), when area increases the current


40 (d) V 2 A VB
A
density decreases so the number of flow of electrons will be
3V same and thus the current will be constant.
Applying KVL, 45 (c) When voltmeter is connected across A and B, the
VA + ΣV = VB + 2 × 2 + 2 × 1 equivalent resistance of the circuit is
VA − VB − 3 = 4 + 2;VA − VB = 9 V 150 Ω
41 (c) If a rated voltage and power are given, then V

V2 A C
Prated = rated 100 Ω B 100 Ω
R
∴ Resistance of bulb, 50 V
100 × 100
Rb = = 20 Ω 150 × 100 15000
500 Req = 100 + = 100 + = 100 + 60 = 160 Ω
100 + 150 250
P
And current in the bulb, i = 50 5
V ∴ Current, i = = A
160 16
500
⇒ i= = 5A Therefore, potential drop across B and C is
100
5 500
Q Resistance R is connected in series, VBC = i RBC = × 100 = = 31.25 V −
~ 31V
16 16
E 230
∴ Current, i = =
R net R + Rb 46 (a) Figure can be redrawn as
230 M
⇒ R + 20 = = 46
5 10 Ω 10 Ω
∴ R = 26 Ω 10 Ω

42 (a) Given, l = 4 m a c
10 Ω b 10 Ω
R = potentiometer wire resistance = 8 Ω
dV
Potential gradient = = 1mVcm −1
dl
6V
So, for 400 cm, ∆V = 400 × 1 × 10 −3 = 0.4 V
Let a resistor RS is connected in series, so as 10 10
As, = (from figure)
V 10 10
∆V = ×R
R + RS So, circuit is said to be Wheatstone bridge in balanced
2 condition and hence current through arm Mb is zero, i.e. no
⇒ 0.4 = ×8 current flows.
8 + RS
Now, the equivalent resistance of the circuit is given by
16
⇒ 8 + RS = = 40 ⇒ RS = 32 Ω 1 1 1 2
0.4 = + = ⇒ Req = 10 Ω
Req 20 20 20
43 (a) The equivalent resistance between Q and S is given by
∴ Current supplied by the battery,
1.5 R V 6
R B i= = = 0.6 A
P Q S R 10
A
C 47 (a) Suppose, actual current through the ammeter is I.
3R Now, we can write
V Ir = kl1 ⇒ I × (1) = k (75) …(i)
where, k is a constant.
Similarly, 1.02 = k (50 ) …(ii)
1 1 1 2+1
= + = ⇒ R′ = R From Eqs. (i) and (ii), we get
R′ 1.5R 3R 3R
I 75 3
Now, VPQ = VA = IR = =
1.02 50 2
Also, VQS = VB = VC = IR
3
Hence, VA = VB = VC ⇒ I = (1.02) = 3 (0.51) = 1.53 A
2
44 (b) As the cross-sectional area of the conductor is non-uniform,
so current density will be different. Thus, error in ammeter reading is
As, I = JA …(i) ∆I = (1.53 − 1.5) A = 0.03 A
for More Books Click HERE -> NEETpassionate.com
262 OBJECTIVE Physics Vol. 2

48 (b) The current in case of voltmeter of range 10 V and 55 (a) Kirchhoff’s first law states that, algebraic sum of currents
resistance 50 Ω is meeting at a point in a circuit is zero. It is based on
10 conservation of charge.
I= A …(i)
50 Kirchhoff’s second law states that, the algebraic sum of
Let X be the resistance connected in order to make a potential differences around a closed loop is zero. It is based
voltmeter with range 15 V having current, on the law of conservation of energy.
15 56 (b) Using the formula, m = ZI, where m = mass of silver
I= A …(ii)
X
deposited = 0.05% of 750 g = 0.375 g = 3.75 × 10 −4 kg
Equating Eqs. (i) and (ii), we get
15 10 (15) × (50 ) Current passing through it, I = 0.8 A
= = = 75 Ω Z = ECE of silver = 11.8 × 10 −7 kgC −1
X 50 (10 )
Then, R = 75 − 50 = 25 Ω must be connected in series, ∴The time needed for depositing silver is given by
becauseV ∝ R when current is constant. m 3.75 × 10 −4
t= = = 397 s = 6 min 37 s
50 (e) Current remains constant throughout the metallic ZI 11.8 × 10 −7 × 0.8
I 57 (c) The shunt resistance required is given by
conductor. Current density J = is not constant because
A  I   100  100
cross-sectional area is a variable parameter. Drift velocity S =  DC  RDC =   × 0.1 = × 0.1
 I − IDC   500 − 100  400
I 1
vd = is not constant. Since, v d ∝ .
neA A ∴ S = 0.025 Ω
If this value of shunt resistance is connected in parallel with
51 (b) Given,
the DC ammeter, then the range will be extended to 0-500 A.
Colour coding of resistance = Red, Red, Green, Silver
First band value = 2 58 (a) The circuit diagram can be redrawn as the potential
Second band value = 2 between A and B is
Third band value = 10 5 A
D E
Fourth band value = ± 10%
- 2V
Now, R = 22 × 10 5 ± 10% Ω 2Ω 2Ω
+
R = 2200 ± 10% kΩ
I + 2Ω +
52 (c) Slope = 2V 2V
V - −
Q V = IR I
C F
I 1 I′ B I−I′
∴ =
V R
Hence, slope is reciprocal of resistance. For the loop ABCDA, +2 − 2I′ + 2 − 2I = 0
53 (a) The resistance of two wires are 2 = I + I′ …(i)
l l For the loop ABFEA ,
R1 = ρ1 and R2 = ρ 2
A A 2 − 2I + 2 − 2(I − I′ ) = 0
Now, equivalent resistance of series connection of wire 4 − 4I + 2I′ = 0
l l l 2 = 2I − I′ …(ii)
Req = R1 + R2 = ρ1 + ρ 2 = (ρ1 + ρ 2 )
A A A From Eqs. (i) and (ii), we get
2l 4 = 3I ⇒ I = 4/ 3 = 1.33 A
The equivalent resistance Req can be given by Req = ρ eq
A l l2
2l l 59 (c) As we know that, R1 = ρ =ρ
⇒ ρ eq = (ρ1 + ρ 2 ) a V
A A where, l = length of wire,
ρ + ρ2 a = area of cross-section of the wire
Hence, ρ eq = 1
2 and V = volume of the wire.
54 (a) Total resistance = 50 + 3950 = 4000 Ω As, R1 ∝ l 2
2 2
For this circuit, deflection of resistance R be R1  l1   1
⇒ =  = 
4000 30 4000 R2  l2   2
⇒ = ⇒ R= = 2000 Ω
R 15 2 ⇒ R2 : R1 = 4 : 1
Then, resistance in series should be = 2000 − 50 = 1950 Ω
for More Books Click HERE -> NEETpassionate.com
Current Electricity 263

60 (c) The combination of resistors can be redrawn as 2 = 1 + [0.005 (T2 − 100 )]


a 0.005 (T2 − 100 ) = 1
1
T2 − 100 =
0.005
R R R 1000
⇒ T2 − 100 =
5
T2 − 100 = 200
b ⇒ T2 = 200 + 100 = 300 ° C
Therefore, equivalent resistance is given by 66 (d) Consider the ring as two parts. As two resistances are
1 1 1 1 R joined in parallel between two points A and B, then two
= + + ⇒ Req =
Req R R R 3 resistances would be
R R
61 (c) From the circuit (potentiometer), R1 = ⋅rθ = θ
2πr 2π
R R
i E and R2 = r (2π − θ ) = (2π − θ )
R
2πr 2π
Now, equivalent or effective resistance between A and B,
B R × R2
A C Req = 1
G
R1 + R2
E1 R R  R 2θ (2π − θ ) 
θ× (2π − θ )  
VCB = E (if no current is drawn from the battery) ⇒ Req = 2π 2π = 4π 2 
R  2 π R 
 E1  [θ + 2π − θ]
⇒ 2π  2π 
R + R  RCB = E …(i)
 AB 
R 2θ (2π − θ ) 2π Rθ (2π − θ )
Given, E1 = 2 V and RAB = 10 Ω = × =
4π 2
2πR 4π 2
40
∴ RCB = × 10 = 4 Ω and E = 10 × 10 −3 V 67 (b) Case I For balanced point of meter bridge,
100
5 l1
On solving Eq. (i), we get R = 790 Ω = …(i)
R 100 − l1
62 (d) We know that, potential drop across a resistance,
Case II When R is shunted with equal resistance, i.e. R
V = I R ⇒ V = constant
1 1 1
1 = + ⇒ R′ = R /2
i.e. I∝ R′ R R
R
5 1.6 l1
1 1 1 1 1 1 ∴ = …(ii)
I1 : I2 : I3 = : : = : : = 6: 4: 3 R / 2 100 − 1.6 l1
R1 R2 R3 2 3 4
From Eqs. (i) and (ii), we get
63 (a) Total internal resistance does not change  l1  1.6 l1
∴ R′ = 4r 2  = ⇒ l1 = 25 cm
 100 − l1 100 − 1.6 l1
Net emf, E ′ = E (n − 2m )
5 25
Here, n = total number of cells = 4 From Eq. (i), = ⇒ R = 15 Ω
R 75
m = wrong connection = 1
E ′ = E[4 − 2] , i.e. E ′ = 2E 68 (c) As, r = internal resistance,
 l − l1 rl
64 (a) Given, I = 3 A andV = 6 V and r = R1   ⇒ 1 = l − l1 …(i)
 l1  R1
We know that,V = IR
V 6  l − l2  rl
R= ⇒ R = =2Ω Also, r = R2   ⇒ 2 = l − l2 …(ii)
I 3  l2  R2
If the ammeter and voltmeter have resistance, then R < 2 Ω. Subtracting Eq. (i) from Eq. (ii), we get
65 (b) Given, R1 = 100 Ω l l 
l1 − l2 = r  2 − 1 
R2 = 200 Ω ⇒ T1 = 100 ° C  2
R R 1

α = 0.005 per °C At, R1 = ∞, l1 = 3 m and at, R2 = 9.5 Ω, l2 = 2.85 m


We know that, new resistance of the bulb filament,  2.85 
⇒ (3 − 2.85) = r −0
R2 = R1 [1 + α (T2 − T1)]  9.5 
200 = 100 [1 + 0.005 (T2 − 100 )] ⇒ r = 0.5 Ω
for More Books Click HERE -> NEETpassionate.com
264 OBJECTIVE Physics Vol. 2

2 Now, 6 Ω and 6 Ω are in series.


69 (a) As, current IG = I
1000 So, R′ = 6 + 6 = 12 Ω
Resistance of 1 1 1 1 1 2
∴ = + = + =
galvanometer R′′ R′ 12 12 12 12
I IG
G ⇒ R′′ = 6 Ω
Ir The required resistance of the circuit = 2 + 6 = 8 Ω.
So, the potential difference,
r VA −VB = iR = 0.5 × 8 = 4 V
 G 
Also, Ir =  I 74 (a) Total emf of the cell = 3E − E = 2E
G + r
E E E E
We know that, potential across G and shunt r are same.
∴ VG = Vr ⇒ IG (G ) = Ir r
2IG GI
⇒ = (r ) ⇒ 2(G + r ) = 1000 r
1000 G + r
Total internal resistance = 4r
G
⇒ G + r = 500 r ⇒ + 1 = 500 ∴Total resistance of the circuit = 4r + R
r
So, the current in the external circuit
1
⇒ G /r = 499 ⇒ r = G 2E  V
499 i= Q i = 
4r + R  R
70 (c) Corresponding to the colours of the first and second bands,
i.e. green and black, the figures are 5 and 0. 75 (b) As, resistances 1Ω, 2 Ω and 1Ω are in parallel.
Corresponding to the colour of third band, i.e. violet, the So, the required internal resistance,
multiplier is 10 7. Therefore, the value of the resistance is E1 = 1V
50 × 10 7 Ω. The gold colour of the fourth band indicates the
E2 = 2V
tolerance of ±5%.
So, the value of the resistor is written as E3 = 3V
P Q
50 × 10 7 Ω ± 5% = 500 × 10 6 Ω ± 5% = 500 ± 5% MΩ
1 1 1 1 1 1 1 2 + 1+ 2 2
R R = + + = + + = ⇒ r= Ω
× r r1 r2 r3 1 2 1 2 5
R 9
71 (c) The required resistance = 2 2 = = Ω
R R 4 4 The potential difference between points P and Q,
+
2 2 E1 E 2 E 3 1 2 3
+ + + +
r r2 r3
72 (c) Let the values of resistances be R1 and R2, respectively. E = 1 =1 2 1
1/r 5/ 2
When R1 and R2 resistances are in series, then
R1 + R2 = 6 (according to question) …(i) 2+ 2+ 6
2 10 / 2 10 2
When R1 and R2 resistances are parallel, then = = = × = 2V
R1 R2 4 5/ 2 5/ 2 5 2
= …(ii)
R1 + R2 3 76 (d) Given, R1 = 2 Ω, R2 = 6 Ω, E = 2 V, r = 0.5 Ω, I = ?
From Eq. (i), we get Q R1, R2 are in parallel combination.
R1 R2 4 1 1 1 1 1 3+ 1 6
= ⇒ R1 R2 = 4 × 2 ⇒ R1 R2 = 8 …(iii) So, = + = + = ⇒ R = = 1.5 Ω
6 3 R R1 R2 2 6 6 4
We know that, Then, the current in the circuit,
R1 − R2 = (R1 + R2 )2 − 4RR
1 2 = 36 − 4 × 8
E 2 2
I= = = = 1A
⇒ R1 − R2 = 4 r + R 0.5 + 1.5 2.0

⇒ R1 − R2 = 2 …(iv) (as internal resistance, r is in series with other resistance)


drift velocity
From Eqs. (i) and (iv), we get 77 (b) As, mobility =
R1 = 4 Ω , R2 = 2 Ω electric field
vd metre 2 [L]2
73 (d) Given, 6 Ω , 6 Ω and 6 Ω are in parallel. ⇒µ = = =
E volt-second (joule/coulomb) -second
1 1 1 1 3
So, = + + = [L]2 [L2]
R 6 6 6 6 = 2 −2
= 2 −2 −1
= [M−1T 2A]
[kg-metre - second ]-second [ML T A ]
6
∴ R = =2Ω ampere-second
3
for More Books Click HERE -> NEETpassionate.com
Current Electricity 265

l 85 (d) As, kl1 = E − ri = E − r (0 ) = E


79 (b) Resistance of wire, R = ρ
A
⇒ kl1 = E …(i)
Given, R1 = 4 Ω, l1 = l, A1 = A
Potentiometer battery
l2 = 2l, A2 = A/ 2, ρ = constant
R1 l1/A1 l1 × A2
∴ = = l1
R2 l2 /A2 l2 × A1
4 l × A/ 2 4 1
⇒ = ⇒ = ⇒ R2 = 16 Ω
R2 2l × A R2 4 G I =0
r i=0 g
E
80 (a) Balanced condition for Wheatstone bridge,
P R
= …(i)
Q s l2
1 1 1 ss G Ig = 0
where, = + ⇒ s= 12 …(ii)
s s1 s2 s1 + s2
i1 E r
From Eqs. (i) and (ii), we get
P R (s1 + s2 )
= i1 R
Q s1s2
and kl2 = E − ri1 = Ri1
81 (b) Force experienced by electron, F = qE E RE
⇒ i1 = ⇒ kl2 = …(ii)
where, E = electric field intensity. R+r R+r
∴2.4 × 10 −19 = 1.6 × 10 −19 E On dividing Eq. (i) by Eq. (ii), we get
⇒ E = 1.5 NC −1 l1 R + r r l 
= = 1 + ⇒ r = R  1 − 1
Moreover, E = potential gradient l2 R R  l2 
dV
E = We know that for internal resistance,
dl
l 
⇒ E × l =V r =  1 − 1
 l2 
⇒ V = 1.5 × 6 = 9 V
V 120 where, l1 = balancing length of potentiometer wire = 240 cm,
82 (b) Resistance, R = − G = − 10 = 12000 − 10 l2 = balancing length after shunting = 120 cm
ig 0.01
and R = shunting resistance = 2 Ω.
R = 11990 Ω Putting all the values, we get
To convert a galvanometer into a voltmeter, high resistance of
value 11990 Ω should be connected in parallel.  240 
Internal resistance, r =  − 1 × 2 = (2 − 1) × 2 = 2 Ω
 120 
83 (b) Given, first balancing length, l1 = 30 cm
86 (b) Let, Rx resistance be connected in series to convert
Second balancing length, galvanometer to voltmeter.
l2 = 40 cm ⇒ E1 = 1.25 V
G
E2 = ? Ig
Rg Rx
So, according to the principle of potentiometer,
V
E1 = Kl1 …(i)
and E 2 = Kl2 …(ii) According to the question, Rg = 100 Ω, Ig = 30 mA
E1 Kl1 1.25 30 (Current corresponding to full scale deflection)
Q = ⇒ = Now, we can write,V = Ig × Rg + IgRx = Ig (Rg + Rx ) …(i)
E 2 Kl2 E2 40
Given, V = 30 V
1.25 × 40
⇒ E2 = From Eq. (i), we get
30
5 30 = (30 × 10 −3) (100 + Rx )
⇒ E 2 = = 1.666 V −
~ 1.67 V
3 30
⇒ = 100 + Rx
84 (a) Potentiometer measures the potential difference more 30 × 10 −3
accurately than a voltmeter because while measuring emf ⇒ 10 3 = 100 + Rx ⇒ Rx = 1000 − 100 = 900 Ω
(electromotive force), it does not draw any current from the
source of known emf. Potentiometer has its own battery 87 (b) By Faraday’s law of electrolysis, m = Zit
which maintains constant potential drop across the Given, m1 = m gram, i1 = 4 A, i2 = 6 A, t1 = 120 s
potentiometer wire. t2 = 40 s, m 2 = ?
for More Books Click HERE -> NEETpassionate.com
266 OBJECTIVE Physics Vol. 2

m1 it11 m 4 × 120 96 (d) We know that,


Q = ⇒ =
m 2 i2t2 m2 6 × 40 E 15 15
i= = = = 1A
m × 6 × 40 m r + Reff 0.5 + 14.5 15
⇒ m2 = =
4 × 120 2
97 (b) Given, temperature coefficient of carbon,
88 (d) As R ∝ l 2. When wire is stretched, α 1 = 4 × 10 −3/ ° C
∴ R′ = n 2R = 22 × 4 = 4 × 4 = 16 Ω Temperature coefficient of copper,
89 (b) As I =
E
or E = I (R + r ) α 2 = − 0.5 × 10 −3/ ° C
R+r Hence, R1α 1 = − R2 α 2
⇒ . = 0.2 (10 + r )
21 R2 −4 × 10 −3
21
. =
10 + r = × 10 R1 0.5 × 10 −3
2 R2 8
∴ r = 10.5 − 10 = 0.5 Ω ⇒ =
R1 1
90 (b) Effective resistance,
98 (*) Let the three conductors having resistances R1, R2 and R3
40 × 120 4800
Reff = = = 30 Ω respectively, and the current drawn by them are 1A, 2 A and
120 + 40 160
3 A respectively, when connected in turn across a battery.
7 7  E  ∴ V = R1 V = 2R2 andV = 3R3
∴ Current, I = = = 0.2 A Q I = 
(30 + 5) 35  R + r V V
So, R1 = V, R2 = and R3 =
2 3
91 (b) Current, I = qf
When the conductors are connected in series, across the same
Given, q = 1.6 × 10 −19 C and f = 1019 battery, then
∴ I = 1.6 A V = I [R1 + R2 + R3]
I  V V
92 (c) vd = ⇒ V =I V + +
nAe  2 3 
Given, I = 2 A, n = 5.86 × 10 28 m−3  6 + 3 + 2 6
⇒ 1= I or I = A
2 −2 2 −19  6  11
A = πr = π (0.1 × 10 ) , e = 1. 6 × 10 C
∴ Drift velocity, v d = 0.68 × 10 −4 ms −1 l12 l22 l32
99 (d) R ∝ l 2 / m ⇒ R1 : R2 : R3 = : :
m1 m 2 m 3
 l d 2  1.5(0.31)2
93 (a) As, R ∝ l / d 2 ⇒ R2 = R1 2 12  = × 4.2 = 25.2 Ω 25 9 1
 l1 d 2  (0.155)2 = : : = 125 : 15 : 1
1 3 5
94 (d) Suppose, m rows are connected in parallel and each row I × 2R 2I
100 (b) We have, I1 = =
contains n identical cells (each cell having E = 1.5 V and r = 1Ω) 3R 3
For maximum current in the external resistance R, the 2
4I
necessary condition ∴ H1 = I12 R = ×R …(i)
9
R=
nr I ×R I
Also, I2 = =
m 3R 3
n ×1 I2
1.5 = ∴ H 2 = I22 (2R ) = × 2R …(ii)
m 9
1.5 m = n …(i) and 2
H 3 = I (1.5 R ) …(iii)
Total cells = 24 = n × m …(ii) From Eqs. (i), (ii) and (iii), we get
On solving Eqs. (i) and (ii), we get 4I 2 I2
H1 : H 2 : H 3 = × R : × 2R : I 2 × 1.5R
m=4 9 9
n=6 4 2
= : : 1.5 = 4 : 2 : 13.5
Therefore, 6 cells in each row are connected in series and 9 9
4 such rows are connected in parallel. = 8 : 4 : 27
95 (d) R1 = R0 (1 + αt ) 101 (a) A milliammeter will have greater resistance than ammeter
because (1) number of turns in the coil in milliammeter is
1 = R0 (1 + 0.00125 × 27) …(i)
increased to compensate low value of current to create large
2 = R0 (1 + 0.00125 × t ) …(ii) deflection. (2) milliammeter does not have usually a shunt in
From Eqs. (i) and (ii), we get parallel because main current is very small. Ammeter usually
t = 854° C = 1127 K have a shunt in parallel which makes overall resistance low.
for More Books Click HERE -> NEETpassionate.com
Current Electricity 267

102 (c) We know that, the resistance of any wire, On solving, we get
ρL (R1 − R2 )r 2 = (R1 − R2 )RR
R= 1 2
A ⇒ r = R1 R2
In given case, L1 = L , L2 = 2L , ρ and A are constants.
R1 L1 R L R 1 111 (b) Applying Kirchhoff’s law, 500I + 100I = 12
Hence, = ⇒ 1= ⇒ 1=
R2 L2 R2 2L R2 2 12 × 10 −2
So, I= = 2 × 10 −2A
103 (c) Power when connected in series, 6
1 1 1 Hence, VB = 100 (2 × 10 −2 ) = 2 V
= +
P P1 P2 112 (a) The full scale deflection current,
Given, P1 = 60 W and P2 = 100 W 25 mV
ig = ampere
60 × 100 G
Hence, P =
60 + 100 where, G is the resistance of the meter.
6000 The value of shunt required for converting it into ammeter of
⇒ P = ⇒ P = 37.5 W igG
160 range 25 A is S = .
e Eτ e τ V i − ig
104 (b) We know that, drift velocity, v d = =
m m l 25 mV
⇒ S= = 0.001 Ω
Here, vd ∝v 25
Given, condition V1 = V and V2 = 2V V2
vd V 113 (c) Power, P =
Hence, = ⇒ v ′d = 2v d = 2v R
v d′ 2V ∆P 2 × ∆V
For small variation, × 100% = × 100%
105 (c) Using,V = IR P V
= 2 × 2.5 = 5%
100 × 10 −3 = I × 10
Therefore, power would decrease by 5%.
⇒ I = 0.01A
114 (a) 6Ω
E i1
Also, I= 6Ω
R + R′
i2 12 Ω
2
⇒ = 0.01 ⇒ R′ = 190 Ω
10 + R′
106 (d) Specific resistance of a material is the characteristic 10 V
property, it does not depends upon the dimensions of 6 × 12 6 × 12
material, hence it does not depend upon length. Resistance, R = = =4Ω
6 + 12 18
107 (b) 7 Ω and 3 Ω are in series, so total 10 Ω and 10 Ω are in Total resistance, Req = 6 + 4 = 10 Ω
parallel, so total resistance across AC = 5 Ω. Now, 5 Ω and 5 V 10
Ω are in series, so they make 10 Ω. Now, 10 Ω and 10 Ω are Current, i = = = 1A
R 10
in parallel, so R net = 5 Ω.
The current in 12 Ω resistor is
108 (a) Equivalent resistance in series = 6 + 9 = 15 Ω
 R1   6  1
V 120 i2 = i   = 1×   ⇒ i2 = A
Current flow in circuit, i = = = 8A  R1 + R2   6 + 12 3
R 15
The potential difference in 12 Ω resistor,
Voltage in 6 Ω resistor,V = iR = 8 × 6 = 48 V
1
Power consumed by 6 Ω resistor, V = i × 12 = × 12 = 4 V
3
V 2 48 × 48
P = = 115 (b) Given, A = 0.3 m2, n = 2 × 10 25 /m3
R 6
= 8 × 48 = 384 W q = 3t2 + 5t + 2
109 (a) Current through each arm DAC and DBC = 1A i=
dq
= 6 t + 5 = 17 A (at t = 2 s)
VD − VA = 2 and VD − VB = 3 dt
We have current, i = ne Av d
∴ VA − VB = 1V
i 17
110 (b) Heat = i 2Rt Drift velocity, v d = =
ne A 2 × 10 25 × 1.6 × 10 −19 × 0.3
2 2
 E   E  17
∴   R1 =   R2 [Q t is the same] = = 1.77 × 10 −5 ms −1
 R1 + r   R2 + r  0.96 × 10 −6
for More Books Click HERE -> NEETpassionate.com
268 OBJECTIVE Physics Vol. 2

116 (b) To convert a galvanometer into an ammeter, a resistance 123 (a) As the current in heating filament increases, it gets more
(shunt) is required to be connected in parallel to the heated. Hence, its temperature increases and resistance also
galvanometer. increases, due to which the current decreases.
Gig 50 × 100 × 10 −6 PP
By using, S = = ≈ 5 × 10 −4 Ω 124 (b) In series, P = 1 2
i − ig (10 − 100 × 10 −6 ) P1 + P2
Given, P1 = P2 = 60
117 (d) By using the relation,
60 × 60
E1 l1 + l2 (8 + 2) 5 where, E ′ + E ′′ = E1 ∴ Power, P = = 30 W
= = =   60 + 60
E 2 l1 − l2 (8 − 2) 3  and E ′− E ′′ = E 2 
2 2 20
125 (b) Ig = A= × 1000 mA = mA
118 (b) The current distribution in the circuit can be shown as 300 300 3
E Ig = 6.67 mA
(i – i1 – i2) As range of ammeter cannot be decreased but can be
F i
increased only, therefore the instrument cannot be converted
B to measure the range 1 mA.
i2 (i1 + i2)
P R
(i – i1)
126 (a) The balanced condition for Wheatstone bridge is = .
i Q S
C
A As is obvious from the given values, no current flows through
galvanometer.
i1 Now, P and R are in series, so
D
Applying Kirchhoff’s law in mesh ABCDA, Resistance, R1 = P + R = 10 + 15 = 25 Ω
− 10 (i − i1) − 10 i2 + 20 i1 = 0 Similarly, Q and S are in series, so
3i1 − i2 = i …(i) Resistance, R2 = Q + S = 20 + 30 = 50 Ω
and in mesh BEFCB, Net resistance of the network, as R1 and R2 are in parallel,
25 × 50 50
−20 (i − i1 − i2 ) + 10 (i1 + i2 ) + 10i2 = 0 Rnet = = Ω
75 3
⇒ 3i1 + 4i2 = 2i …(ii)
V 6×3
From Eqs. (i) and (ii), we get i= = = 0.36 A
Rnet 50
2i i 2i
i1 = , i2 = ⇒ iAD = i1 = I
5 5 5 127 (b) Drift velocity, v d =
J neA
119 (d) The drift velocity, v d = 5
ne ⇒ vd =
(5 × 10 ) × (1.6 × 10 −19 ) × (4 × 10 −6 )
26
i
(where, J is current density = )
A = 1.56 × 10 −2 ms −1
So, v d ∝ J 128 (c)
A A
i 2i i
∴ J1 = and J2 = = = J1
A 2A A
So, (v d )1 = (v d )2 = v r r r r/3
r r
120 (b) As resistance, R ∝ length Þ
12 r
∴Resistance of each arm = =4Ω
3 B C B C
r/2
So, effective resistance between any two corners, r
4 × 8 32 8 Resistance between points A and B,
Reff = = = Ω
4 + 8 12 3  5 
r × r 
V 20 r r  6  5
121 (a) We have, R = = =8Ω RAB = r ||  +  = = r
 3 2  5  11
I 2.5 r + r 
∆R ∆V ∆I 1 0.50 1  6 
= + = + =
R V I 20 2.5 4 Resistance between points B and C,
R 8 r 4r
∆R = = = 2 Ω ×
4 4 r  r 4
RBC = || r +  = 2 3 = r
⇒ R = (8 ± 2) Ω 2  3 r
+
4r 11
nr 2 3
122 (c) We have R = , so grouping of cells depends upon the Resistance between points C and A,
m
relative values of internal and external resistance.
for More Books Click HERE -> NEETpassionate.com
Current Electricity 269

 r 3r  So, the reading of ammeter,


 × 
r r  3 2 3 V 2 6 3
RCA = ||  + r  = = r i= = = = A
3 2  r
+
3r 11 R 8/ 3 8 4
3 2
134 (d) The resistances of branches ABC and ADC are in parallel.
RAB : RBC : RCA = 5 : 4 : 3
The current entering at node A will equally divided in each
130 (a) Given circuit is a balanced Wheatstone bridge. So, diagonal parallel branch.
resistance of 2 Ω will be ineffective. ∴ VA − VB = IR = 1 × 2 = 2 V
Equivalent resistance of upper arm = 2 + 2 = 4 Ω Point A is at greater potential with respect to point B, so
Equivalent resistance of lower arm = 2 + 2 = 4 Ω VA − VB = + 2 V
4× 4
∴ RAB = = 2Ω 135 (b) Equivalent potential in the given parallel circuit is given as
4+ 4
E1 E 2
+
131 (b) In open circuit, current through the circuit i = 0 r r2 E1r2 + E 2r1
E eq = 1 =
Potential difference between x and y will be 1 1 r1 + r2
+
V = E = 120 V r1 r2
1 1 1 1 Here, E1 = 2 V, E 2 = 1V, r1 = 1 Ω
132 (d) = + +
R′ 10 10 20 and r2 = 2 Ω
R 2 × 2 + 1× 1 4 + 1 5
0.5 A
⇒ E eq = = = V
2+1 3 3
25 V R¢ 136 (c) Here are two batteries with emf E each and the internal
resistances r1 and r2, respectively.
E E
20
⇒ R′ = = 4Ω r1 r2
5
25 I
Now, using Ohm’s law, i =
R + R′ R
25
⇒ 0.5 = Hence, we have I (R + r1 + r2 ) = 2E
R+4
2E
25 Thus, I = … (i)
⇒ R + 4= R + r1 + r2
0.5
⇒ R = 50 − 4 = 46 Ω Now, the potential difference across the first cell would be
0.5 × 5 2 .5 equal toV = E − Ir1. From the question,V = 0, hence
Current through 20 Ω resistor = = = 0.1A 2Er1
20 + 5 25 E = Ir1 = [from Eq. (i)]
R + r1 + r2
Potential difference across middle resistor
= Potential difference across 20 Ω = 20 × 0.1 = 2 V ⇒ R + r1 + r2 = 2r1
Hence, R = r1 − r2
133 (b) The given circuit can be shown as the resistance between
2 137 (b) The net resistance,
A and B = Ω.
3 12 × 4 48
R= + 2= + 2= 5 Ω
Total resistance of the circuit 12 + 4 16
2Ω 2V Electric current,
A E 12
i= = = 2A
2Ω R + r 5+ 1
A B From current division rule,
2Ω  12 
i2 =  × 2
 12 + 4
2Ω 3
= × 2 = 1.5 A
4
2 8
R =2+ = Ω and i1 = 2 − 1.5 = 0.5 A
3 3
for More Books Click HERE -> NEETpassionate.com

CHAPTER
04

Magnetic Effect of
Current and Moving
Charges
In previous unit, we have studied about the electric current, its measurement and
the thermal effects of current. In this unit, we shall study about magnetic effects
of current. Earlier, it was thought that there is no connection between electricity
and magnetism. However in the year 1820, Oersted realised that electricity and
magnetism were related to each other. He showed experimentally that the electric
current through a straight wire causes noticeable deflection of the magnetic
compass needle held near the wire.
Inside
He also found that the iron fillings sprinkled around the wire arrange themselves 1 Magnetic field
in concentric circles with wire as the centre in the plane perpendicular to the Biot-Savart’s law
wire. This shows that the magnetic field is associated with a moving charge or a (Magnetic field due
to a current carrying conductor)
current carrying conductor. The branch of physics which deals with the
Applications of
magnetism due to electric current is called electromagnetism. Biot-Savart’s law
2 Ampere’s circuital law
MAGNETIC FIELD Applications of Ampere’s
circuital law
The space in the surroundings of a magnet, in which magnetic influence of 3 Force on a moving charge in a
moving charges or a current carrying conductor can be experienced is called uniform magnetic field
magnetic field. Magnetic field is a vector quantity and is denoted by B. The SI Motion of a charged particle
unit of magnetic field is weber/m 2 or tesla (T) and its CGS unit is gauss or in combined electric and
maxwell/cm 2 . magnetic fields : Lorentz force
Cyclotron
1 tesla = 10 4 gauss 4 Force on a current carrying
conductor in a magnetic field
Magnetic field due to moving charges Force between two parallel
current carrying conductors
(Oersted’s experiment) Magnetic force between two
Oersted found experimentally that a magnetic field is established around a current moving charges
carrying conductor just as it occurs around a magnet. In this experiment, Oersted Magnetic dipole moment
placed a magnetic compass near a current carrying conductor and observed that The moving coil galvanometer
(MCG)
the compass needle shows deflection. Also, when the current is reversed, the
needle shows deflection in opposite direction.
for More Books Click HERE -> NEETpassionate.com
Magnetic Effect of Current and Moving Charges 271

Key Key Key ● inversely proportional to the square of distance between


Battery (open) (closed) (closed)
+ – + – the element and the point, i.e. dB ∝ 1/r 2 …(iv)
+ –

Conducting Combining all the above four relations, we get


wire
Idl sinθ
A B A B A B dB ∝
N S r2
S N
S N This relation is called Biot-Savart’s law.
Magnetic compass Compass showing Compass showing If conductor is placed in air or vacuum, then magnitude of
with no deflection deflection in deflection in
one direction opposite direction
magnetic field is given by
(a) (b) (c) µ 0 Idl sinθ
Fig. 4.1 Oersted’s experiment |dB | =
4π r 2
Thus, from the above figures, he concluded that a
magnetic field is established around a current carrying µ0
where, is the proportionality constant.
wire due to which such deflections are visible. 4π
Oersted’s experiment shows the connectivity between the µ 0 is called the permeability of free space (or vacuum).
magnetic field and electric current and as electric current
In SI unit, µ 0 = 4π × 10 −7 Tm/A or Wb/A-m and its
means moving charges, he also concluded that moving
µ
charges also produce magnetic field in their surroundings. dimensions are [MLT −2A −2 ]. The value of 0 in CGS

Biot-Savart’s law (Magnetic field due system is unity.

to a current carrying conductor) In vector form, Biot-Savart’s law can be written as


Id l × r µ 0 Id l × r
Biot-Savart’s law is an experimental law predicted by Biot dB ∝ = ⋅
and Savart. This law deals with the magnetic field |r |3 4π r3
produced at a point due to a small current element (a part The direction of dB is represented by the right hand screw
of any conductor carrying current). rule or right hand thumb rule.
Consider a wire XY carrying current I. Let dl be the Similarly, magnetic field induction at point P due to
infinitesimal element of the conductor, dB be the magnetic current through entire wire is
field at point P at a distance r from the element.
µ Id l × r
Y B=∫ 0 ⋅
4π r3
θ
I dl µ 0 Id l sin θ
r or B= ∫ 4π ⋅ r2
Current P dB
element I
Biot-Savart’s law in a medium
If the conductor is placed in a medium, then vector form
X of Biot-Savart’s law is given as
Fig. 4.2 A long current carrying conductor
µ Idl × r µ 0 µ r I dl × r
Biot-Savart’s law states that, the magnitude of the dB = =
4π | r |3 4π r3
magnetic field induction dB at the point P due to the
µ
current element is where, µ r = relative permeability = =1
● directly proportional to the current flowing through the
µ0
conductor, i.e. dB ∝ I …(i) (for air or vacuum)
● directly proportional to the length of the element, and µ = absolute permeability of the medium.
i.e. dB ∝ dl …(ii) Biot-Savart’s law in terms of current density J
● directly proportional to the sine of angle between the
µ 0 ( J × r)  I Idl Idl 
length of element and line joining the element to the dB = ⋅ dV Q J = = = 
4π r3  A Adl dV 
point, i.e. dB ∝ sin θ …(iii)

You might also like